You are on page 1of 171

2014 BAR EXAMINATIONS

REMEDIAL LAW NOTES Part III


By:

ELEUTERIO L. BATHAN
Presiding Judge, RTC, Br. 92, (NCJR) Quezon City
Acting Presiding Judge, RTC, Br. 5, Lemery, Batangas
Judge Designate, RTC, Br. 39, Calapan City, Or. Mindoro
Judge Designate, RTC, Br. 40, Calapan City, Or. Mindoro
Former Executive/Presiding Judge, MTCC, Br. 2, Batangas City
Former Acting Judge: MTCC, Br. 1, Batangas City; MTC, Lian,
Batangas; Sablayan, Occidental Mindoro;
Former Judge Designate, MTC, Romblon, Romblon
Law Professor, Remedial Law Review and Political Law
University of Batangas (U.B.) (Batangas & Lipa City Campuses)
MCLE Lecturer
Author, Handbook on Local Governance (Rex Book Store, Inc.)
Awardee, Judicial Excellence (2014), Rotary Club of Sta. Mesa
District 3780, Quezon City
Outstanding Graduate (2014), SSC-R, College of Law
Awardee, Quezon City RTCJA 2012 Outstanding Judge
Former Member, Sangguniang Bayan, San Jose, Batangas
Former Law Practitioner
A.B.; Ll.B., San Sebastian College-R (Manila)

---0---

REMEDY OF PLAINTIFF IF DEFENDANTS


MOTION TO DISMISS COMPLAINT
IS GRANTED
If defendants motion to dismiss is granted, the remedy of the
plaintiff is to appeal the order of dismissal by way of filing a notice of
appeal, because an order of dismissal is a final order (See Rules 40 and
41, 1997 Revised Rules of Civil Procedure).
An order of dismissal, whether correct or not, is a final order. It is
not interlocutory because the proceedings were terminated; it leaves
nothing more to be done by the lower court. Therefore, the remedy of the
plaintiff is to appeal the order (Dael vs. Spouses Beltran, 4).
The plaintiff may resort first to the filing of a motion for
reconsideration before filing a notice of appeal. In situation like this, filing
a motion for reconsideration is optional. It is discretionary on the part of
the aggrieved party or plaintiff.
If he chooses to file answer, he may avail of an extension of time
to file one (See above-discussions; See also Virata vs. Sandiganbayan).

---0---

REMEDY OF DEFENDANT IF HIS


MOTION TO DISMISS COMPLAINT IS DENIED
If the defendants motion to dismiss is denied, the following are
the remedies available to the defendant:
1. upon denial of the motion for reconsideration to the order
of denial, he may file a petition for certiorari under Rule
65; or to
2. file an answer

Though filing a petition for certiorari under Rule 65 is the most


dangerous option, but still, defendant can avail of this remedy. The danger
or the risk is that instead of focusing on the alternative mode which is the
filing of an answer, defendant, upon plaintiffs motion might be declared
by the court in default for failure to file an answer. So, if he chooses to
resort to the filing of certiorari under Rule 65, and in order to avoid the
possibility of declaring him in default later on, he shall see to it that his
petition should be accompanied with an application for ancillary relief of
temporary restraining order and/or preliminary injunction, but also take
note of the limitations provided under A.M. No. 07-7-12-SC dated
December 4, 2007 [effective December 27, 2007] amending Rule 65 of the
1997 Revised Rules of Civil Procedure).
As a general rule, an order denying a motion to dismiss is merely
interlocutory and cannot be subject of appeal until final judgment or order
is rendered (Newsweek, Inc vs. IAC,).
It is interlocutory in the sense that it does not finally dispose of the
case, and, in effect, directs the case to proceed until final adjudication by
the court (Marmo et al., vs. Anacay,).
The ordinary procedure to be followed in such a case is to file an
answer, go to trial and if the decision is adverse, reiterate the issue on
appeal from the final judgment, but this may be subject to certain
exceptions (Newsweek, Inc vs. IAC,).

EXCEPTIONS:
Only when the court issues an order outside or in excess of
jurisdiction or with grave abuse of discretion, and the remedy of appeal
would not afford adequate and expeditious relief, will certiorari be
considered an appropriate remedy to assail an interlocutory order (Heirs of
Hinog vs. Melicor,).
If the court, in denying the motion to dismiss, acts without or in
excess of jurisdiction or with grave abuse of discretion, then certiorari or
prohibition lies. The reason is that it would be unfair to require the
defendant to undergo the ordeal and expense of a trial if the court has no

jurisdiction over the subject matter, or is not the court of proper venue, or
if the denial of the motion to dismiss is made with grave abuse of
discretion or a whimsical and capricious exercise of judgment. In such
cases, the ordinary remedy of appeal cannot be plain and adequate. The
following are a few examples of the exceptions to the general rule
(Newsweek, Inc vs. IAC,).
In De Jesus vs. Garcia, , upon the denial of a motion to dismiss
based on lack of jurisdiction over the subject matter, the High Court
granted the petition for certiorari and prohibition against the City Court of
Manila and directed the respondent court to dismiss the case.
However, in Bank of America vs. CA, , it was held that the remedy
of the defendant is to file an answer to the complaint, proceed to trial and
await judgment before making an appeal instead of petition of certiorari
under Rule 65.
The High Court in Bank of America:
It is a well-settled rule that the order denying the
motion to dismiss cannot be the subject of petition for
certiorari. Petitioners should have filed an answer to the
complaint, proceed to trial and await judgment before
making an appeal. As repeatedly held by this Court:
An order denying a motion to dismiss is
interlocutory and cannot be the subject of the
extraordinary petition for certiorari or
mandamus. The remedy of the aggrieved party
is to file an answer and to interpose as defenses
the objections raised in his motion to dismiss,
proceed to trial, and in case of an adverse
decision, to elevate the entire case by appeal in
due course. xxx Under certain situations,
recourse to certiorari or mandamus is
considered appropriate, i.e., (a) when the trial
court issued the order without or in excess of
jurisdiction; (b) where there is patent grave
abuse of discretion by the trial court; or (c)

appeal would not prove to be a speedy and


adequate remedy as when an appeal would not
promptly relieve a defendant from the injurious
effects of the patently mistaken order
maintaining the plaintiffs baseless action and
compelling the defendant needlessly to go
through a protracted trial and clogging the court
dockets by another futile case.
Records show that the trial court acted within its
jurisdiction when it issued the assailed Order denying
petitioners motion to dismiss. Does the denial of the
motion to dismiss constitute a patent grave abuse of
discretion? Would appeal, under the circumstances, not
prove to be a speedy and adequate remedy? We will
resolve said questions in conjunction with the issues raised
by the parties.
First issue. Did the trial court commit grave abuse of
discretion in refusing to dismiss the complaint on the
ground that plaintiffs have no cause of action against
defendants since plaintiffs are merely stockholders of the
corporations which are the registered owners of the vessels
and the borrowers of petitioners?
No. Petitioners argument that private respondents,
being mere stockholders of the foreign corporations, have
no personalities to sue, and therefore, the complaint should
be dismissed, is untenable. A case is dismissible for lack of
personality to sue upon proof that the plaintiff is not the
real party-in-interest. Lack of personality to sue can be
used as a ground for a Motion to Dismiss based on the fact
that the complaint, on the face thereof, evidently states no
cause of action. In San Lorenzo Village Association, Inc.
vs. Court of Appeals, this Court clarified that a complaint
states a cause of action where it contains three essential
elements of a cause of action, namely: (1) the legal right of
the plaintiff, (2) the correlative obligation of the defendant,
and (3) the act or omission of the defendant in violation of

said legal right. If these elements are absent, the complaint


becomes vulnerable to a motion to dismiss on the ground of
failure to state a cause of action. To emphasize, it is not the
lack or absence of cause of action that is a ground for
dismissal of the complaint but rather the fact that the
complaint states no cause of action. Failure to state a
cause of action refers to the insufficiency of allegation in
the pleading, unlike lack of cause of action which refers
to the insufficiency of factual basis for the action. Failure
to state a cause of action may be raised at the earliest
stages of an action through a motion to dismiss the
complaint, while lack of cause of action may be raised
any time after the questions of fact have been resolved on
the basis of stipulations, admissions or evidence presented.
In the case at bar, the complaint contains the three
elements of a cause of action. It alleges that: (1) plaintiffs,
herein private respondents, have the right to demand for an
accounting from defendants (herein petitioners), as trustees
by reason of the fiduciary relationship that was created
between the parties involving the vessels in question; (2)
petitioners have the obligation, as trustees, to render such
an accounting; and (3) petitioners failed to do the same.
Petitioners insist that they do not have any obligation
to the private respondents as they are mere stockholders of
the corporation; that the corporate entities have juridical
personalities separate and distinct from those of the private
respondents. Private respondents maintain that the
corporations are wholly owned by them and prior to the
incorporation of such entities, they were clients of
petitioners which induced them to acquire loans from said
petitioners to invest on the additional ships.
We agree with private respondents. As held in the
San Lorenzo case,
xxx assuming that the allegation of facts
constituting plaintiffs cause of action is not as

clear and categorical as would otherwise be


desired, any uncertainty thereby arising should
be so resolved as to enable a full inquiry into the
merits of the action.
As this Court has explained in the San Lorenzo case,
such a course, would preclude multiplicity of suits which
the law abhors, and conduce to the definitive determination
and termination of the dispute. To do otherwise, that is, to
abort the action on account of the alleged fatal flaws of the
complaint would obviously be indecisive and would not
end the controversy, since the institution of another action
upon a revised complaint would not be foreclosed.

In Enriquez vs. Macadaeg, , upon the denial of a motion to


dismiss based on improper venue, this Court granted the petition for
prohibition and enjoined the respondent judge from taking cognizance of
the case except to dismiss the same.
In Manalo vs. Mariano, , upon the denial of a motion to dismiss
based on bar by prior judgment, this Court granted the petition for
certiorari and directed the respondent judge to dismiss the case.
In Yuviengco vs. Dacuycuy, , upon the denial of a motion to
dismiss based on the Statute of Frauds, the High Court granted the petition
for certiorari and dismissed the amended complaint.
Take note however, that in Marmo et al., vs. Anacay, , the High
Court ruled that the trial court did not grave abuse its discretion when it
denied the motion to dismiss the complaint for failure of the plaintiff to
implead indispensable parties.
---0---

EFFECT OF DISMISSAL

IF MOTION TO DISMISS IS GRANTED


If the dismissal is based on the grounds that: (1) the cause of action
is barred by a prior judgment or by the statute of limitations (Paragraph
(f), Section 1, Rule 16, 1997 Revised Rules of Civil Procedure); (2) the
claim or demand set forth in the plaintiffs pleading has been paid, waived,
abandoned, or otherwise extinguished (Paragraph (h), Section 1, Rule 16,
1997 Revised Rules of Civil Procedure); and (3) the claim on which the
action is founded is unenforceable under the provisions of the statute of
frauds; and (Paragraph (i), Section 1, Rule 16, 1997 Revised Rules of
Civil Procedure), subject to the right of appeal, shall bar the refiling of
the same action or claim (Section 5, Rule 16, 1997 Revised Rules of Civil
Procedure). The dismissal is with prejudice.

Strongworld Construction Corporation vs. Judge Perello, said:


Section 1, Rule 16 of the 1997 Revised Rules of
Civil Procedure enumerates the grounds for which a
Motion to Dismiss may be filed, viz.:
SECTION 1. Grounds. Within the time
for but before filing the answer to the complaint
or pleading asserting a claim, a motion to
dismiss may be made on any of the following
grounds:
(a)
(b)
(c)
(d)

(e)

That the court has no jurisdiction over the


person of the defending party;
That the court has no jurisdiction over the
subject matter of the claim;
That venue is improperly laid;That the
plaintiff has no legal capacity to sue;
That there is another action pending
between the same parties for the same
cause;
That the cause of action is barred by a
prior judgment or by the statute of
limitations;

(f)
(g)

(h)

(i)

That the pleading asserting the claim states


no cause of action;
That the claim or demand set forth in the
plaintiffs pleading has been paid, waived,
abandoned, or otherwise extinguished;
That the claim on which the action is
founded is unenforceable under the
provisions of the statute of frauds; and
That a condition precedent for filing the
claim has not been complied with.

Section 5 of the same Rule, recites the effect of a


dismissal under Sections 1(f), (h), and (i), thereof, thus:
SEC. 5. Effect of dismissal. Subject to
the right of appeal, an order granting a motion to
dismiss based on paragraphs (f), (h), and (i) of
section 1 hereof shall bar the refiling of the
same action or claim.
Briefly stated, dismissals that are based on the
following grounds, to wit: (1) that the cause of action is
barred by a prior judgment or by the statute of limitations;
(2) that the claim or demand set forth in the plaintiffs
pleading has been paid, waived, abandoned or otherwise
extinguished; and (3) that the claim on which the action is
founded is unenforceable under the provisions of the statute
of frauds, bar the refiling of the same action or claim.
Logically, the nature of the dismissal founded on any of the
preceding grounds is with prejudice because the
dismissal prevents the refiling of the same action or claim.
Ergo, dismissals based on the rest of the grounds
enumerated are without prejudice because they do not
preclude the refiling of the same action.
Verily, the dismissal of petitioners Complaint by the
court a quo was not based on any of the grounds specified
in Section 5, Rule 16 of the 1997 Revised Rules of Civil
Procedure; rather, it was grounded on what was

encapsulated in Section 1(g), Rule 16 of the 1997 Revised


Rules of Civil Procedure. As the trial court ratiocinated in
its 9 January 1998 Order, the Complaint is not prosecuted
by the proper party in interest. Considering the heretofore
discussion, we can say that the order of dismissal was
based on the ground that the Complaint states no cause of
action.
For this reason, the dismissal of petitioners
Complaint cannot be said to be a dismissal with prejudice
which bars the refiling of the same action.
As has been earlier quoted, Section 1(h), Rule 41 of
the 1997 Revised Rules of Civil Procedure mandates that
no appeal may be taken from an order dismissing an action
without prejudice. The same section provides that in such
an instant where the final order is not appealable, the
aggrieved party may file an appropriate special civil action
under Rule 65.
-0-0-0DISMISSAL ON VARIOUS GROUNDS AND ITS EFFECT
(WITH PREJUDICE AND WITHOUT PREJUDICE)
On filing of initiatory pleading, If act of lawyer or counsel
constitute willful and deliberate forum shopping, shall be a ground for the
summary dismissal of the case with prejudice (Section 5, Rule 7, 1997
Rules of Court).
Regular Procedure (1997 Rules of Court): plaintiffs failure to
appear at the pre-trial conference, the dismissal is with prejudice, unless
otherwise ordered by the court (Section 5, Rule 18). General Rule:
dismissal is always with prejudice, unless the order says that dismissal is
without prejudice. If the order is silent, the dismissal is with prejudice;
PLAINTIFFS failed defendant may be allowed to present evidence on
his counter-claim, if any; DEFENDANTS failed plaintiffs presentation
of ex-parte evidence, and the court to render judgment on the basis
thereof.

Summary Procedure: plaintiffs failure to appear at the


preliminary conference can cause the dismissal of the action. The
dismissal is with prejudice, unless otherwise ordered by the court (Apply
Section 5, Rule 18, 1997 Rules of Court, suppletory application). General
Rule: dismissal is always with prejudice, unless the order says that
dismissal is without prejudice. If the order is silent, the dismissal is with
prejudice.
Plaintiffs failure to file pre-trial brief (in ordinary cases) has the
same effect of failure to appear at the pre-trial conference (See: Section 6,
Rule 18, 1997 Rules of Court).
Plaintiffs failure to file preliminary conference brief (pre-trial
brief under summary procedure) has the same effect with failure to appear
at the preliminary conference (See: Rule 18, 1997 Rules of Court
suppletory application).
Rule on Small Claims: Failure of plaintiff to appear at the trial can
cause the dismissal of the claim. Take note: The dismissal is always
without prejudice (Section 18, Rule on Small Claim).
Rules of Procedure for Environmental Cases repeated and
unjustified failure of plaintiff to appear at the pre-trial conference, or to
file pre-trial brief, the court shall dismiss the case. The dismissal shall be
without prejudice (Section 7, Rule 3 of the Rule).
Lack of certification against forum shopping under Section 5,
Rule 7, 1997 Rules, shall be cause for the dismissal of the case dismissal
is without prejudice, unless otherwise provided.
---0---

DEFENDANT MAY FILE MOTION


FOR EXTENSION OF TIME TO ANSWER

This discussion is confined only to all (civil) cases governed by the


rules on regular procedure, excluding the cases governed by the Rules on
Summary Procedure (See separate discussion on the matter).
If after plaintiff has complied with the order of the court requiring
him to submit a bill of particulars, and/or defendants motion to dismiss
(and/or the motion for reconsideration thereto) is denied, the defendant
has three (3) more remaining remedial options. The defendant may either:
(1) may still ask for extension of time to file answer; or
(2) file an answer; or
(3) he may deliberately disregard the complaint by inaction.
The defendant may still ask for an extension of time to file answer
or motion to dismiss. This is a non-litigious motion which can be done and
filed ex-parte. Some trial courts do not anymore rule or resolve the
motion for extension of time, and that is, equivalent to allowance. If that is
the case, it is incumbent upon the movant to file the intended pleading on
or before the expiration of the period prayed for within which to file the
same.
The rule says:
1. Upon motion and on such terms as may be just, the court may
extend the time to plead provided in these Rules (Section 11, Rule 11,
1997 Revised Rules of Civil Procedure).
2. The court may also, upon like terms, allow an answer or other
pleading to be filed after the time fixed by the Rules (Paragraph 2,
Section 11, Rule 11, 1997 Revised Rules of Civil Procedure).
---0--FILING AN ANSWER TO THE COMPLAINT:

If the defendants motion to dismiss is denied his other remedial


option aside from filing a certiorari petition (under Rule 65) is to file
answer.

A. When to file answer


The defendant shall file his answer to the complaint within fifteen
(15) days after service of summons, unless a different period is fixed by
the court (Section 1, Rule 11, 1997 Revised Rules of Civil Procedure).
Where the defendant is a foreign private juridical entity and service
of summons is made on the government official designated by law to
receive the same, the answer shall be filed within thirty (30) days after
receipt of summons by such entity (Section 2, Rule 11, 1997 Revised
Rules of Civil Procedure).
When the plaintiff files an amended complaint as a matter of right,
the defendant shall answer the same within fifteen (15) days after being
served with a copy thereof (Section 3, Rule 11, 1997 Revised Rules of
Civil Procedure).
Where its filing is not a matter of right, the defendant shall answer
the amended complaint within ten (l0) days from notice of the order
admitting the same. An answer earlier filed may serve as the answer to the
amended complaint if no new answer is filed (Paragraph 2, Section 3,
Rule 11, 1997 Revised Rules of Civil Procedure).

B. Defenses and objections not pleaded waived


Defenses and objections not pleaded either in a motion to dismiss
or in the answer are deemed waived (Section 1, Rule 9, 1997 Revised
Rules of Civil Procedure).

C. Contents of answer

An answer shall contain in a methodical and logical form, a plain,


concise and direct statement of the ultimate facts on which the defendant
relies for his defense, omitting the statement of mere evidentiary facts
(Section 1, Rule 8, 1997 Revised Rules of Civil Procedure). If a defense
relied on is based on law, the pertinent provisions thereof and their
applicability to him shall be clearly and concisely stated (Paragraph 2,
Section 1, Rule 8, 1997 Revised Rules of Civil Procedure).
A defendant may set forth two or more statements of defense
alternatively or hypothetically, either in one defense or in separate
defenses (Section 2, Rule 8, 1997 Revised Rules of Civil Procedure).
Facts showing the capacity of a party to sue or be sued or the
authority of a party to sue or be sued in a representative capacity or the
legal existence of an organized association of person that is made a party,
must be averred. A party desiring to raise an issue as to the legal existence
of any party or the capacity of any party to sue or be sued in a
representative capacity, shall do so by specific denial, which shall include
such supporting particulars as are peculiarly within the pleader's
knowledge (Section 4, Rule 8, 1997 Revised Rules of Civil Procedure).
In all averments of fraud or mistake the circumstances constituting
fraud or mistake must be stated with particularity. Malice, intent,
knowledge, or other condition of the mind of a person may be averred
generally (Section 5, Rule 8, 1997 Revised Rules of Civil Procedure).
Whenever a defense is based upon a written instrument or
document, the substance of such instrument or document shall be set forth
in the answer, and the original or a copy thereof shall be attached to the
answer as an exhibit, which shall be deemed to be a part of the answer, or
said copy may with like effect be set forth in the pleading (Section 7, Rule
8, 1997 Revised Rules of Civil Procedure).
When a defense is founded upon a written instrument, copied in or
attached to the corresponding pleading as provided in the preceding
section, the genuineness and due execution of the instrument shall be
deemed admitted unless the adverse party, under oath specifically denies
them, and sets forth what he claims to be the facts, but the requirement of
an oath does not apply when the adverse party does not appear to be a

party to the instrument or when compliance with an order for an inspection


of the original instrument is refused (Section 8, Rule 8, 1997 Revised
Rules of Civil Procedure). In pleading an official document or official act,
it is sufficient to aver that the document was issued or the act done in
compliance with law (Section 9, Rule 8, 1997 Revised Rules of Civil
Procedure).
A defendant must specify each material allegation of fact the truth
of which he does not admit and, whenever practicable, shall set forth the
substance of the matters upon which he relies to support his denial. Where
a defendant desires to deny only a part of an averment, he shall specify so
much of it as is true and material and shall deny only the remainder.
Where a defendant is without knowledge or information sufficient to form
a belief as to the truth of a material averment made to the complaint, he
shall so state, and this shall have the effect of a denial (Section 10, Rule 8,
1997 Revised Rules of Civil Procedure).
Material averment in the complaint, other than those as to the
amount of unliquidated damages, shall be deemed admitted when not
specifically denied. Allegations of usury in a complaint to recover
usurious interest are deemed admitted if not denied under oath (Section
11, Rule 8, 1997 Revised Rules of Civil Procedure).

DENIAL AND CONTESTING THE GENUINENESS AND DUE


EXECTION OF ACTIONABLE DOCUMENT
A. How to deny the genuineness and due execution of an
actionable document?
The defendant is required to make an effective specific denial. This
means that the defendant must declare under oath that he did not sign the
document or that it is otherwise false or fabricated. Neither does the
statement of the answer to the effect that the instrument was procured by
fraudulent representation raise any issue as to its genuineness or due
execution. On the contrary such a plea is an admission both of the
genuineness and due execution thereof, since it seeks to avoid the
instrument upon a ground not affecting either (Songco vs. Sellner,).

B. Case No. 1
Plaintiff alleged in the complaint that defendant is indebted to the
former in the amount of P1M attaching in the complaint several annexes
including the photocopy of the promissory note. In his answer, he alleged
that he has never signed the promissory note attached to the complaint in
his personal and/or individual capacity, thus ineffective, unenforceable
and void for lack of valid consideration. In his verification, he stated that:
(1) that the defendant, after having been duly sworn to in accordance with
law, hereby depose and declare that she is the named defendant in the
above-entitled case; he has cause the preparation of the answer upon facts
and figures supplied by his to his retained counsel; have read each and
every allegations contained therein and hereby certify that the same are
true and correct of her own knowledge and information.
At the pre-trial, plaintiff was able to prove that the defendant
received the amount of P1M as loan after signing the Promissory Note
(Annex A), that defendant, upon receipt of the demand letter made a reply
seeking an extension to pay her obligation. On the Formal Offer of
Exhibits Plaintiff sought the admission of the duplicate original of the PN
on the ground that the original copy could no longer be found. The trial
court initially admitted into evidence the duplicate original of the PN, and
allowed Defendant to amend her answer to conform with this new
evidence. Upon Defendants motion for reconsideration arguing that the
duplicate Original PN was not properly identified and there were markings
in the photocopy which were not contained in duplicate original, the trial
court granted the MR and dismissed the case on the ground that Plaintiff
had no longer possessed any proof of Defendants alleged indebtedness.
The trial court found that there can be no dispute to the fact that the
allegations in the answer of defendant, she denied generally and
specifically under oath the genuineness and due execution of the
promissory note and by way of special and affirmative defenses herein
states that she never signed the promissory note attached to the complaint
in his personal and/or individual capacity. She also deny generally and
specifically the rest of the allegations. It would be considered that there is
a sufficient compliance of the requirement of the law for specific denial.
Is the denial specific?

Solidbank vs. Del Monte Motor Works, Inc., said defendants


denial is not specific. Section 8, Rule 8, 1997 Rules of Court provides that
when an action or defense is founded upon a written instrument, copied in
or attached to the corresponding pleading as provided in the preceding
section, the genuineness and due execution of the instrument shall be
deemed admitted unless the adverse party, under oath, specifically
denies them and sets forth what he claims to be the facts.

Is secondary evidence still needed?


Since Defendant failed to deny specifically the execution of the
promissory note, there was no need for the Plaintiff to present the original
of the promissory note in question. Defendants judicial admission with
respect to the genuineness and execution of the promissory note
sufficiently established her liability to Plaintiff regardless of the fact that
Plaintiff failed to present the original of said note. Therefore, there is thus
no need of proof of execution and authenticity with respect to the loan
document because of Defendants implied admission of loan transaction.
There is no need to comply with what Section 22, Rule 132 of the Rules of
Court which requires that before a private documents can be received in
evidence, presentation and examination of witnesses to testify to prove its
due execution and authenticity (Solidbank vs. Del Monte Motor Works,
Inc.,).
C. Case No. 2
Defendants answer contained the following:
1. The allegations in par. 2, Complaint, on the
existence of the alleged loan of P2-M, and the purported
documents evidencing the same, only the signature
appearing at the back of the promissory note, Annex A
seems to be that of herein defendant. However, as to any
liability arising therefrom, the receipt of the said amount of
P2-M shows that the amount was received by another
person, not the herein defendant. Hence, no liability

attaches and as further stated in the special and affirmative


defenses that, assuming the promissory note exists, it does
not bind much less is there the intention by the parties to
bind the herein defendant. In other words, the documents
relative to the loan do not express the true intention of the
parties.
2. (Verification) I, Defendant, of age, am the
defendant in this case, that I caused the preparation of the
complaint and that all the allegations thereat are true and
correct; that the promissory note sued upon, assuming that
it exists and bears the genuine signature of herein
defendant, the same does not bind him and that it did not
truly express the real intention of the parties as stated in
the defenses

Is this an effective specific denial as contemplated by law?


No. A reading of Defendants Answer, shows that he did not
specifically deny that he signed the loan documents. What he merely
stated in his Answer was that the signature appearing at the back of the
promissory note seemed to be his. Defendant also denied any liability on
the promissory note as he allegedly did not receive the amount stated
therein, and the loan documents do not express the true intention of the
parties. Defendant reiterated these allegations in his denial under oath,
stating that the promissory note sued upon, assuming that it exists and
bears the genuine signature of herein defendant, the same does not bind
him and that it did not truly express the real intention of the parties as
stated in the defenses.
His answer amounts to an implied admission of the due execution
and genuineness of the promissory note. The admission of the genuineness
and due execution of a document means that the party whose signature it
bears admits that he voluntarily signed the document or it was signed by
another for him and with his authority; that at the time it was signed it was
in words and figures exactly as set out in the pleading of the party relying
upon it; that the document was delivered; and that any formalities required
by law, such as a seal, an acknowledgment, or revenue stamp, which it

lacks, are waived by him. Also, it effectively eliminated any defense


relating to the authenticity and due execution of the document, e.g., that
the document was spurious, counterfeit, or of different import on its face
as the one executed by the parties; or that the signatures appearing thereon
were forgeries; or that the signatures were unauthorized.
Therefore, Defendant is deemed to have admitted the loan
documents and acknowledged his obligation with Plaintiff; and with
Defendants implied admission, it was not necessary for Plaintiff to
present further evidence to establish the due execution and authenticity of
the loan documents sued upon (Permanent Savings and Loan Bank vs.
Mariano Velarde,).

D. Adoptive admission concept and meaning


A party may, by his words or conduct, voluntarily adopt or ratify
another's statement. Where it appears that a party clearly and
unambiguously assented to or adopted the statements of another, evidence
of those statements is admissible against him. This is the essence of the
principle of adoptive admission (Republic vs. Kenrick Development
Corporation,).
An adoptive admission is a party's reaction to a statement or action
by another person when it is reasonable to treat the party's reaction as an
admission of something stated or implied by the other person. By adoptive
admission, a third person's statement becomes the admission of the party
embracing or espousing it (Republic vs. Kenrick Development
Corporation, G.R. No. 149576, August 8, 2006).
E. Adoptive admission concept and meaning
Adoptive admission may occur when a party:
(a) expressly agrees to or concurs in an oral statement made
by another;
(b) hears a statement and later on essentially repeats it;

(c) utters an acceptance or builds upon the assertion of


another;
(d) replies by way of rebuttal to some specific points raised
by another but ignores further points which he or she
has heard the other make or
(e) reads and signs a written statement made by another
(Republic vs. Kenrick Development Corporation,).

F. Answer must be signed


An unsigned answer is invalid and produces no legal effect.
Defendant may be declared in default and allow plaintiff to present
evidence ex-parte (Republic vs. Kenrick Development Corporation,).
---0---

COUNTERCLAIM

A. What is a counterclaim?
A counterclaim is any claim which a defending party may have
against an opposing party (Section 6, Rule 6, 1997 Revised Rules of Civil
Procedure). This may be a claim for money or other relief (See Sapugay
vs. CA,).

B. Kinds, Principles and its Limitations


Counterclaim may be compulsory or permissive.
Counterclaims are generally allowed in order to avoid a
multiplicity of suits and to facilitate the disposition of the whole
controversy in a single action, such that the defendants demand may be

adjudged by a counterclaim rather than by an independent suit (Lafarge


Cement Philippines, Inc., et al, vs. Continental Cement Corporation, et
al,).
The only limitations to this principle are (1) that the court should
have jurisdiction over the subject matter of the counterclaim, and (2) that
it could acquire jurisdiction over third parties whose presence is essential
for its adjudication suit (Lafarge Cement Philippines, Inc., et al, vs.
Continental Cement Corporation,).

COMPULSORY COUNTERCLAIM
A compulsory counterclaim is one which, being cognizable by the
regular courts of justice, arises out of or is connected with the transaction
or occurrence constituting the subject matter of the opposing party's claim
and does not require for its adjudication the presence of third parties of
whom the court cannot acquire jurisdiction. Such a counterclaim must be
within the jurisdiction of the court both as to the amount and the nature
thereof, except that in an original action before the Regional Trial Court,
the counter-claim may be considered compulsory regardless of the amount
(Section 7, Rule 6, 1997 Revised rules of Civil Procedure).
It is compulsory in the sense that it is within the jurisdiction of the
court. It does not require for its adjudication the presence of third parties
over whom the court cannot acquire jurisdiction, and will be barred in the
future if not set up in the answer to the complaint in the same case. Any
other counterclaim is permissive (Cruz-Agana vs. Judge SantiagoLagman,).
A compulsory counterclaim set up in the answer is not an
initiatory or similar pleading. The initiatory pleading is the plaintiffs
complaint. A defendant
has no choice but to raise a compulsory
counterclaim the moment the plaintiff files the complaint, otherwise
defendant waives the compulsory counterclaim. In short, the compulsory
counterclaim is a reaction or response, mandatory upon pain of waiver, to
an initiatory pleading which is the complaint (Cruz-Agana vs. Judge
Santiago-Lagman,).

A. Criteria or Tests of Compulsoriness


The following criteria to determine whether a counterclaim is
compulsory or permissive:
(1) Are issues of fact and law raised by the claim and by the
counterclaim largely the same?
(2) Would res judicata bar a subsequent suit on defendants claim,
absent the compulsory counterclaim rule?
(3) Will substantially the same evidence support or refute
plaintiffs claim as well as defendants counterclaim?
(4) Is there any logical relation between the claim and the
counterclaim? A positive answer to all four questions would indicate that
the counterclaim is compulsory (NAMARCO vs. Federation of United
Namarco Distributors,).
In short, if (1) the issues of fact or law raised by the claim and the
counterclaim largely the same; (2) res judicata will bar a subsequent suit
on defendants claim absent the compulsory counterclaim rule; (3) the
same evidence will substantially support or refute plaintiffs claim as well
as the defendants counterclaim; and (4) there is any logical relation
between the claim and the counterclaim, the claim is compulsory.
A counterclaim to be considered as compulsory it must passed the
so-called compelling test of compulsoriness, and that is, the test of
logical relationship between the main claim and the counterclaim must
exist. There exists such a relationship when conducting separate trials of
the respective claims of the parties would entail substantial duplication of
time and effort by the parties and the court; when the multiple claims
involve the same factual and legal issues; or when the claims are offshoots
of the same basic controversy between the parties (Quintanilla vs. CA).
B. Situationer

If for example, X moves to intervene as party defendant in a suit


between Plaintiff and Defendant, and in his answer he asks plaintiff for
attorneys fees, moral and exemplary damages as counterclaim for, it is
humbly opines that his counterclaim is permissive, because, he voluntarily
participated in the case without waiting first to be impeladed as party
defendant. It would have been different had Plaintiff later on amends his
complaint by impleading X as party defendant (indispensable or
necessary) and X files his answer pursuant to summons served upon him,
this time, it is humbly opines that counter claim for attorneys fees, moral
and exemplary damages pleaded in his answer is compulsory. Therefore, if
the attorneys fees, moral and exemplary damges is permissive, then he is
obliged to pay docket fees because, payment of correct docket fees is
jurisdictional.

C. Compulsory Counterclaim not set up on time is barred


A compulsory counterclaims not set up shall be barred (Section 2,
Rule 9, 1997 Revised Rules of Civil Procedure). The same should be set
up in the same action, otherwise, they would be barred forever (Lafarge
Cement Philippines, Inc., et al, vs. Continental Cement Corporation,).
If it is not set up shall be barred if not raised on time and the party
in error is precluded from setting it up in a subsequent litigation on the
ground of res judicata, the theory being that what are barred by prior
judgment are not only the matters actually raised and litigated upon, but
also such matters as could have been raised but were not (Metals
Engineering Resources Corporation vs. CA,).
It cannot be made the subject of a separate action but should be
asserted in the same suit involving the same transaction or occurrence
giving rise to it (Metals Engineering Resources Corp. vs. Court of
Appeals). This is because it is an auxiliary to the proceeding in the original
suit and derives its jurisdictional support therefrom, inasmuch as it arises
out of or is necessarily connected with the transaction or occurrence that is
the subject matter of the complaint (Metals Engineering Resources Corp.
vs. Court of Appeals.), and if made the subject of a separate suit, it may be
abated upon a plea of auter action pendant or litis pendentia, and/or

dismissed on the ground of res judicata (Visayan Packing Corporation


vs. Reparations Commission,).

D. Compulsory Counterclaim is not an initiatory pleading


A compulsory counterclaim does not require a certificate of nonforum shopping because a compulsory counterclaim is not an initiatory
pleading. Lack of certificate of non-forum shopping is immaterial (CruzAgana vs. Judge Santiago-Lagman,).
E. Effect of Dismissal of Plaintiffs Complaint
The dismissal of the complaint due to the fault of plaintiff does not
necessarily carry with it the dismissal of the counterclaim, compulsory or
otherwise. In fact, the dismissal of the complaint is without prejudice to
the right of defendants to prosecute the counterclaim (Pinga vs. Heirs of
Santiago,).
In short, the defendants counterclaims must be disposed of based
on the merit of the counterclaim itself and not on the survival of the main
complaint; if the counterclaim is palpably without merit or suffers
jurisdictional flaws which stand independent of the complaint, the trial
court is not precluded from dismissing it under the amended rules,
provided that the judgment or order dismissing the counterclaim is
premised on those defects; and if the counterclaim is justified,
counterclaim is protected by the rules from peremptory dismissal by
reason of the dismissal of the complaint (Pinga vs. Heirs of Santiago,).
It is worthy to note that what was given emphasis in Pinga is
dismissal of actions under Sections 2 and 3 of Rule 17 of the 1997 Revised
Rules of Civil Procedure. Dismissal of plaintiffs complaint by reason of
defendants motion to dismiss is not touched in Pinga. So, by analogy
application of the ruling in Pinga, if the complaint is dismissed by reason
of defendants motion (to dismiss), the defendants compulsory
counterclaim survives, as complaints dismissal does not result in the
automatic dismissal of the counterclaim.

Pinga case abandoned the ruling in Financial Building


Corporation vs. Forbes Park Association,) that dismissal of the main
action results in the dismissal of the counterclaim already filed. Therefore,
the rationale that it stands to reason that the filing of a motion to dismiss
the complaint is an implied waiver of the compulsory counterclaim
because the grant of the motion ultimately results in the dismissal of the
counterclaim in Financial Building Corporation case is no longer
controlling.
It also abandoned the ruling in Metals Engineering Resources
Corp. vs. CA). In that case the complaint was dismissed through
defendants motion on the ground of lack of jurisdiction for non-payment
of docket fees which dismissal was affirmed by the High Court holding
that that by reason of said dismissal, it is as if no claim was filed against
the defendant, hence the counterclaim has no leg to stand on. The doctrinal
pronouncement in Metals that if the court does not have jurisdiction to
entertain the main action of the case and dismisses the same, the
compulsory counterclaim, being ancillary to the principal controversy,
must likewise be dismissed since no jurisdiction remained for any grant of
relief under the counterclaim is also no longer controlling.
Now, it is no longer tree that dismissal of the complaint necessarily
carries the dismissal of the compulsory counterclaim.

F. Remedy to file by defendant if he has a compulsory


counterclaim against the Plaintiff: Either Answer or Motion to
Dismiss
The filing of a motion to dismiss and the setting up of a
compulsory counterclaim are incompatible remedies (Financial Building
Corporation vs. Forbes Park Association).
In the event that a defending party has a ground for dismissal and a
compulsory counterclaim at the same time, he must choose only one
remedy. If he decides to file a motion to dismiss, he will lose his
compulsory counterclaim. But if he opts to set up his compulsory
counterclaim, he may still plead his ground for dismissal as an affirmative

defense in his answer (Financial Building Corporation vs. Forbes Park


Association, Inc.,).
In short, the defendant may either file a motion to dismiss or file an
answer and asserts therein his motion to dismiss as an affirmative defense.
This of course, goes without saying that in the event that if defendant opts
to file a motion to dismiss, he may still file an answer with compulsory
counterclaim if his motion to dismiss will be denied.

G. Effects of plaintiffs failure to appear during pre-trial


conference to defendants compulsory counterclaim
The defendants compulsory counterclaim is not dismissible by
reason of dismissal of the plaintiffs complaint for plaintiffs failure to
appear during the pre-trial. According to Spouses Corpuz vs. Citibank,
N.A,), plaintiffs failure to appear during the pre-trial conference pursuant
to Section 3, Rule 17, 1997 Revised Rules of Civil Procedure amounts to
failure to comply with the Rules or any Order of the Court.
---0---

PERMISSIVE COUNTERCLAIM
A counterclaim is permissive if it does not arise out of or is not
necessarily connected with the subject matter of the opposing partys
claim (Lopez v. Gloria,). It is essentially an independent claim that may
be filed separately in another case (Lafarge Cement Philippines, Inc., et
al, vs. Continental Cement Corporation,), or is capable of proceeding
independently of the main case (Bungcayao, Sr. vs. Fort Ilocandia
Property Holdings and Development Corporation,).

A. Set-off and Recoupment as grounds for counterclaims

A set-off and recoupment can be a subject of counterclaims (Korea


Exchange Bank vs. Judge Gonzales,) provided that the following must
concur: (1) the same be essentially a genuine action of the defendant
against the plaintiff; (2) the same should have as its object to neutralize,
wholly or partially, that which the plaintiff is trying to obtain; (3) the same
does not have for its object to destroy directly the action of the plaintiff;
and (4) the same ought not to pray for a positive remedy distinct from the
payment of money (Lopez vs. Gloria).

B. Set-off (Compensacion)
distinguished

and

recoupment

(reconvencion)

A set-off (compensacion) is a money demand by the defendant


against the plaintiff arising upon contract and constituting a debt
independent of and unconnected with the cause of actions set forth in the
complaint, and may be used to offset a plaintiffs claim but not to recover
affirmatively (Lopez vs. Gloria).
A recoupment (reconvencion) differs from a counterclaim
(contrarreclamacion) in that, under a counterclaim, the defendant may
have an affirmative judgment where he is able to prove a demand in
excess of the plaintiffs demand, whereas in the case of recoupment,
whatever the damages proved by the defendant, they can go only to reduce
or extinguish the claim against him. Recoupment must arise out of the
contract or transaction upon which the plaintiffs claim is founded.
Recoupment is of French origin and means the cutting back of the
plaintiffs claim by the defendant. It thus implies an admission of the
plaintiffs claim (Lopez vs. Gloria).

C. Payment of docket fees for permissive counterclaim is


jurisdictional
The counterclaimant is bound to pay the prescribed docket fees
(Sandejas v. Ignacio, Jr.,).
A permissive counterclaim also requires payment of the prescribed
docket fees, because it is not only the filing of the complaint or

appropriate initiatory pleading, but the payment of the prescribed docket


fees that vests the court with jurisdiction over the subject matter of the
action (Tan vs. Planters Products,).
The non-payment of which is a ground for the dismissal of the
permissive counterclaim on ground of lack of jurisdiction over it, and the
court by authority of Section 1, Rule 9 of the 1997 Revised Rules of Civil
Procedure may dismiss it motu proprio or sans motion to dismiss (Tan vs.
Planters Products, Inc,).
Any decision rendered without jurisdiction with respect to
permissive counterclaim is a total nullity and may be struck down at any
time, even on appeal before this Court (Sandejas v. Ignacio, Jr.,).
In Sapugay vs. CA , it was held that payment of docket fees is
specifically applicable to permissive counterclaims only. Compulsory
counterclaim is excluded from its purview.

D. Counterclaimant should be ordered to pay the prescribed


docket fees
If the trial court made a mistake by saying that the counterclaim is
compulsory and on appeal it found out that the counterclaim is permissive,
the counterclaim is not dismissible, the counterclaimant should be given a
reasonable period of time, but in no case beyond the reglementary period,
within which to pay the prescribed docket fees because the
counterclaimant should not be made to suffer due to the mistake of the
trial court (Calibre Traders, Inc. vs. Bayer Philippines,).

E. Permissive Counterclaim requires a certificate of non-forum


shopping
A permissive counterclaim requires a certificate of non-forum
shopping because it is an initiatory pleading. Lack of it is fatal (CruzAgana vs. Judge Santiago-Lagman,), and shall be cause for the dismissal
of the counterclaim. The dismissal is always without prejudice, unless

otherwise provided (See Section 5(2) Rule 7, 1997 Revised Rules of Civil
Procedure).

F. Summons is not required to file answer in counterclaim


Well settled is the rule that the purpose of summons is to enable
the court to acquire jurisdiction over the person of the defendant. But as
regards counterclaim, the Rules of Court does not require that summons
should first be served on the defendant before an answer to counterclaim
must be made. Although a counterclaim is treated as an entirely distinct
and independent action, the defendant in the counterclaim, being the
plaintiff in the original complaint, has already submitted to the jurisdiction
of the court (Francisco Motors Corporation vs. CA,).

G. Rule on default in counterclaim


If the defendant (plaintiff in the complaint) fails within the
reglementary period, that is, ten (10) days from service (Section 4, Rule
11, 1997 Revised Rules of Civil Procedure) within which to file an answer,
he, upon motion of the counterclaimant (or the plaintiff in the
counterclaim) may be declared in default (Section 3, Rule 9, 1997 Rules of
Civil Procedure) (Francisco Motors Corporation vs. CA,).

H. Moral, actual and exemplary damages can either be compulsory


or permissive
Counterclaim for moral, actual and exemplary damages and
attorneys fees on account of plaintiffs malicious and unfounded
complaint is compulsory (Tiu Po vs. Bautista,), but in Korea Exchange
Bank vs. Judge Gonzales), a counterclaim for moral and exemplary
damages is considered permissive. Therefore, not all claims for moral and
exemplary damages is always compulsory, because this may also be in the
nature of permissive counterclaim. It can be compulsory or permissive
depending upon the circumstances by which the same is claimed.

---0---

PARTIES REMEDIES AFTER


ANSWER IS FILED
The following are the remedies available to the parties after an
answer is filed:
1. File a Motion for Judgment Based on Pleadings;
2. File a Motion for Summary Judgment; or
3. (Defendant) to file Motion for Preliminary Hearing on his
affirmative defense asserted in the answer (under Section 6, Rule
16, 1997 Revised Rules of Civil Procedure).

I. FILE A MOTION FOR JUDGMENT BASED ON


PLEADINGS

A. The rule
Where an answer fails to tender an issue, or otherwise admits the
material allegations of the adverse party's pleading, the court may, on
motion of that party, direct judgment on such pleading. xxx xxx xxx
(Section 1, Rule 34, 1997 Revised Rules of Civil Procedure).
B. Concept
Judgment on the pleadings is improper when the answer to the complaint
tenders several issues. It is proper when the answer admits all the material
averments of the complaint. But where several issues are properly

tendered by the answer, a trial on the merits must be resorted to in order to


afford each party his day in court (Municipality of Tiwi, vs. Betito, ).
Where a motion for judgment on the pleadings is filed, the
essential question is whether there are issues generated by the pleadings.
In a proper case for judgment on the pleadings, there is no ostensible issue
at all because of the failure of the defending partys answer to raise an
issue. The answer would fail to tender an issue, of course, if it does not
deny the material allegations in the complaint or admits said material
allegations of the adverse partys pleadings by confessing the truthfulness
thereof and/or omitting to deal with them at all (Tan v. De la Vega,).
When what is left are not genuinely issues requiring trial but
questions concerning the proper interpretation of the provisions of some
written contract attached to the pleadings, judgment on the pleadings is
proper (Philippine National Bank vs. Utility Assurance & Surety Co.,
Inc.,).
A motion for judgment on the pleadings admits the truth of all the
material and relevant allegations of the opposing party and the judgment must
rest on those allegations taken together with such other allegations as are
admitted in the pleadings. It is proper when an answer fails to tender an issue, or
otherwise admits the material allegations of the adverse partys pleading.
However, when it appears that not all the material allegations of the complaint
were admitted in the answer for some of them were either denied or disputed,
and the defendant has set up certain special defenses which, if proven, would
have the effect of nullifying plaintiffs main cause of action, judgment on the
pleadings cannot be rendered (Municipality of Tiwi, vs. Betito,).
The trial court has the discretion to grant a motion for judgment on
the pleadings filed by a party if there is no controverted matter in the case
after the answer is filed. A judgment on the pleadings is a judgment on the
facts as pleaded, and is based exclusively upon the allegations appearing
in the pleadings of the parties and the accompanying annexes (Sunbanun
vs. Go,).
If it is the defendant who moves for a judgment on the pleadings
without offering proof as to the truth of her own allegations and without
giving plaintiff the opportunity to introduce evidence, defendant is deemed

to have admitted the material and relevant averments of the complaint, and
to rest her motion for judgment based on the pleadings of the parties
(Rodriguez vs. Llorente,).
If the plaintiff asks the court for judgment on the pleadings and
defendant has consented to it, and if judgment is not favorable to the
defendant, he can no longer question the validity of the judgment. In
Tropical Homes, Inc. vs. CA, , the High Court said:
As to the amount of damages awarded as a
consequence of this violation of plaintiffs rights, the lower
court based its award from the allegations and prayer
contained in the complaint. The defendant, however,
questions this award for the reason that, according to the
defendant, the plaintiff, in moving for judgment on the
pleadings, did not offer proof as to the truth of his own
allegations with respect to the damages claimed by him,
and gave no opportunity for the appellant to introduce
evidence to refute his claims. We find this objection
without merit. It appears that when the plaintiff moved to
have the case decided on the pleadings, the defendant
interposed no objection and has practically assented
thereto. The defendant, therefore, is deemed to have
admitted the allegations of fact of the complaint, so that
there was no necessity for plaintiff to submit evidence of
his claim.
If the court can resolve the issues presented by the complaint and
the answer can be resolved within the four corners of said pleadings
without need to conduct further hearings, and what remains to be done is
the proper interpretation of the contracts or documents attached to the
pleadings, then judgment on the pleadings is proper (Pacific Rehouse
Corporation vs. EIB Securities, Inc.,).
In an action for sum of money and defendants have admitted that
they obtained the loan; have admitted the due execution of the loan
documents and their receipt of the demand letter made by the plaintiff, and
only issue is whether the obligation is matured or not, the High Court held
that the case can be decided summarily, because when plaintiff made its

demand, the obligation matured. The matter proferred as a defense could


be resolved judiciously by plain resort to the stipulations in the promissory
note (Wood Technology vs. Equitable Bank,).

II. FILE A MOTION FOR SUMMARY JUDGMENT


A. The rules
A party seeking to recover upon a claim, counterclaim, or crossclaim or to obtain a declaratory relief may, at any time after the pleading
in answer thereto has been served, move with supporting affidavits,
depositions or admissions for a summary judgment in his favor upon all or
any part thereof (Section 1, Rule 35, 1997 Revised Rules of Civil
Procedure).
A party against whom a claim, counterclaim, or cross-claim is
asserted or a declaratory relief is sought may, at any time, move with
supporting affidavits, depositions or admissions for a summary judgment
in his favor as to all or any part thereof (Section 2, Rule 35, 1997 Revised
Rules of Civil Procedure).
The motion shall be served at least ten (10) days before the time
specified for the hearing. The adverse party may serve opposing affidavits,
depositions, or admissions at least three (3) days before the hearing. After
the hearing, the judgment sought shall be rendered forthwith if the
pleadings, supporting affidavits, depositions, and admissions on file, show
that, except as to the amount of damages, there is no genuine issue as to
any material fact and that the moving party is entitled to a judgment as a
matter of law (Section 3, Rule 35, 1997 Revised Rules of Civil
Procedure).
If on motion under this Rule, judgment is not rendered upon the
whole case or for all the reliefs sought and a trial is necessary, the court at
the hearing of the motion, by examining the pleadings and the evidence
before it and by interrogating counsel shall ascertain what material facts
exist without substantial controversy and what are actually and in good
faith controverted. It shall thereupon make an order specifying the facts

that appear without substantial controversy, including the extent to which


the amount of damages or other relief is not in controversy, and directing
such further proceedings in the action as are just. The facts so specified
shall be deemed established, and the trial shall be conducted on the
controverted facts accordingly (Section 4, Rule 35, 1997 Revised Rules of
Civil Procedure).
Supporting and opposing affidavits shall be made on personal
knowledge, shall set forth such facts as would be admissible in evidence,
and shall show affirmatively that the affiant is competent to testify to the
matters stated therein. Certified true copies of all papers or parts thereof
referred to in the affidavit shall be attached thereto or served therewith
(Section 5, Rule 35, 1997 Revised Rules of Civil Procedure).
Should it appear to its satisfaction at any time that any of the
affidavits presented pursuant to this Rule are presented in bad faith, or
solely for the purpose of delay, the court shall forthwith order the
offending party or counsel to pay to the other party the amount of the
reasonable expenses which the filing of the affidavits caused him to incur
including attorney's fees, it may, after hearing further adjudge the
offending party or counsel guilty of contempt (Section 6, Rule 35, 1997
Revised Rules of Civil Procedure).

B. Concept and purpose


A summary judgment is granted to settle expeditiously a case if, on
motion of either party, there appears from the pleadings, depositions,
admissions, and affidavits that no important issues of fact are involved,
except the amount of damages. In such event, the moving party is entitled
to a judgment as a matter of law. Trial courts have limited authority to
render summary judgments and may do so only when there is clearly no
genuine issue as to any material fact ().
In Calubaquib vs. Republic, , the High Court explained the concept
of summary judgment, thus:
Summary judgments are proper when, upon motion of
the plaintiff or the defendant, the court finds that the answer filed

by the defendant does not tender a genuine issue as to any


material fact and that one party is entitled to a judgment as a
matter of law. A deeper understanding of summary judgments is
found in Viajar v. Estenzo:
Relief by summary judgment is intended to
expedite or promptly dispose of cases where the
facts appear undisputed and certain from the
pleadings, depositions, admissions and affidavits.
But if there be a doubt as to such facts and there be
an issue or issues of fact joined by the parties,
neither one of them can pray for a judgment. Where
the facts pleaded by the parties are disputed or
contested, proceedings for a judgment cannot take
the place of a trial.
An examination of the Rules will readily show
that a judgment is by no means a hasty one. It
assumes a scrutiny of facts in a summary hearing
after the filing of a motion for judgment by one party
supported by affidavits, depositions, admissions, or
other documents, with notice upon the adverse party
who may file an opposition to the motion supported
also by affidavits, depositions, or other documents
x x x. In spite of its expediting character, relief by
judgment can only be allowed after compliance with
the minimum requirement of vigilance by the court
in a summary hearing considering that this remedy is
in derogation of a party's right to a plenary trial of his
case. At any rate, a party who moves for judgment
has the burden of demonstrating clearly the absence
of any genuine issue of fact, or that the issue posed
in the complaint is so patently unsubstantial as not to
constitute a genuine issue for trial, and any doubt as
to the existence of such an issue is resolved against
the movant.
A summary judgment is permitted only if there is no
genuine issue as to any material fact and [the] moving party is

entitled to a judgment as a matter of law. The test of the


propriety of rendering summary judgments is the existence of a
genuine issue of fact, as distinguished from a sham, fictitious,
contrived or false claim. [A] factual issue raised by a party is
considered as sham when by its nature it is evident that it cannot
be proven or it is such that the party tendering the same has
neither any sincere intention nor adequate evidence to prove it.
This usually happens in denials made by defendants merely for
the sake of having an issue and thereby gaining delay, taking
advantage of the fact that their answers are not under oath
anyway.
In determining the genuineness of the issues, and hence the
propriety of rendering a summary judgment, the court is obliged
to carefully study and appraise, not the tenor or contents of the
pleadings, but the facts alleged under oath by the parties and/or
their witnesses in the affidavits that they submitted with the
motion and the corresponding opposition. Thus, it is held that,
even if the pleadings on their face appear to raise issues, a
summary judgment is proper so long as the affidavits,
depositions, and admissions presented by the moving party
show that such issues are not genuine.

In Nocom vs. Camerino, , a summary judgment is explained as a


procedural device resorted to in order to avoid long drawn out litigations
and useless delays. When the pleadings on file show that there are no
genuine issues of fact to be tried, the Rules allow a party to obtain
immediate relief by way of summary judgment, that is, when the facts are
not in dispute, the court is allowed to decide the case summarily by
applying the law to the material facts. Conversely, where the pleadings
tender a genuine issue, summary judgment is not proper. A genuine
issue is such issue of fact which requires the presentation of evidence as
distinguished from a sham, fictitious, contrived or false claim. Section 3
of the said rule provides two (2) requisites for summary judgment to be
proper: (1) there must be no genuine issue as to any material fact, except

for the amount of damages; and (2) the party presenting the motion for
summary judgment must be entitled to a judgment as a matter of law. A
summary judgment is permitted only if there is no genuine issue as to any
material fact and a moving party is entitled to a judgment as a matter of
law. A summary judgment is proper if, while the pleadings on their face
appear to raise issues, the affidavits, depositions, and admissions presented
by the moving party show that such issues are not genuine.
The law itself determines when a summary judgment is proper.
Under the rules, summary judgment is appropriate when there are no
genuine issues of fact which call for the presentation of evidence in a fullblown trial. Even if on their face the pleadings appear to raise issues,
when the affidavits, depositions and admissions show that such issues are
not genuine, then summary judgment as prescribed by the rules must
ensue as a matter of law. What is crucial for determination, therefore, is
the presence or absence of a genuine issue as to any material fact (Vadel
Realty and Development Corporation vs. Spouses Soriano,).
The the amount of damages is excluded in the coverage,
because in order to prove the issue as to amount of damages, presentation
of evidence is necessary.
In Bungcayao, Sr. vs. Fort Ilocandia Property Holdings, the trial
court in an action for declaration of nullity of contract noted the stipulated
issues and admissions been made by both parties, made to confirm the
agreement of the parties to cancel the Deed of Assignment, Release,
Waiver and Quitclaim and the return of the money to the respondent, and
summarily decided the case dismissing the plaintiffs claim for damages
for want of legal basis, and directed to immediately vacate the premises
subject of the case.

C. Whether a case is appropriate for summary judgment,


determining guide
In other words, in a motion for summary judgment, the crucial
question is: are the issues raised in the pleadings genuine, sham or

fictitious, as shown by affidavits, depositions or admissions accompanying


the motion? (Manufacturers Hanover Trust Co. vs. Guerrero, ).
So, if there is genuine issue to be resolved and necessitates the
presentation of evidence to resolve the issue, then summary judgment is
not proper.

A. Genuine issue, concept/definition


A genuine issue is an issue of fact which requires the
presentation of evidence as distinguished from a sham, fictitious,
contrived or false claim. When the facts as pleaded appear uncontested or
undisputed, then there is no real or genuine issue or question as to the
facts, and summary judgment is called for. The party who moves for
summary judgment has the burden of demonstrating clearly the absence of
any genuine issue of fact, or that the issue posed in the complaint is
patently unsubstantial so as not to constitute a genuine issue for trial. Trial
courts have limited authority to render summary judgments and may do so
only when there is clearly no genuine issue as to any material fact. When
the facts as pleaded by the parties are disputed or contested, proceedings
for summary judgment cannot take the place of trial (Evadel Realty and
Development Corporation v. Soriano,).
In Cotabato Timberland Co., Inc. vs. C. Alcantara and Sons,
Inc., , it was held that summary judgment is not proper because there is a
necessity of trial on the merits and presentation of evidence for the trial
court to properly determine which among the parties must shoulder the
loss. In that case, the special and affirmative defenses raised by private
respondents invoking, inter alia, the alleged fault and negligence of
petitioner as the proximate cause of the loss of the subject logs indubitably
tender a genuine and factual issue as regards the proximate cause of the
loss. A full-blown trial on the merits and presentation of additional
evidence is called for.
When variance in the allegations of the parties in the pleadings is evident,
and the facts as pleaded by the parties are disputed or contested, the case on
appeal should be remanded to the trial court for further proceedings and proper
disposition according to the rudiments of a regular trial on the merits and not

through an abbreviated termination of the case by summary judgment, because


proceedings for summary judgment cannot take the place of trial (Atty. Ferrer
vs. Spouses Diaz,).

B. Sham issue or claim, concept


A factual issue raised by a party is considered as sham when by its nature
it is evident that it cannot be proven or it is such that the party tendering the same
has neither any sincere intention nor adequate evidence to prove it. This
usually happens in denials made by defendants merely for the sake of having an
issue and thereby gaining delay, taking advantage of the fact that their answers
are not under oath anyway (Calubaquib vs. Republic, ).

F. Filing of motion and conduct of hearing necessary, non-compliance


warrants setting aside of the summary judgment
The filing of a motion and the conduct of a hearing on the motion are
therefore important because these enable the court to determine if the parties
pleadings, affidavits and exhibits in support of, or against, the motion are
sufficient to overcome the opposing papers and adequately justify the finding
that, as a matter of law, the claim is clearly meritorious or there is no defense to
the action. The non-observance of the procedural requirements of filing a motion
and conducting a hearing on the said motion warrants the setting aside of the
summary judgment (Eland Philippines, Inc. vs. Garcia, ).

In Calubaquib vs. Republic, , it was held that when the trial court
proceeded to render summary judgment with neither of the parties filing a motion
therefor it disragarded and ignored the guidelines and safeguards for the rendition
of a summary judgment thereby violating the defendants due process right to a
trial where they can present their evidence and prove their defense. In this case, it
was the trial court that directed the plaintiff to file a motion for summary
judgment which was opposed by the defendant.

Also in Calubaquib vs. Republic, , it was held that it was improper for
the trial court to have persisted in rendering summary judgment; to make a
baseless assumption is premature and unfair.

III. DEFENDANT TO FILE MOTION FOR PRELIMINARY


HEARING ON HIS AFFIRMATIVE DEFENSE ASSERTED
IN THE ANSWER
A. The rule
Preliminary hearing is not mandatory, but subject to the discretion
of the trial court.1
If no motion to dismiss has been filed, any of the grounds for
dismissal provided for in this Rule may be pleaded as an affirmative
defense in the answer and, in the discretion of the court, a preliminary
hearing may be had thereon as if a motion to dismiss had been filed
(Section 6, Rule 16, 1997 Revised Rules of Civil Procedure).
The allowance for a preliminary hearing, while left in the
discretion of the court, is authorized only if no motion to dismiss has been
filed but any of the grounds for a motion to dismiss had been pleaded as
an affirmative defense in the answer (Rasdas vs. Estenor,).
Section 6, Rule 16 of the 1997 Revised Rules of Civil Procedure is
explicit in stating that the defendant may reiterate any of the grounds for
dismissal provided under Rule 16 of the Rules of Court as affirmative
defenses but that a preliminary hearing may no longer be had thereon if a
motion to dismiss had already been filed (Abrajano vs. Heirs of Salas,
Jr.,).

Municipality of Bian, Laguna vs. CA, 219 SCRA 69, February 17, 1993 and Regalado,
Remedial Law Compendium, Vol. 1, pp. 163-164 (1993) cited in California and
Hawaiian Sugar Company vs. Pioneer Insurance and Surety Corporation, G.R. No.
139273, November 28, 2000.

B. Preliminary hearing may no longer be had thereon if a motion


to dismiss had already been filed, exception
The rule does not contemplate a situation where there are several
defendants but only one filed a motion to dismiss. Other defendants who
did not file a motion to dismiss may still ask and may be allowed for a
preliminary hearing on their own affirmative defenses, otherwise it will be
prejudicial to other defendants who did not file a motion to dismiss
(Abrajano vs. Heirs of Salas,).
Not only that, it will violate the due process and equal protection
clauses enshrined in our Constitution.

B. Preliminary hearing allowed even an answer is filed after a


motion to dismiss was filed and deferred its resolution
When a motion to dismiss is filed and its resolution is deferred,
and thereafter directed the defendant to file answer, and if answer is filed
defendant is still entitled to a preliminary hearing on his affirmative
defenses asserted in the Answer (California and Hawaiian Sugar
Company vs. Pioneer Insurance and Surety Corporation,).
In California and Hawaiian Sugar Company vs. Pioneer
Insurance and Surety Corporation, G it was held that defendant is still
entitled to preliminary hearing despite of filing a motion to dismiss,
because the trial court deferred the resolution of the motion to dismiss and
required the defendant to file answer instead.
The High Court said in California:
True, Section 6, Rule 16 of the 1997 Rules,
specifically provides that a preliminary hearing on the
affirmative defenses may be allowed only when no motion
to dismiss has been filed. Section 6, however, must be
viewed in the light of Section 3 of the same Rule, which
requires courts to resolve a motion to dismiss and prohibits
them from deferring its resolution on the ground of
indubitability.
Clearly then, Section 6 disallows a

preliminary hearing of affirmative defenses once a motion


to dismiss has been filed because such defense should have
already been resolved. In the present case, however, the
trial court did not categorically resolve petitioners Motion
to Dismiss, but merely deferred resolution thereof.
The case of Rasdas vs. Estenor is quite peculiar, because the High
Court affirmed the order of dismissal of the case on ground of res judicata
pleaded in the answer after a preliminary hearing was conducted. The
rationale advanced by the High Court is that disallowing the dismissal
would violate the primordial objective of procedural law to secure a just,
speedy and inexpensive disposition of every action and proceeding. The
High Court said in Rasdas:
We observe at the onset that it does appear that the
RTCs act of staging preliminary hearing on the affirmative
defense of lack of jurisdiction and res judicata is not in
regular order. Under Section 6, Rule 16 of the 1997 Rules
of Civil Procedure, the allowance for a preliminary hearing,
while left in the discretion of the court, is authorized only if
no motion to dismiss has been filed but any of the grounds
for a motion to dismiss had been pleaded as an affirmative
defense in the answer. In this case, respondents had filed a
motion to dismiss on the ground of res judicata, but the
same was denied. They thus filed an answer alleging res
judicata as a special affirmative defense, but later presented
a Motion for Preliminary Hearing which was granted,
leading to the dismissal of the case.
The general rule must be reiterated that the
preliminary hearing contemplated under Section 6, Rule 16
applies only if no motion to dismiss has been filed. This is
expressly provided under the rule, which relevantly states
[i]f no motion to dismiss has been filed, any of the
grounds for dismissal provided for in [Rule 16] may be
pleaded as an affirmative defense in the answer and, in the
discretion of the court, a preliminary hearing may be had
thereon as if a motion to dismiss had been filed. An
exception was carved out in California and Hawaiian Sugar

Company v. Pioneer Insurance, wherein the Court noted


that while Section 6 disallowed a preliminary hearing of
affirmative defenses once a motion to dismiss has been
filed, such hearing could nonetheless be had if the trial
court had not categorically resolved the motion to dismiss.
Such circumstance does not obtain in this case, since the
trial court had already categorically denied the motion to
dismiss prior to the filing of the answer and the motion for
preliminary hearing.
We observe in this case that the judge who had earlier
denied the motion to dismiss, Hon. Teodulo E. Mirasol,
was different from the judge who later authorized the
preliminary hearing, Hon. Isaac R. de Alban, a
circumstance that bears some light on why the RTC
eventually changed its mind on the motion to dismiss. Still,
this fact does not sanction the staging of a preliminary
hearing on affirmative defenses after the denial of the
motion to dismiss. If a judge disagrees with his/her
predecessors previous ruling denying a motion to dismiss,
the proper recourse is not to conduct a preliminary hearing
on affirmative defenses, but to utilize the contested ground
as part of the basis of the decision on the merits.
On the part of the movant whose motion to dismiss
had already been filed and denied, the proper remedy is to
file a motion for reconsideration of the denial of the
motion. If such motion for reconsideration is denied, the
ground for the dismissal of the complaint may still be
litigated at the trial on the merits.
Clearly, the denial of a motion to dismiss does not
preclude any future reliance on the grounds relied
thereupon. However, nothing in the rules expressly
authorizes a preliminary hearing of affirmative defenses
once a motion to dismiss has been filed and denied. Thus,
the strict application of Section 6, Rule 16 in this case
should cause us to rule that the RTC erred in conducting
the preliminary hearing.

However, there is an exceptional justification for us to


overlook this procedural error and nonetheless affirm the
dismissal of the complaint. The complaint in question is so
evidently barred by res judicata, it would violate the
primordial objective of procedural law to secure a just,
speedy and inexpensive disposition of every action and
proceeding should the Court allow this prohibited
complaint from festering in our judicial system. Indeed, the
rule sanctioning the liberal construction of procedural rules
is tailor-made for a situation such as this, when a by-thenumbers application of the rule would lead to absurdity,
such as the continued litigation of an obviously barred
complaint.

C. Though discretionary, denial of preliminary hearing sometimes


constitutes grave abuse of discretion
Also in California and Hawaiian Sugar Company vs. Pioneer
Insurance and Surety Corporation, , it was held that the trial court
gravely abused its discretion when it denied the motion for preliminary
hearing, thus:
The more crucial question that we must settle here is
whether the trial court committed grave abuse of discretion
when it denied petitioners Motion for a Preliminary
Hearing on their affirmative defense of lack of cause of
action. Undeniably, a preliminary hearing is not mandatory,
but subject to the discretion of the trial court. In the light of
the circumstances in this case, though, we find that the
lower court committed grave abuse of discretion in refusing
to grant the Motion.
We note that the trial court deferred the resolution of
petitioners Motion to Dismiss because of a single issue. It
was apparently unsure whether the charter party that the
bill of lading referred to was indeed the Baltimore Berth
Grain Charter Party submitted by petitioners.

Considering that there was only one question, which


may even be deemed to be the very touchstone of the whole
case, the trial court had no cogent reason to deny the
Motion for Preliminary Hearing. Indeed, it committed
grave abuse of discretion when it denied a preliminary
hearing on a simple issue of fact that could have possibly
settled the entire case. Verily, where a preliminary hearing
appears to suffice, there is no reason to go on to trial. One
reason why dockets of trial courts are clogged is the
unreasonable refusal to use a process or procedure, like a
motion to dismiss, which is designed to abbreviate the
resolution of a case.
---0---

RULE ON DEFAULT
There are times that a defendant after service of summons fails to
file answer to the complaint. This maybe because he does not know what
to do, or maybe because he is not really interested in the suit, he will
deliberately disregards the complaint by his inaction because he opt not to
be bothered by it.
If this happens, the plaintiff is on the advantageous position
because if the case is governed by the Rules on Summary Procedure
judgment can be rendered as may be warranted by the facts alleged in the
complaint under Section 6 of the 1991 Revised Rules on Summary
Procedure; and if the case is governed by the regular the defendant can be
declared in default under Section 3, Rule 9 of the 1997 Revised Rules of
Civil Procedure. In either, his case can be expeditiously disposed of.

A. Cases governed by the Revised Rules on Summary


Procedure;remedy when defendant did not file his answer

Section 6 of the 1991 Revised Rules on Summary Procedure


provides:
Sec. 6. Effect of failure to answer. Should the
defendant fail to answer the complaint within the period
above provided, the court, motu proprio, or on motion of the
plaintiff, shall render judgment as may be warranted by the
facts alleged in the complaint and limited to what is prayed
for therein: xxx xxx xxx.

B. Basic principles and guidelines; interpretation of the rules, strict


rather than liberal; exception
The use of the word shall in the Rule on Summary Procedure
underscores mandatory character, not directory. Giving the provisions a
directory application would subvert the nature of the Rule on Summary
Procedure and defeat its objective of expediting the adjudication of suits
(Gachon vs. Devera,).
Liberal interpretation and application of the rules applies only in
proper cases and under justifiable causes and circumstances (Gachon vs.
Devera, Jr.,).
Filing an answer beyond the prescribed period is a ground for the
court to render judgment under Section 6 of the 1991 Revised Rules on
Summary Procedure, except when the late filing for justifiable causes and
circumstances. This is where the liberal interpretation and application of
Rules applies (Gachon vs. Devera, Jr., ().
The case of Leynes vs. CA is somehow peculiar. Defendants last
day to file answer fell on Saturday. When his answer was filed the
following working day, a Monday, the trial court granted plaintiffs
motion to render judgment pursuant to Section 6 of the 1991 Revised
Rules on Summary Procedure holding that on that Saturday there were
employees present in court as required by A.M. Circular No. 2-99
primarily to act on petitions for bail and other urgent matters. The High
Court ordered that defendants answer be admitted.

C. Summary Judgment or Judgment on the Pleading not proper


Motions for Summary Judgment or Judgment on the Pleading are
not proper. While it is true that motion for motions for summary judgment
or judgment on the pleading are not prohibited (Section 16, 1991 Revised
Rules on Summary Procedure). The reason is obvious. For summary
judgment to apply there must exists a genuine issue (Narra Integrated
Corporation vs. CA,). Section 6 of the 1991 Revised Rules on Summary
Section applies because defendants inaction to the complaint. Same is
true with motion for judgment on the pleading. It presupposes that an
answer is filed, because a judgment on the pleadings is proper when an
answer fails to tender an issue, or otherwise admits the material allegations
of the adverse partys pleading (Municipality of Tiwi vs. Betito,).

D. Cases governed by regular procedure, remedy when defendant


did not file his answer
A declaration or order of default is issued as a punishment for
unnecessary delay in joining issues (Vlason Enterprises Corporation vs.
CA,).
Section 3, Rule 9 of the 1997 Revised Rules of Civil Procedure
provides:
Section 3. Default; declaration of. If the defending
party fails to answer within the time allowed therefor, the
court shall, upon motion of the claiming party with notice
to the defending party, and proof of such failure, declare
the defending party in default. Thereupon, the court shall
proceed to render judgment granting the claimant such
relief as his pleading may warrant, unless the court in its
discretion requires the claimant to submit evidence. Such
reception of evidence may be delegated to the clerk of
court.

From the foregoing procedure, if defendant did not file an answer


the trial court cannot motu proprio declare him in default. There must be a
motion to that effect.
The case of Spouses delos Santos vs. Judge Carpio) enumerated
the three requirements to which claiming party must be complied with
before the court may declare the defending party in default, to wit: (1) the
claiming party must file a motion asking the court to declare the defending
party in default; (2) the defending party must be notified of the motion to
declare him in default; (3) the claiming party must prove that the
defending party has failed to answer within the period provided by the
Rule.

E. The claiming party must file a motion asking the court to


declare the defending party in default; the defending party
must be notified of the motion to declare him in default
The claiming party must file a motion asking the court to declare
the defending party in default. In short, there must be a motion. The
plaintiff must file a motion to that effect. And considering that the trial
court cannot resolve the motion without asking the defendant of his
comment and/or opposition, the same is considered as motion of litigious
character. And therefore, the same must comply with the rules on motions,
particularly the requisite notice, service and hearing (See Sections 4, 5 and
6, Rule 15 of the 1997 Revised Rules of Civil Procedure; see also
discussions on Rules on Motions).
The case of Spouse delos Santos vs. Judge Carpio ) clarified that
present rule expressly requires that the motion of the claiming party
should be with notice to the defending party. The purpose of a notice of a
motion is to avoid surprises on the opposite party and to give him time to
study and meet the arguments. The notice of a motion is required when the
party has the right to resist the relief sought by the motion and principles
of natural justice demand that his right be not affected without an
opportunity to be heard (Spouse delos Santos vs. Judge Carpio,).
Therefore, as the present rule on default requires the filing of a motion and
notice of such motion to the defending party, it is not enough that the

defendant failed to answer the complaint within the reglementary period to


be a sufficient ground for declaration in default. The motion must also be
heard (Spouse delos Santos vs. Judge Carpio, ).

F. The claiming party must prove that the defending party has
failed to answer within the period provided by the Rule
To prove this element, the plaintiff must prove that the fifteen day
period, or the period fixed by the court for the defendant to file responsive
pleading has expired or lapsed. This can be done by giving emphasis to the
Sheriffs Return of Summons as proof that the fifteen day period, or the
period fixed by the court for the defendant to file responsive pleading has
expired or lapsed.

G. Effect to defendant if declared in default


A party in default is entitled to notice of subsequent proceedings
but shall not take part in the trial (Section 3 (a), Rule 9, 1997 Revised
Rules of Civil Procedure). He loses his standing in court and his right to
adduce evidence and to present his defense (Rural Bank of Sta. Catalina
vs. Land Bank of the Philippines,); and if pleading is filed, he should
expect the trial court to act upon their pleadings ((Vlason Enterprises
Corporation vs. CA,).

H. Plaintiff not automatically entitled to the relief prayed for when


defendant is declared in default;
I. A declaration of default not an admission of the truth or the
validity of the plaintiffs claims; and
J. The trial court has no authority to order plaintiff to present
evidence ex parte without declaring first the defendant in
default

Declaration of default is somehow like a foul shot in a basketball


game wherein the free-thrower is allowed to throw his shot free to garn a
point with no player allowed to defend the shot.
Like a free-throw in a basketball game, the plaintiff in default
proceedings is not automatically entitled to the relief prayed for in the
complaint.
Though plaintiff is allowed to present evidence ex-parte, he is
still duty bound to present sufficient evidence to establish his cause of
action because failure to do so will result in the denial of his claim or
dismissal of his action. The plaintiff is duty bound to establish by
competent evidence all the material allegations of his complaint upon
which he bases his prayer for relief (Lopez vs. Mendezona, ;Luxuria
Homes and/or Posadas vs. CA,), because a judgment by default against a
defendant does not imply a waiver of rights except that of being heard and
of presenting evidence in his favor (De los Santos v. De la Cruz, cited in
Luxuria Homes and/or Posadas vs. CA,). Declaration of default does not
imply admission by the defendant of the facts and causes of action of the
plaintiff, because the codal section requires the latter to adduce his
evidence in support of his allegations as an indispensable condition before
final judgment could be given in his favor. Nor could it be interpreted as
an admission by the defendant that the plaintiffs causes of action finds
support in the law or that the latter is entitled to the relief prayed for
(Luxuria Homes and/or Posadas vs. CA,).
A declaration of default is not an admission of the truth or the
validity of the plaintiffs claims (Macondray & Co. vs. Eustaquio,).
Although defendant may be declared in default, the plaintiff is still
required to substantiate his allegations in the complaint because the
judgment of default against defendant who has not appeared or filed his
answer does not imply a waiver of all his rights, except his right to be
heard and to present evidence in his favor; the defendants failure to
answer does not imply his admission of the facts and the causes of action
of the plaintiff, because plaintiff is still required to adduce evidence to
support his allegation, thus the defaulting defendant is still protected by
law (Vlason Enterprises Corporation vs. CA,).

Not only that, the only evidence that may be allowed by the trial
court are those evidence that will tend to support the relief sought and
specified in the complaint (Vlason Enterprises Corporation vs. CA), and
no award greater than or different in kind from that specified in the
complaint can be granted (Vlason Enterprises Corporation vs. CA,), and
if a relief other than that specifically prayed for in the complaint is
awarded in a judgment by default, such an award is null and void because
the court is bereft of jurisdiction to grant such an award (Policarpio vs.
RTC of Quezon City,).
Vlason Enterprises Corporation vs. CA (made to emphasize that
without a declaration that petitioner is in default as required in Section 1,
Rule 18 of the Rules of Court, the trial court has no authority to order the
presentation of evidence ex parte against petitioner to render judgment
against it by default.
Once the defendant is declared in default the plaintiff is not
automatically entitled to the relief prayed for. The court can grant the
favorable relief sought only after it has ascertained that the evidence
offered and the facts proven by the presenting party warrant the grant of
the same (Pascua vs. Florendo cited in Luxuria Homes and/or Posadas
vs. CA,), because it would be meaningless to require presentation of
evidence if everytime the other party is declared in default, a decision
would automatically be rendered in favor of the non-defaulting party and
exactly according to the tenor of his prayer (Luxuria Homes and/or
Posadas vs.).
A defaulted defendant is not actually thrown out of court. The
rules see to it that any judgment against him must be in accordance with
law. The evidence to support the plaintiffs cause is, of course, presented
in his absence, but the court is not supposed to admit that which is
basically incompetent. Although the defendant would not be in a position
to object, elementary justice requires that only legal evidence should be
considered against him. If the evidence presented should not be sufficient
to justify a judgment for the plaintiff, the complaint must be dismissed.
And if an unfavorable judgment should be justifiable, it cannot exceed the
amount or be different in kind from what is prayed for in the complaint
(Luxuria Homes and/or Posadas vs. CA,).

K. Plaintiff must prove his cause by preponderance of evidence


In civil cases, the party having the burden of proof must establish
his case by a preponderance of evidence. In determining where the
preponderance or superior weight of evidence on the issues involved lies,
the court may consider all the facts and circumstances of the case, the
witnesses manner of testifying, their intelligence, their means and
opportunity of knowing the facts to which they are testifying, the nature of
the facts to which they testify, the probability or improbability of their
testimony, their interest or want of interest, and also their personal
credibility so far as the same may legitimately appear upon the trial. The
court may also consider the number of witnesses, though the
preponderance is not necessarily with the greater number (Section 1, Rule
133, Rules of Court).
Plaintiff is not excused from not establishing his claims alleged in
the complaint by the required quantum of proof under Section 1 of Rule
133; any advantage they he have gained from the ex parte presentation of
evidence does not lower the degree of proof required (Gajudo, et al., vs.
Traders Royal Bank,).

L. Section 3 of Rule 9 and Section 1 of Rule 133 of the Rules of


Court are not incompatible with each other
In one case the quantum of evidence for judgments flowing from a
default order under Section 3 of Rule 9 was raised as it is not the same as
that provided for in Section 1 of Rule 133. This issue had already been
clarified by the High Court in Gajudo, et al., vs. Traders Royal Bank
holding that Section 3 of Rule 9 and Section 1 of Rule 133 of the Rules of
Court are not incompatible with each other.
The High Court said:
For ease of discussion, these two rules will be
reproduced below, starting with Section 3 of Rule 9 of the
Rules of Court:

Sec. 3. Default; declaration of. If the


defending party fails to answer within the time
allowed therefor, the court shall, upon motion of
the claiming party with notice to the defending
party, and proof of such failure, declare the
defending party in default. Thereupon, the court
shall proceed to render judgment granting the
claimant such relief as his pleading may
warrant, unless the court in its discretion
requires the claimant to submit evidence. Such
reception of evidence may be delegated to the
clerk of court.
(a) Effect of order of default. A party
in default shall be entitled to notice of
subsequent proceedings but not to take part in
the trial.
(b) Relief from order of default. A party
declared in default may at any time after notice
thereof and before judgment file a motion under
oath to set aside the order of default upon proper
showing that his failure to answer was due to
fraud, accident, mistake or excusable negligence
and that he has a meritorious defense. In such
case, the order of default may be set aside on
such terms and conditions as the judge may
impose in the interest of justice.
(c) Effect of partial default. When a
pleading asserting a claim states a common
cause of action against several defending
parties, some of whom answer and the others
fail to do so, the court shall try the case against
all upon the answers thus filed and render
judgment upon the evidence presented.
(d) Extent of relief to be awarded. A
judgment rendered against a party in default

shall not exceed the amount or be different in


kind from that prayed for nor award
unliquidated damages.
(e) Where no defaults allowed. If the
defending party in an action for annulment or
declaration of nullity of marriage or for legal
separation fails to answer, the court shall order
the prosecuting attorney to investigate whether
or nor a collusion between the parties exists, and
if there is no collusion, to intervene for the State
in order to see to it that the evidence submitted
is not fabricated.
We now quote Section 1 of Rule 133:
SECTION 1. Preponderance of evidence,
how determined. In civil cases, the party
having the burden of proof must establish his
case by a preponderance of evidence. In
determining where the preponderance or
superior weight of evidence on the issues
involved lies, the court may consider all the
facts and circumstances of the case, the
witnesses manner of testifying, their
intelligence, their means and opportunity of
knowing the facts to which they are testifying,
the nature of the facts to which they testify, the
probability or improbability of their testimony,
their interest or want of interest, and also their
personal credibility so far as the same may
legitimately appear upon the trial. The court
may also consider the number of witnesses,
though the preponderance is not necessarily
with the greater number.
Between the two rules, there is no incompatibility that
would preclude the application of either one of them. To
begin with, Section 3 of Rule 9 governs the procedure

which the trial court is directed to take when a defendant


fails to file an answer. According to this provision, the
court shall proceed to render judgment granting the
claimant such relief as his pleading may warrant, subject
to the courts discretion on whether to require the
presentation of evidence ex parte. The same provision also
sets down guidelines on the nature and extent of the relief
that may be granted. In particular, the courts judgment
shall not exceed the amount or be different in kind from
that prayed for nor award unliquidated damages.
As in other civil cases, basic is the rule that the party
making allegations has the burden of proving them by a
preponderance of evidence. Moreover, parties must rely on
the strength of their own evidence, not upon the weakness
of the defense offered by their opponent. This principle
holds true, especially when the latter has had no
opportunity to present evidence because of a default order.
Needless to say, the extent of the relief that may be granted
can only be as much as has been alleged and proved with
preponderant evidence required under Section 1 of Rule
133. (Citations Omitted).

M. Judgment by default cannot grant relief not prayed for;


exception, when filed an answer but were absent during trial
Vlason Enterprises Corporation vs. CA, , said that judgment by
default cannot grant relief not prayed for; exception, when filed an answer
but were absent during trial.
The High Court in Vlason:
A declaration or order of default is issued as a
punishment for unnecessary delay in joining issues. In
such event, defendants lose their standing in court, they
cannot expect the trial court to act upon their pleadings, and
they are not entitled to notice of the proceeding until the
final termination of the case. Thus, the trial court proceeds

with the reception of the plaintiffs evidence upon which a


default judgment is rendered.
Section 1 of Rule 18 provides that after the defendant
has been declared in default, the court shall proceed to
receive the plaintiffs evidence and render judgment
granting him such relief as the complaint and the facts
proven may warrant. The reliefs that may be granted,
however, are restricted by Section 5, which provides that a
judgment entered against a party in default shall not exceed
the amount or be different in kind from that prayed for.
In other words, under Section 1, a declaration of
default is not an admission of the truth or the validity of the
plaintiffs claims. The claimant must still prove his claim
and present evidence. In this sense the law gives defaulting
parties some measure of protection because plaintiffs,
despite the default of defendants, are still required to
substantiate their allegations in the complaint.
The
judgment of default against defendants who have not
appeared or filed their answers does not imply a waiver of
all their rights, except their right to be heard and to present
evidence in their favor. Their failure to answer does not
imply their admission of the facts and the causes of action
of the plaintiffs, because the latter are required to adduce
evidence to support their allegations.
Moreover, the trial court is not allowed by the Rules
to receive evidence that tends to show a relief not sought or
specified in the pleadings. The plaintiff cannot be granted
an award greater than or different in kind from that
specified in the complaint.
This case should be distinguished, however, from that
of defendants, who filed an answer but were absent during
trial. In that case, they can be held liable for an amount
greater than or different from that originally prayed for,
provided that the award is warranted by the proven facts.
This rule is premised on the theory that the adverse party

failed to object to evidence relating to an issue not raised in


the pleadings. (Underscoring Supplied)

N. A default judgment only affects defaulted defendant


A default judgment against several defendants cannot affect the
rights of the other defendants who are not declared in default. The trial
court has no authority to allow plaintiff to evidence ex parte against a
defendant is not declared in default. To do otherwise will result to a
manifest failure or miscarriage of justice (Vlason enterprises Corporation
vs. CA,).

O. A default judgment cannot includean award not prayed for in


the complaint
A judgment in default proceeding cannot include an award not
prayed for in the complaint, even if proven ex parte, because the trial court
is not allowed by the Rules to receive evidence that tends to show a relief
not sought or specified in the pleadings and the plaintiff cannot be granted
an award greater than or different in kind from that specified in the
complaint (Vlason enterprises Corporation vs. CA,).
P. Remedy of defendant if plaintiffs motion todeclare defendant
in default is granted
Settled is the rule that a party in default is entitled to notice of
subsequent proceedings but shall not take part in the trial; that he loses his
standing in court and his right to adduce evidence and to present his
defense, and if pleading is filed he should expect the trial court to act upon
their pleadings (Section 3 (a), Rule 9, 1997 Revised Rules of Civil
Procedure; Rural Bank of Sta. Catalina vs. Land Bank of the
Philippines,). Nevertheless, a party in default has still available remedies.
The following are the remedial measures available to defendants in
case he is ordered declared in default:

(1) a motion to set aside the order of default under Section


3(b), Rule 9 of the Rules of Court, if the default was
discovered before judgment could be rendered;
(2) a motion for new trial under Section 1(a) of Rule 37, if
the default was discovered after judgment but while
appeal is still available;
(3) a petition for relief under Rule 38, if judgment has
become final and executory; and
(4) an appeal from the judgment under Section 1, Rule 41,
even if no petition to set aside the order of default has
been resorted to (Lina vs. CA, 135 SCRA 637 cited in
Indiana Aerospace University vs. CHED, G.R. No.
139371, April 4, 2001).
Martinez vs. Republic () allows the filing of appeal under Section
1, Rule 41 of the 1997 Revised Rules of Civil Procedure on the ground
that the plaintiff failed to prove the material allegations of the complaint,
or that the decision is contrary to law, even without need of the prior filing
of a motion to set aside the order of default. In short, there is no need to
secure an order lifting an order of default.
In Rural Bank of Sta. Catalina vs. LBP, the High Court said:
It bears stressing that a defending party declared in
default loses his standing in court and his right to adduce
evidence and to present his defense. He, however, has the
right to appeal from the judgment by default and assail said
judgment on the ground, inter alia, that the amount of the
judgment is excessive or is different in kind from that
prayed for, or that the plaintiff failed to prove the material
allegations of his complaint, or that the decision is contrary
to law. Such party declare in default is proscribed from
seeking a modification or reversal of the assailed decision
on the basis of the evidence submitted by him in the Court
of Appeals, for if it were otherwise, he would thereby be

allowed to regain his right to adduce evidence, a right


which he lost in the trial court when he was declare in
default, and which he failed to have vacated. In this case,
the petitioner sought the modification of the decision of the
trial court based on the evidence submitted by it only in the
Court of Appeals.
(Authors observation: there is no need to secure an order of
lifting an order of default because filing a notice of appeal is considered
as a post judgment remedy).
The foregoing remedial measures is available to party in default if
the order declaring him in default is not a patent nullity or void, because if
the order declaring him in default is a clear and patent nullity, his available
remedy is certiorari under Rule 65.
Certiorari under Rule 65. Why certiorari under Rule 65?
The explanation is so simple.
An order granting a motion to declare defendant in default is
interlocutory nature (Indiana Aerospace University vs. CHED,). This is
because the order granting a motion to declare defendant in default does
not disposes of the case. Such an order is not a final order, because it
leaves something to be done by the court before the case is finally decided
on the merits (Philgreen Trading Construction Corporation vs. CA,).
Therefore the same cannot be a subject of an appeal under Section 1, Rule
41 of the 1997 Revised Rules of Civil Procedure.
Paragraph 2(c), Section 1, Rule 41, 1997 Revised Rules of Civil
Procedure provides that no appeal may be taken from an interlocutory
order. Therefore, the only available remedy is Certiorari under Rule 65
(Cerezo vs. David Tuazon,).
Certiorari is a remedy when any tribunal, board or officer
exercising judicial or quasi-judicial functions has acted without or in
excess its or his jurisdiction, or with grave abuse of discretion amounting
to lack or excess of jurisdiction, and there is no appeal, or any plain,

speedy, and adequate remedy in the ordinary course of law, a person


aggrieved thereby may file a verified petition in the proper court, alleging
the facts with certainty and praying that judgment be rendered annulling or
modifying the proceedings of such tribunal, board or officer, and granting
such incidental reliefs as law and justice may require (Section 1, Rule 65,
1997 Revised Rules of Civil Procedure).
In Guanzon vs. Arradaza, G., the High Court said that a petition
for certiorari under Rule 65 to declare the nullity of judgment by default is
also available if the trial court improperly decalred a party in default, or
even if the trial court properly decalred a party in default, if grave abuse of
discretion attended such declaration.

However, there is no harm if he will avail of the foregoing


remedial measures laid down in Lina case, but he may no longer later on
question the patent nullity of the order because he may deemed to have
waive his right to question the fact of nullity.
Same remedial measure also is available to plaintiff if his motion
to declare defendant in default is denied, or against order setting aside
order of default, because an order setting aside order of default is
interlocutory (Denso (Phils.), Inc., vs. IAC,).
In Indiana Aerospace University vs. CHED,), the subject of the
action is CHEds regulation or administration of educational institutions,
and therefore, treated as action imbued with public interest, hence, the
filing of the requisite motion for reconsideration was dispensed with
(Liberty Insurance Corporation vs. CA,).
The case of Spouse delos Santos vs. Judge Carpio) is also peculiar
in the sense that in this case, the trial judge motu proprio issued an order
declared defendant in default without waiting for the scheduled hearing on
the motion to declare defendant in default. Thats foul according to the
High Court, and said:
We could not see any justifiable reason why the
trial court chose not to hear the petitioners on the date and
time fixed in Metrobanks motion, and instead, hastily

granted the motion before it could be heard on the ground


that it had found the motion to be impressed with merit.
Indeed, in totally disregarding the purpose for which the
filing of a motion and notice to defending party are
required by the Rules, the trial court had acted in a despotic
manner that is correctly assailed through a petition for
certiorari which petitioners have seasonably filed with the
CA.
Again, respondent Judge acted capriciously when he
totally ignored petitioners Opposition to Metrobanks
Motion to Declare them in Default and denied their Motion
to Admit Answer, both filed on February 15, 2001, a day
before the scheduled hearing, which showed their desire to
be heard before the motion to declare them in default is
resolved by the trial court.
Q. Resorting of plaintiff to filing of motion for summary judgment
or judgment on the pleading not proper
Motions for summary judgment or judgment on the pleading are
not proper. Again, the reason is obvious. For these remedies to apply, it
presupposes that an answer is filed. For summary judgment to apply there
must exists a genuine issue (Narra Integrated Corporation vs. CA,).
Judgment on the pleadings is proper when an answer fails to tender an
issue, or otherwise admits the material allegations of the adverse partys
pleading (Municipality of Tiwi vs. Betito,).

R. Requirements must comply with in filing motion to lift order of


default
The defendant in default may, at any time after discovery thereof
and before judgment, file a motion under oath to set aside the order of
default on the ground that his failure to answer was due to fraud, accident,
mistake or excusable negligence, and that he has a meritorious defense
(Section 3(b), Rule 9, 1997 Revised Rules of Civil Procedure).

In short, the motion must be under oath, and must contain an


allegation that the defendants failure to file his answer was due to fraud,
accident, mistake, or excusable negligence.

S. Certiorari (under Rule 65) not always grantable against an


order denying motion to lift order of default
In David vs. Judge Gutierrez-Fruelda, , it was held that the trial judge
did not commit grave abuse of discretion when she denied the motion to lift
order of default on ground the motion was not under oath. The motion did not
also contain an allegation that the defendants failure to file his answer
was due to fraud, accident, mistake, or excusable negligence.
T. Remedy of plaintiff if his motion to declare defendant in default
is denied
If there is grave abuse of discretion in denying the motion to
declare defendant in default, petition for certiorari under Rule 65 is the
remedy. Reason: an order denying the motion to declare defendant in
default is an interlocutory order.

U. Rule on default in counterclaim


If the defendant (plaintiff in the complaint) fails within the
reglementary period, that is, ten (10) days from service (Section 4, Rule
11, 1997 Revised Rules of Civil Procedure) within which to file an answer,
he, upon motion of the counterclaimant (or the plaintiff in the
counterclaim) may be declared in default (Section 3, Rule 9, 1997 Rules
of Civil Procedure; Francisco Motors Corporation vs. CA,).
---0---

DEMURRER TO EVIDENCE
A. Demurrer to evidence, definition, concept and purpose

It is defined as an objection by one of the parties in an action, to


the effect that the evidence which his adversary produced is insufficient in
point of law, whether true or not, to make out a case or sustain the issue
(Gutib vs. CA, G).
It authorizes a judgment on the merits of the case without the
defendant having to submit evidence on his part as he would ordinarily
have to do, if plaintiffs evidence shows that he is not entitled to the relief
sought (Dandoy vs. CA,).
It is an aid or instrument for the expeditious termination of an
action, similar to a motion to dismiss, which the court or tribunal may
either grant or deny (Dandoy vs. CA,).
Its purpose is precisely to expeditiously terminate the case without
the need of the defendants evidence (Spouses Condes vs. CA,).

B. May be issued, when?


A demurrer to evidence may be issued when, upon the facts
adduced and the applicable law, the plaintiff has shown no right to relief.
Where the totality of plaintiffs evidence, together with such inferences
and conclusions as may reasonably be drawn therefrom, does not warrant
recovery against the defendant, a demurrer to evidence should be
sustained (Dandoy vs. CA,). It may be granted if, after the presentation of
plaintiffs evidence, it appears upon the facts and the law that the plaintiff
has shown no right to relief (Republic vs. Tuvera,).
A demurrer to evidence is likewise sustainable when, admitting
every proven fact favorable to the plaintiff and indulging in his favor all
conclusions fairly and reasonably inferable therefrom, the plaintiff has
failed to make out one or more of the material elements of his case, or
when there is no evidence to support an allegation necessary to his claim.
It should be sustained where the plaintiffs evidence is prima facie
insufficient for a recovery (Dandoy vs. CA,).

It is, therefore, premature to speak of preponderance of


evidence in a demurrer to evidence because it is filed before the
defendant presents his evidence (Spouses Condes vs. CA).

C. When to file?
After the plaintiff has completed the presentation of his evidence,
the defendant may move for dismissal on the ground that upon the facts
and the law the plaintiff has shown no right to relief. If his motion is
denied, he shall have the right to present evidence. If the motion is granted
but on appeal the order of dismissal is reversed he shall be deemed to have
waived the right to present evidence (Section 1, Rule 33, 1997 Revised
Rules of Civil Procedure).

D. Resolving a demurrer to evidence, basis


What should be resolved in a motion to dismiss based on a
demurrer to evidence is whether the plaintiff is entitled to the relief based
on the facts and the law. The evidence contemplated by the rule on
demurrer is that which pertains to the merits of the case, excluding
technical aspects such as capacity to sue (Casent Realty Development
Corp. vs. Philbanking Corporation,).
The plaintiffs evidence should not be the only basis in resolving a
demurrer to evidence. The facts referred to in Section 8, Rule 8 of the
Rules of Court should include all the means sanctioned by the Rules of
Court in ascertaining matters in judicial proceedings. These include
judicial admissions, matters of judicial notice, stipulations made during
the pre-trial and trial, admissions, and presumptions, the only exclusion
being the defendants evidence (Casent Realty Development Corp. vs.
Philbanking Corporation,). In short, Courts may consider other facts
within the range of judicial notice as well as relevant laws and
jurisprudence which the courts are bound to take into account, and they
are also fairly entitled to examine records/documents duly incorporated
into the complaint by the pleader himself in ruling on the demurrer to the
complaint (U. Baez Electric Light Company vs. Abra Electric
Cooperative, Inc., et al.,).

E. Section 8, Rule 8 of the Rules of Court


When an action or defense is founded upon a written instrument,
copied in or attached to the corresponding pleading as provided in the
preceding section, the genuineness and due execution of the instrument
shall be deemed admitted unless the adverse party, under oath, specifically
denies them, and sets forth, what he claims to be the facts; but the
requirement of an oath does not apply when the adverse party does not
appear to be a party to the instrument or when compliance with an order
for an inspection of the original instrument is refused2 (See Casent Realty
Development Corp. vs. Philbanking Corporation,).

F. Judicial admission also considered in resolving the demurrer to


evidence
An admission, verbal or written, made by a party in the course of
the proceeding in the same case, does not require proof. The admission
may be contradicted only by showing that it was made through palpable
mistake or that no such admission was made3 (See Casent Realty
Development Corp. vs. Philbanking Corporation,).
In Casent Realty Development Corp. vs. Philbanking
Corporation, , it was held that court must consider the deemed admitted
genuineness and due execution of the Dacion and Confirmation Statement
under oath in resolving the demurrer to evidence.
The High Court said in Casent:
On appeal to the CA, respondent claimed that even
though it failed to file a Reply, all the new matters alleged
in the Answer are deemed controverted anyway, pursuant
to Rule 6, Section 10:

Section 8. How to contest such documents.


Section 4. Judicial admissions.

Section 10. Reply.A reply is a pleading,


the office or function of which is to deny, or
allege facts in denial or avoidance of new
matters alleged by way of defense in the answer
and thereby join or make issue as to such new
matters. If a party does not file such reply, all
the new matters alleged in the answer are
deemed controverted.
We agree with petitioner.
Rule 8, Section 8
specifically applies to actions or defenses founded upon a
written instrument and provides the manner of denying it. It
is more controlling than Rule 6, Section 10 which merely
provides the effect of failure to file a Reply. Thus, where
the defense in the Answer is based on an actionable
document, a Reply specifically denying it under oath must
be made; otherwise, the genuineness and due execution of
the document will be deemed admitted. Since respondent
failed to deny the genuineness and due execution of the
Dacion and Confirmation Statement under oath, then these
are deemed admitted and must be considered by the court
in resolving the demurrer to evidence. We held in
Philippine American General Insurance Co., Inc. v. Sweet
Lines, Inc. that [w]hen the due execution and genuineness
of an instrument are deemed admitted because of the
adverse partys failure to make a specific verified denial
thereof, the instrument need not be presented formally in
evidence for it may be considered an admitted fact.

G. Res judicata ground for demurrer to evidence, inappropriate


Res judicata is an inappropriate ground for sustaining a demurrer
to evidence, even as it stands as a proper ground for a motion to dismiss.
A demurrer may be granted if, after the presentation of plaintiffs
evidence, it appears upon the facts and the law that the plaintiff has shown
no right to relief. In contrast, the grounds for res judicata present
themselves even before the presentation of evidence, and it should be at
that stage that the defense of res judicata should be invoked as a ground

for dismissal. Properly speaking, the movants for demurral who wish to
rely on a controlling value of a settled case as a ground for demurrer
should invoke the ground of stare decisis in lieu of res judicata (Republic
vs. Tuvera, G).

H. Effect of judgment on demurrer to evidence


Again, after the plaintiff has completed the presentation of his
evidence, the defendant may move for dismissal on the ground that upon
the facts and the law the plaintiff has shown no right to relief. If his
motion is denied, he shall have the right to present evidence. If the motion
is granted but on appeal the order of dismissal is reversed he shall have be
deemed to have waived the right to present evidence (Section 1, Rule 33,
1997 Revised Rules of Civil Procedure).
The general rule is that upon the dismissal of the demurrer in the
appellate court, the defendant loses the right to present his evidence and
the appellate court shall then proceed to render judgment on the merits
on the basis of plaintiffs evidence (Republic vs. Tuvera,).
The rationale behind the rule and doctrine is simple and logical.
The defendant is permitted, without waiving his right to offer evidence in
the event that his motion is not granted, to move for a dismissal (i.e.,
demur to the plaintiffs evidence) on the ground that upon the facts as thus
established and the applicable law, the plaintiff has shown no right to
relief. If the trial court denies the dismissal motion, i.e., finds that
plaintiffs evidence is sufficient for an award of judgment in the absence
of contrary evidence, the case still remains before the trial court which
should then proceed to hear and receive the defendants evidence so that
all the facts and evidence of the contending parties may be properly placed
before it for adjudication as well as before the appellate courts, in case of
appeal. Nothing is lost. The doctrine is but in line with the established
procedural precepts in the conduct of trials that the trial court liberally
receive all proffered evidence at the trial to enable it to render its decision
with all possibly relevant proofs in the record, thus assuring that the
appellate courts upon appeal have all the material before them necessary
to make a correct judgment, and avoiding the need of remanding the case
for retrial or reception of improperly excluded evidence, with the

possibility thereafter of still another appeal, with all the concomitant


delays. The rule, however, imposes the condition by the same token that if
his demurrer is granted by the trial court, and the order of dismissal is
reversed on appeal, the movant loses his right to present evidence in his
behalf and he shall have been deemed to have elected to stand on the
insufficiency of plaintiffs case and evidence. In such event, the appellate
court which reverses the order of dismissal shall proceed to render
judgment on the merits on the basis of plaintiffs evidence (Republic vs.
Tuvera,).

I. Judicial action on demurrer to evidence is discretionary


A judicial action on a motion to dismiss on demurrer to evidence
rests within the sound discretion of the court. In addition, an order denying
a demurrer to evidence is interlocutory. It is not appealable. Neither can
it be the subject of a petition for certiorari in the absence of grave abuse of
discretion or excess of jurisdiction, or an oppressive exercise of judicial
authority (Katigbak vs. Sandiganbayan,).
As experienced in the Bench, petitions for certiorari under Rule 65
against the denial of demurrer to evidence (in criminal cases) are usually
filed by aggrieved parties arguing that a one or two liner sentences
denying the demurrer and the motion for reconsideration for utter lack of
merit violates Section 1, Rule 36 of the 1997 Revised Rules of Civil
Procedure, and Section 14, Article VIII of the 1987 Constitution.
Section 14, Article VIII of the 1987 Constitution
provides that No decision shall be rendered by any court
without expressing therein clearly and distinctly the facts
and the law on which it is based.
Section 1, Rule 36 of the 1997 Revised Rules of Civil
Procedure provides that A judgment or final order
determining the merits of the case shall be in writing
personally and directly prepared by the judge, stating
clearly and distinctly the facts and the law on which is it
is based, signed by him, and filed with the clerk of court.

This is not a good ground of filing a petition for certiorari under


Rule 65.
An Order resolving a demurrer to evidence has double-character:
first, a final order, and second, as an interlocutory order. If it is a final
order, the, the remedy available to the aggrieved party is appeal, and if it is
an interlocutory order, the remedy is petition for certiorari under Rule 65.
The relevance of having this matter discussed herein is to determine the
importance of observing the decision writing foundations provided in
Section 14, Article VIII of the 1987 Constitution and Section 1, Rule 36 of
the 1997 Revised Rules of Civil Procedure.
The requirement of specificity of rulings is stringently applied only
to judgments and final orders,4 but not to interlocutory one.
If the order denying a demurrer to evidence and the motion for
reconsideration for the denial do not contain clearly and distinctly the facts
and the law on which it is based, it is believed that constitutional and
procedural requirements are not violated because the order is not in the
nature of a final order or judgment that completely disposing of the case.
Judicial technique is sometimes observed by some courts wherein
they will just issue an order of outright denial of demurrer to evidence in
order bit the pressing deadline to resolve pending incidents because of
tremendous judicial paper works on equally important cases rather than
disposing the case on the merit which undeniably requires more precious
time doing research work.
An order granting a demurrer to evidence, both in civil and
criminal cases, partake the nature of a final order, because nothing is left
to the court that issued the order. The issuance of this type of order is the
order (final character) that the trial judge should comply with the requisite
constitutional and procedural mandates. This, the trial judge must state
clearly and distinctly the facts and the law on which is it is based. The
rationale behind why the final order should state clearly and distinctly the
facts and the law on which is it is based is because for the aggrieved party
to easily know why he lost, so that he can easily evaluate what error or
errors that he can assign or impute to the court that issued it if he desires
4

Dandoy vs. CA, G.R. No. 150089, August 28, 2007.

and permitted to elevate the matter on appeal. This is the reason why the
remedy available to the aggrieved party is appeal and not certiorari under
Rule 65.
On the other hand, an order denying a demurrer to evidence is
interlocutory order. The constitutional provision does not apply to
interlocutory order, because after the issuance of the order of denial there
will be continuing proceedings that will takes place, i.e.,
actions/proceedings left to be done by the court, such as such as
presentation of defense evidence. This is the reason why the remedy
available under paragraph 2 (c), in relation to paragraph 3, Section 1,5 Rule
41, 1997 Revised Rules of Civil Procedure is certiorari, and not appeal.
Besides, the order of denial is not subject to the requirement of
Section 14, Article VIII of the 1987 Constitution, because as what the
High Court held in Dandoy vs. CA, the order of denial neither terminate
nor finally dispose of the case as there are proceedings still left to be done
by the court before the case is finally decided on the merits.

I. An order denying the demurrer to evidence reviewable by


Certiorari under Section 1, Rule 65 of the Rules of Court, and
when not
Generally, the order denying the motion for leave of court to file
demurrer to evidence or the demurrer itself shall not be reviewable by
appeal or by certiorari before judgment,6 because action on a demurrer or
on a motion to dismiss rests on the sound exercise of judicial discretion.7 It

No appeal may be taken from: (c) An interlocutory order.


In all the above instances where judgment or final order is not appealable, the aggrieved
party may file an appropriate special civil action under Rule 65.
6
Section 23, Rule 119, 2000 Revised Rules of Criminal Procedure.
7
Tan vs. Court of Appeals, 347 Phil. 320, 329 (1997); Bernardo vs. Court of Appeals, 344
Phil. 335, 346 (1997) cited in Nicolas vs. Sandiganbayan and the companion case, G.R.
Nos. 175930-31, February 11, 2008. Also in People vs. Singh, G.R. No. 129782, June 29,
2001, 360 SCRA 404; People vs. Mercado, No. L-33492, March 30, 1988, 159 SCRA
453 cited in People vs. Almendras, G.R. No. 145915, April 24, 2003.
5

is an interlocutory order, not appealable neither can it be the subject of a


petition for certiorari.8
However, this admits an exception: when the denial of a demurrer
to evidence is attended by grave abuse of discretion, patently erroneous or
issued with grave abuse of discretion.9 Though interlocutory in character,
an order denying a demurrer to evidence may be the subject of a certiorari
proceeding, provided the petitioner can show that it was issued with grave
abuse of discretion; and that appeal in due course is not plain, adequate or
speedy under the circumstances. When the plaintiffs evidence is utterly
and patently insufficient to prove the complaint, it would be capricious for
a trial judge to deny the demurrer and to require the defendant to present
evidence to controvert a non- existing case. The denial of the demurrer to
evidence will constitute an unwelcome imposition on the courts docket
and an assault on the defendants resources and peace of mind, and if
denied, it effectively denies justice.10
---0---

POST JUDGMENT REMEDIES


Fairness dictates that a party who has not appealed from a
judgment of the trial court is bound by the terms of the judgment.11
Lets say plaintiff, the party that prevails in the decision, upon
finality of the judgment he should move to execute the same to enjoy the
fruits and result of his litigation.
But, how if the loosing party wants to exhaust other remedies to
protect his interest subject of the suit, what are his available remedies?

David vs. Rivera, 464 Phil. 1006; Tadeo vs. People, 360 Phil. 914, 919 (1998); Cruz vs.
People, 363 Phil. 156; Katigbak vs. Sandiganbayan, 453 Phil. 515 cited in Nicolas vs.
Sandiganbayan and the companion case, G.R. Nos. 175930-31, February 11, 2008.
9
People vs. Ong, G.R. No. 140904, October 9, 2000 citing Cruz vs. People (303 SCRA
533 [1999]).
10
Choa vs. Choa, G.R. No. 143376, November 26, 2002.
11
Five Star Bus, Co, Inc. vs. CA, G.R. No. 120496, July 17, 1996.

Depending upon each applicability requirement, the following are


the remedies available to the loosing or aggrieved party, namely:
1.
2.
3.
4.
5.
6.

execution of judgment
appeal
motion for reconsideration
motion for new trial
petition for relief from judgment
annulment of judgment

EXECUTION OF JUDGMENT
(Rule 39)
Litigation must end and terminate sometime and somewhere, and it
is essential to an effective and efficient administration of justice that once
a judgment has become final, the winning party be not, through a mere
subterfuge, deprived of the fruits of the verdict (Li Kim Tho vs. Sanchez,).
Once a judgment becomes final, it is basic that the prevailing party
is entitled as a matter of right to a writ of execution the issuance of which
is the trial courts ministerial duty (Torno vs. IAC,).
Final and executory judgments are enforced by a writ of execution
not by mandamus.12 Execution is the fruit and end of the suit and is very
aptly called the life of the law.13 It is the process which carries into effect
a decree or judgment.14
Execution of a judgment can be issued only against a party to the
action and not against one who did not have his day in court (St. Dominic
Corporation vs. IAC,), and therefore, a strangers to a case are not bound
by the judgment rendered by a court. It will not divest the rights of a party

12

Vital-Gozon vs. CA, G.R. No. 101428, August 5, 1992.


Ipekdjian Merchandising Co. vs. Court of Tax Appeals, 8 SCRA 59 cited in PAL, Inc.
vs. CA, G.R. No. L-49188, January 30, 1990.
14
Painter v. Berglund, 31 Cal. App. 2d. 63, 87 P 2d 360, 363; Miller v. London, 294 Mass
300, 1 NE 2d 198, 200; Black's Law Dictionary cited in PAL, Inc. vs. CA, G.R. No. L49188, January 30, 1990.
13

who has not and never been a party to a litigation (Panotes vs. City
Townhouse Development Corporation,).

Execution of judgment vs. Cash


The sheriff shall demand judgment obligor the immediate payment
in CASH of the full stated in the writ of execution and all lawful fees.
He is not required to give the judgment debtor some time to raise
cash because the property may be placed in danger of being lost or
absconded, otherwise the sheriff may be administratively disciplined
(Torres vs. Cabling, July 11, 1997).
The executing sheriff is prohibited from demanding payment by
check payable to him (Section 9 (a), Rule 39, 1997 Revised Rules of Civil
Procedure).

A. Third party claim


When a third person claims interest in the property levied upon, his
remedy is to file a third party claim (or tercera).
When the sheriff seizes property of a third person in which the
judgment debtor holds no right or interest, the supervisory power of the
court which has authorized execution may be invoked by the third person.
Upon due application by the third person, and after summary hearing, the
court may command that the property be released from the mistaken levy
and restored to the rightful owner or possessor. If that is the case, the
courts duty is limited to determine whether the sheriff has acted rightly or
wrongly in the performance of his duties in the execution of judgment,
more specifically, if the sheriff has indeed taken hold of the property not
belonging to the judgment debtor. The court does not and cannot pass
upon the question of title to the property, with any character of finality; it
can treat that matter only in so far as may be necessary to decide if the
sheriff has acted correctly or not (Quebral vs. Garduno,).

The court can require the sheriff to restore the property to the
claimants possession if warranted by the evidence; if claimants proofs do
not however persuade the court his title or right of possession thereof, his
claim will be denied (Ong vs. Tating, G).

B. Execution of Judgment vs. Bank Deposits


Execution of judgment against bank deposits is through
garnishment.
Garnishment has been defined as a species of attachment for
reaching any property or credits pertaining or payable to a judgment
debtor. In legal contemplation, it is a forced novation by the substitution
of creditors: the judgment debtor, who is the original creditor of the
garnishee is, through service of the writ of garnishment, substituted by the
judgment creditor who thereby becomes creditor of the garnishee.
Garnishment has also been described as a warning to a person having in
his possession property or credits of the judgment debtor, not to pay the
money or deliver the property to the latter, but rather to appear and answer
the plaintiff's suit (Perla Compania De Seguros, Inc., vs. Judge
Ramolete,).

C. Alias writ of execution be issued without a prior return of the


original writ by the implementing officer
The issue of whether an alias writ of execution be issued without a
prior return of the original writ by the implementing officer had already
been settled by the High Court in PAL, Inc. vs. CA,. Resolving in the
affirmative, the High Court said in PAL:
We rule in the affirmative and we quote the
respondent court's decision with approval:
"The issuance of the questioned alias writ
of execution under the circumstances here
obtaining is justified because even with the
absence of a Sheriff's return on the original writ,

the unalterable fact remains that such a return is


incapable of being obtained (sic) because the
officer who is to make the said return has
absconded and cannot be brought to the Court
despite the earlier order of the court for him to
appear for this purpose. (Order of Feb. 21, 1978,
Annex C, Petition). Obviously, taking
cognizance of this circumstance, the order of
May 11, 1978 directing the issuance of an alias
writ was therefore issued. (Annex D, Petition).
The need for such a return as a condition
precedent for the issuance of an alias writ was
justifiably dispensed with by the court below
and its action in this regard meets with our
concurrence. A contrary view will produce an
abhorent situation whereby the mischief of an
erring officer of the court could be utilized to
impede indefinitely the undisputed and awarded
rights which a prevailing party rightfully
deserves to obtain and with dispatch. The final
judgment in this case should not indeed be
permitted to become illusory or incapable of
execution for an indefinite and over extended
period, as had already transpired." (Rollo, pp.
35-36)
Judicium non debet esse illusorium; suum effectum
habere debet (A judgment ought not to be illusory; it ought
to have its proper effect).
Indeed, technicality cannot be countenanced to defeat
the execution of a judgment for execution is the fruit and
end of the suit and is very aptly called the life of the law
(Ipekdjian Merchandising Co. v. Court of Tax Appeals, 8
SCRA 59 [1963]; Commissioner of Internal Revenue v.
Visayan Electric Co., 19 SCRA 697, 698 [1967]). A
judgment cannot be rendered nugatory by the unreasonable
application of a strict rule of procedure. Vested rights were
never intended to rest on the requirement of a return, the

office of which is merely to inform the court and the


parties, of any and all actions taken under the writ of
execution. Where such information can be established in
some other manner, the absence of an executing officer's
return will not preclude a judgment from being treated as
discharged or being executed through an alias writ of
execution as the case may be. More so, as in the case at bar.
Where the return cannot be expected to be forthcoming, to
require the same would be to compel the enforcement of
rights under a judgment to rest on an impossibility, thereby
allowing the total avoidance of judgment debts. So long as
a judgment is not satisfied, a plaintiff is entitled to other
writs of execution (Government of the Philippines v.
Echaus and Gonzales, 71 Phil. 318). It is a well known
legal maxim that he who cannot prosecute his judgment
with effect, sues his case vainly.

G. Execution cannot be equated with satisfaction of a judgment;


execution and satisfaction of judgment, distinguished
PAL, Inc. vs. CA, said that execution is the process which carries
into effect a decree or judgment (Painter v. Berglund, 31 Cal. App. 2d. 63,
87 P 2d 360, 363; Miller v. London, 294 Mass 300, 1 NE 2d 198, 200;
Black's Law Dictionary), whereas the satisfaction of a judgment is the
payment of the amount of the writ, or a lawful tender thereof, or the
conversion by sale of the debtor's property into an amount equal to that
due, and, it may done otherwise, than upon an execution (Section 47, Rule
39). Levy and delivery by an execution officer are not prerequisites to the
satisfaction of a judgment when the same has already been realized in fact
(Section 47, Rule 39). Execution is for the sheriff to accomplish while
satisfaction of the judgment is for the creditor to achieve. Section 15,
Rule 39 merely provides the sheriff with his duties as executing officer
including delivery of the proceeds of his levy on the debtor's property to
satisfy the judgment debt. It is but to stress that the implementing officer's
duty should not stop at his receipt of payments but must continue until
payment is delivered to the obligor or creditor (G.R. No. L-49188,
January 30, 1990).
H. Motion to quash execution when proper

A motion to quash execution is only proper where:


(a) the writ of execution varies the judgment;
(b) there has been a change in the situation of the parties
making execution inequitable or unjust;
(c) execution is sought to be enforced against property
exempt from execution;
(d) it appears that the controversy has never been submitted
to the judgment of the court;
(e) the terms of the judgment are not clear enough and there
remains room for interpretation thereof; or
(f) it appears that the writ of execution has been
improvidently issued, or that it is defective in substance
or is issued against the wrong party, or that the
judgment debt has been paid or otherwise satisfied, or
the writ was issued without authority (Gutierrez vs,
Valiente,).

I. Execution pending appeal


On motion of the prevailing party will notice to the adverse party
filed in the trial court while it has jurisdiction over the case and is in
possession of either the original record or the record on appeal, as the case
may be, at the time of the filing of such motion, said court may, in its
discretion, order execution of a judgment or final order even before the
expiration of the period to appeal.
After the trial court has lost jurisdiction, the motion for execution
pending appeal may be filed in the appellate court.

Discretionary execution may only issue upon good reasons to be


stated in a special order after due hearing (Section 2, Rule 39, 1997
Revised Rules of Civil Procedure).
Discretionary execution of appealed judgments may be allowed
upon concurrence of the following requisites:
(a) there must be a motion by the prevailing party with notice
to the adverse party;
(b) there must be a good reason for execution pending
appeal; and
(c) the good reason must be stated in a special order
(Manacop vs. Equitable Banking Corporation,).
The exercise of the power to grant or deny immediate execution is
addressed to the sound discretion of the court and the existence of good
reasons is precisely what confers such discretionary power upon the court
(Philippine National Bank vs. Puno,).
J. Good Reasons
Good reasons consist of compelling circumstances justifying
immediate execution lest judgment becomes illusory, or the prevailing
party after the lapse of time be unable to enjoy it, considering the tactics of
the adverse party who may have apparently no cause but to delay. Such
reasons must constitute superior circumstances demanding urgency which
will outweigh the injury or damages should the losing party secure a
reversal of the judgment. Execution of a judgment pending appeal is an
exception to the general rule that only a final judgment may be executed.
Thus, the existence of good reasons is essential for it is what confers
discretionary power on a court to issue a writ of execution pending appeal
(Villamor vs. National Power Corporation,).

J. Supersedeas Bond

The purpose of the supersedeas bond is to answer for the rents,


damages and costs accruing down to the judgment of the inferior court
appealed from, the amount of which is to be determined from the
judgment of said court. The postulation of complainants and their counsel
that the execution sought was effectively stayed by the filing of a
supersedeas bond was sufficiently refuted and justifiably rejected when we
consider the circumstances then obtaining (Cordova vs. Judge Labayen,
A).
---0---

REVIVAL OF JUDGMENT
A. The principle and Rule
An action for revival of judgment is no more than a procedural
means of securing the execution of a previous judgment which has
become dormant after the passage of five years without it being executed
upon motion of the prevailing party. It is not intended to re-open any issue
affecting the merits of the judgment debtors case nor the propriety or
correctness of the first judgment (Panotes, vs. City Townhouse
Development Corporation,).
It is premised on the assumption that the decision to be revived,
either by motion or by independent action, is already final and executory
(Saligumba vs. Palanog,).
A final and executory judgment or order may be executed on
motion within five (5) years from the date of its entry. After the lapse of
such time, and before it is barred by the statute of limitations, a judgment
may be enforced by action. The revived judgment may also be enforced by
motion within five (5) years from the date of its entry and thereafter by
action before it is barred by the statute of limitations (Section 6, Rule 39,
1997 Revised Rules of Civil Procedure [Execution by motion or by
independent action]).

The purpose of the law in prescribing time limitations for


enforcing judgment by action is precisely to prevent the winning parties
from sleeping on their rights (Macias vs. Lim,).

B. It is a new and independent action


An action for revival of judgment is a new and independent action,
different and distinct from either the recovery of property case or the
reconstitution case, wherein the cause of action is the decision itself and
not the merits of the action upon which the judgment sought to be
enforced is rendered (Juco vs. Heirs of Tomas Siy Chung Fu,).
C. Period within which to file action for revival of judgment
An action may be revived by mere motion within the period
between five (5) years from the date of entry of judgment, and by
independent action within the period or between five (5) years and one (1)
day from the date of entry of judgment to ten (10) years.
The guide in filing a revival of action is that: first, observe the 10
year prescription period counted from the date said judgment become final
or from the date of its entry, and second, observe the five (5) year period
mentioned in Section 6, Rule 39 of the Rules of Court final and
executory judgment or order may be executed on motion within five (5)
years from the date of its entry.
Again, this may be by mere motion or by action. If the action for
revival of judgment is filed after the five (5) year period provided for in
the Rules of Court, but beyond the ten (10) year period provided for in the
Civil Code, the action is considered barred by the statute of limitations,
thus, an action can no longer be enforced by action.
In Quesada vs. CA, , a writ of execution was issued on September
15, 1976, but was not enforced. An action for revival of judgment was
filed on August 26, 1985. The CA ruled that the action was already barred
by prescription. The High Court ruled that the action to revived judgment
was not barred by prescription because it was filed within the 10 year
period.

The period within which to file action for revival of judgment was
explained by the High Court in Shipside Incorporated vs. CA, , thus:
The action instituted by the Solicitor General in the
trial court is one for revival of judgment which is governed
by Article 1144(3) of the Civil Code and Section 6, Rule 39
of the 1997 Rules on Civil Procedure. Article 1144(3)
provides that an action upon a judgment must be brought
within 10 years from the time the right of action accrues."
On the other hand, Section 6, Rule 39 provides that a final
and executor judgment or order may be executed on motion
within five (5) years from the date of its entry, but that after
the lapse of such time, and before it is barred by the statute
of limitations, a judgment may be enforced by action.
Taking these two provisions into consideration, it is plain
that an action for revival of judgment must be brought
within ten years from the time said judgment becomes
final.
In this case (Shipside Incorporated), the High Court
said that:
From the records of this case, it is clear
that the judgment sought to be revived became
final on October 23, 1973. On the other hand,
the action for revival of judgment was instituted
only in 1999, or more than twenty-five (25)
years after the judgment had become final.
Hence, the action is barred by extinctive
prescription considering that such an action can
be instituted only within ten (10) years from the
time the cause of action accrues.
0-0-0
In Villeza vs. German Management and Services, Inc, , made to
emphasize that an action for revival of judgment is governed by Article
1144 (3), Article 1152 of the Civil Code and Section 6, Rule 39 of the
Rules of Court; and that judgment must be enforced by the institution of a

complaint in a regular court within ten years from the time the judgment
becomes final.
The High Court said:
Art. 1144. The following actions must be
brought within ten years from the time the right of
action accrues:
xxxx
(3) Upon a judgment
Article 1152 of the Civil Code states:
Art. 1152. The period for prescription of
actions to demand the fulfillment of obligations
declared by a judgment commences from the
time the judgment became final.
Apropos, Section 6, Rule 39 of the Rules of Court reads:
Sec. 6.
Execution by motion or by
independent action. A final and executory
judgment or order may be executed on motion
within five (5) years from the date of its entry.
After the lapse of such time, and before it is
barred by the statute of limitations, a judgment
may be enforced by action. The revived
judgment may also be enforced by motion
within five (5) years from the date of its entry
and thereafter by action before it is barred by
the statute of limitations. (emphasis supplied)
The rules are clear. Once a judgment becomes final
and executory, the prevailing party can have it executed as
a matter of right by mere motion within five years from the
date of entry of judgment. If the prevailing party fails to
have the decision enforced by a motion after the lapse of

five years, the said judgment is reduced to a right of action


which must be enforced by the institution of a complaint in
a regular court within ten years from the time the judgment
becomes final.
When petitioner Villeza filed the complaint for revival of
judgment on October 3, 2000, it had already been eleven (11) years from
the finality of the judgment he sought to revive. Clearly, the statute of
limitations had set in said the High Court.15
The High Court added: xxx xxx xxx it was petitioner Villeza, the
prevailing party himself, who moved to defer the execution of judgment.
The losing party never had any hand in the delay of its execution. Neither
did the parties have any agreement on that matter. After the lapse of five
years (5) from the finality of judgment, petitioner Villeza should have
instead filed a complaint for its revival in accordance with Section 6, Rule
39 of the Rules of Court. He, however, filed a motion to execute the same
which was a wrong course of action. On the 11th year, he finally sought
its revival but he requested the aid of the courts too late.16
-0-0-0D. Instances that prescriptive period within which to revive action
is interrupted or tolled
(1). If it is impossible for the winning party to have sought the
execution of the judgment because of the dilatory schemes and maneuvers
resorted to by the other party (Republic vs. CA,).
(2). There is an agreement between parties to defer or suspend the
enforcement of the judgment, or when the order of ejectment was not
carried out, however, due to the judgment debtors begging to withhold
the execution of judgment because of financial difficulties (Torralba vs.
delos Angeles,).

15

Villeza vs. German Management and Services, Inc, G.R. No. 182937, August 8, 2010.
Villeza vs. German Management and Services, Inc, G.R. No. 182937, August 8, 2010.

16

(3). When the oblior moves for the suspension of the writ of
execution, or when the obligee was not in delay because he exhausted all
legal means within his power to eject the obligor from his land; or where
the writs of execution issued by the lower court were not complied with
and/or were suspended by reason of acts or causes not of obligees own
making and against his objections (Casela vs. CA).

E. Venue of revival of judgment involving personal action (accion


in personam) and real action
In revival of judgment, if the revival of judgment action is a
personal one (accion in personam), the venue of the action is in the
residence of the plaintiff, or at the latters option, residence of the
defendant, and if the revival of judgment action involves a real action, the
venue of the action is in the place where the property subject of the revival
action is situated (Infante vs, Aran Builders,).
The High Court in Infante:
xxx xxx xxx. Petitioner claims that the CA erred in
finding that the complaint for revival of judgment is an
action in rem which was correctly filed with the RTC of the
place where the disputed real property is located.
The petition is unmeritorious.
Petitioner insists that the action for revival of
judgment is an action in personam; therefore, the complaint
should be filed with the RTC of the place where either
petitioner or private respondent resides. Petitioner then
concludes that the filing of the action for revival of
judgment with the RTC of Muntinlupa City, the place
where the disputed property is located, should be dismissed
on the ground of improper venue.
Private respondent is of the opinion that the judgment
it is seeking to revive involves interest over real property.

As such, the present action for revival is a real action, and


venue was properly laid with the court of the place where
the realty is located.
Thus, the question that must be answered is: where is
the proper venue of the present action for revival of
judgment?
Section 6, Rule 39 of the 1997 Rules of Civil
Procedure provides that after the lapse of five (5) years
from entry of judgment and before it is barred by the statute
of limitations, a final and executor judgment or order may
be enforced by action. The Rule does not specify in which
court the action for revival of judgment should be filed.
In Aldeguer v. Gemelo, the Court held that:
x x x an action upon a judgment must be
brought either in the same court where said
judgment was rendered or in the place where the
plaintiff or defendant resides, or in any other
place designated by the statutes which treat
of the venue of actions in general. (Emphasis
supplied)
but emphasized that other provisions in the rules of
procedure which fix the venue of actions in general must be
considered.
Under the present Rules of Court, Sections 1 and 2 of
Rule 4 provide:
Section 1.
Venue of real actions. Actions affecting title to or possession of real
property, or interest therein, shall be
commenced and tried in the proper court which
has jurisdiction over the area wherein the real
property involved, or a portion thereof, is
situated.

xxxx
Section 2.
Venue of personal actions. All other actions may be commenced and tried
where the plaintiff or any of the principal
plaintiffs resides, or where the defendant or any
of the principal defendants resides, or in the
case of a non-resident defendant where he may
be found, at the election of the plaintiff.
Thus, the proper venue depends on the determination
of whether the present action for revival of judgment is a
real action or a personal action. Applying the afore-quoted
rules on venue, if the action for revival of judgment affects
title to or possession of real property, or interest therein,
then it is a real action that must be filed with the court of
the place where the real property is located. If such action
does not fall under the category of real actions, it is then a
personal action that may be filed with the court of the place
where the plaintiff or defendant resides.
In support of her contention that the action for revival
of judgment is a personal action and should be filed in the
court of the place where either the plaintiff or defendant
resides, petitioner cites the statements made by the Court in
Aldeguer v. Gemelo and Donnelly v. Court of First Instance
of Manila. Petitioner, however, seriously misunderstood
the Court's rulings in said cases.
In Aldeguer, what the Court stated was that [t]he
action for the execution of a judgment for damages is a
personal one, and under section 377 [of the Code of Civil
Procedure], it should be brought in any province where the
plaintiff or the defendant resides, at the election of the
plaintiff (Emphasis and underscoring supplied). Petitioner
apparently took such statement to mean that any action for
revival of judgment should be considered as a personal one.
This thinking is incorrect. The Court specified that the
judgment sought to be revived in said case was a judgment

for damages.
The judgment subject of the action for
revival did not involve or affect any title to or possession of
real property or any interest therein. The complaint filed in
the rivival case did not fall under the category of real
actions and, thus, the action necessarily fell under the
category of personal actions.
In Donnelly, the portion of the Decision being relied
upon by petitioner stated thus:
Petitioner raises before this Court two (2)
issues, namely: (a) whether an action for revival
of judgment is one quasi in rem and, therefore,
service of summons may be effected thru
publication; and (b) whether the second action
for revival of judgment (Civil Case No. 76166)
has already prescribed. To our mind, the first
is not a proper and justiciable issue in the
present proceedings x x x. Nevertheless, let it
be said that an action to revive a judgment is a
personal one. (Emphasis supplied)
The Court clearly pointed out that in said case, the
issue on whether an action for revival of judgment is quasi
in rem was not yet proper and justiciable. Therefore, the
foregoing statement cannot be used as a precedent, as it
was merely an obiter dictum. Moreover, as in Aldeguer,
the judgment sought to be revived in Donnelly involved
judgment for a certain sum of money. Again, no title or
interest in real property was involved.
It is then
understandable that the action for revival in said case was
categorized as a personal one.
Clearly, the Court's classification in Aldeguer and
Donnelly of the actions for revival of judgment as being
personal in character does not apply to the present case.

The allegations in the complaint for revival of


judgment determine whether it is a real action or a personal
action.
The complaint for revival of judgment alleges that a
final and executor judgment has ordered herein petitioner to
execute a deed of sale over a parcel of land in Ayala
Alabang Subdivision in favor of herein private respondent;
pay all pertinent taxes in connection with said sale; register
the deed of sale with the Registry of Deeds and deliver to
Ayala Corporation the certificate of title issued in the name
of private respondent. The same judgment ordered private
respondent to pay petitioner the sum of P321,918.25 upon
petitioner's compliance with the aforementioned order. It is
further alleged that petitioner refused to comply with her
judgment obligations despite private respondent's repeated
requests and demands, and that the latter was compelled to
file the action for revival of judgment. Private respondent
then prayed that the judgment be revived and a writ of
execution be issued to enforce said judgment.
The previous judgment has conclusively declared
private respondent's right to have the title over the disputed
property conveyed to it. It is, therefore, undeniable that
private respondent has an established interest over the lot in
question; and to protect such right or interest, private
respondent brought suit to revive the previous judgment.
The sole reason for the present action to revive is the
enforcement of private respondent's adjudged rights over a
piece of realty. Verily, the action falls under the category
of a real action, for it affects private respondent's interest
over real property.
The present case for revival of judgment being a real
action, the complaint should indeed be filed with the
Regional Trial Court of the place where the realty is
located.
Section 18 of Batas Pambansa Bilang 129 provides:

Sec. 18.
Authority to define territory
appurtenant to each branch. - The Supreme
Court shall define the territory over which a
branch of the Regional Trial Court shall
exercise its authority. The territory thus
defined shall be deemed to be the territorial
area of the branch concerned for purposes of
determining the venue of all suits,
proceedings or actions, whether civil or
criminal, as well as determining the
Metropolitan Trial Courts, Municipal Trial
Courts and Municipal Circuit Trial Courts over
which the said branch may exercise appellate
jurisdiction. The power herein granted shall be
exercised with a view to making the courts
readily accessible to the people of the different
parts of the region and making the attendance of
litigants and witnesses as inexpensive as
possible. (Emphasis supplied)
From the foregoing, it is quite clear that a branch of
the Regional Trial Court shall exercise its authority only
over a particular territory defined by the Supreme
Court.
Originally, Muntinlupa City was under the
territorial jurisdiction of the Makati Courts. However,
Section 4 of Republic Act No. 7154, entitled An Act to
Amend Section Fourteen of Batas Pambansa Bilang 129,
Otherwise Known As The Judiciary Reorganization Act of
1981, took effect on September 4, 1991. Said law provided
for the creation of a branch of the Regional Trial Court in
Muntinlupa. Thus, it is now the Regional Trial Court in
Muntinlupa City which has territorial jurisdiction or
authority to validly issue orders and processes concerning
real property within Muntinlupa City.
Thus, there was no grave abuse of discretion
committed by the Regional Trial Court of Muntinlupa City,
Branch 276 when it denied petitioner's motion to dismiss;
and the CA did not commit any error in affirming the same.

F. Judgment does not attain finality when record is destroyed


during pendency of motion for reconsideration

When the records of the original case were destroyed in the fire
during the pendency of the motion for reconsideration of the disapproval
of the record on appeal, a motion for reconsideration has the effect of
suspending the statutory period after which an order, decision, or
judgment, in connection with which said motion was filed, becomes final.
The motion for reconsideration prevents the decision from attaining
finality. Cannot therefore be a proper subject of an action for revival of
judgment (Juco vs. Heirs of Tomas Siy Chung Fu,).
Juco said:
As pointed out by the appellate court, an action for
revival of judgment is a new and independent action,
different and distinct from either the recovery of property
case or the reconstitution case, wherein the cause of action
is the decision itself and not the merits of the action upon
which the judgment sought to be enforced is rendered.
However, revival of judgment is premised on the
assumption that the decision to be revived, either by motion
or by independent action, is already final and executory.
Hence, the need to make a determination of whether or not
the decision in Civil Case No. 7281 has indeed become
final and executory. For if the subject decision has already
reached finality, then the conclusion of the appellate court
is correct that the dismissal of the reconstitution case would
not prevent respondents from reviving and thereafter
executing the said decision.
A decision issued by a court is final and executory
when such decision disposes of the subject matter in its
entirety or terminates a particular proceeding or action,
leaving nothing else to be done but to enforce by execution
what has been determined by the court, such as when after
the lapse of the reglementary period to appeal, no appeal

has been perfected. In the case at bar, it is an undisputed


fact that when the records of the original case were
destroyed in the fire there was a pending motion for
reconsideration of the disapproval of the record on appeal
filed by petitioner. A motion for reconsideration has the
effect of suspending the statutory period after which an
order, decision, or judgment, in connection with which said
motion was filed, becomes final. In effect, such motion for
reconsideration has prevented the decision from attaining
finality.
The findings of the Court of Appeals that
notwithstanding the pendency of the motion for
reconsideration, the decision in Civil Case No. 7281 has
become final and executory by reason of laches cannot be
sustained. As discussed above, the doctrine of laches
cannot operate to lend finality to the decision since
petitioners failure to pursue the motion for reconsideration
was not due to her negligence or abandonment, but was
rather brought upon by the dismissal of the reconstitution
case. Therefore, it is clear that the case has not reached
finality at the time the records of the case were burnt.

---0---

APPEAL
A. Right to appeal is a statutory privilege
The right to appeal is a statutory privilege and of statutory origin,
not a constitutional, natural or inherent right. Therefore, available only if
granted or as provided by statutes. It may be exercised only in the manner
prescribed by the provisions of the law. The period to appeal is
specifically governed by Section 39 of Batas Pambansa Blg. 129 (BP
129), as amended amd Section 3 of Rule 41 of the 1997 Rules of Civil
Procedure (Yu vs. Judge Samson-Tatad,).

B. When to appeal?
The period for appeal from final orders, resolutions, awards,
judgments, or decisions of any court in all cases shall be fifteen (15) days
counted from the notice of the final order, resolution, award, judgment, or
decision appealed from (Section 39, BP 129, as amended).
Under Section 39 of Batas Pambansa Bilang (B.P. Blg.) 129, the
petitioners had fifteen (15) days within which to file their notice of appeal,
from the time their counsel received notice or was served a copy of the
trial courts decision. The fifteen-day period provided therein is mandatory
and jurisdictional. It bears stressing that the right to appeal is not a natural
right or a part of due process. It is a procedural remedy of statutory origin
and, as such, may be exercised only in the manner and within the time
frame provided by the provisions of law authorizing its exercise. Failure
of a party to perfect an appeal within the period fixed by law renders the
decision sought to be appealed final and executory. As a result, no court
could exercise appellate jurisdiction to review the decision. After a
decision is declared final and executory, vested rights are acquired by the
winning party who has the right to enjoy the finality of the case (Alon vs.
CA, ).
Section 39, BP 129, as amended, is the substantive source of the
remedy of appeal, while Section 3, Rule 41 of the 1997 Rules of Civil
Procedure is the procedural source.

In Ortigas and Company Limited Partnership v. Velasco, this Court


expounded on the interpretation of certain phrases used by the courts in
their judgments or resolutions
C. Denial of motion for reconsideration with words with finality
is of no consequence
While the denial of a motion for reconsideration of a judgment or
final order is normally accompanied by the modifier, final, or with
finality, there may be a denial not so qualified. That is of no

consequence. By no means may it be taken as indicating any uncertainty


or indecisiveness on the part of the Court regarding its denial of
reconsideration, or an encouragement or expectation of a second motion
for reconsideration. The modifier serves simply to emphasize the import
and effect of the denial of the motion for reconsideration, i.e., that the
Court will entertain and consider no further arguments or submissions
from the parties respecting its correctness; that in the Courts considered
view, nothing more is left to be discussed, clarified or done in the case, all
issues raised having been passed upon and definitely resolved, and any
other which could have been raised having been waived and no longer
being available as ground for a second motion. A denial with finality
stresses that the case is considered closed (Ortigas and Company Limited
Partnership v. Velasco,).

APPEALS
FROM FIRST TO SECOND LEVEL COURT
A. How, where and when to appeal?
The appeal is taken by filing a notice of appeal with the court that
rendered the judgment or final order appealed from. The notice of appeal
shall indicate the parties to the appeal, the judgment or final order or part
thereof appealed from, and state the amterial dates showing the timeliness
of the appeal (Section 3, Rule 40, 1997 Revised Rules of Civil
Procedure).
An appeal from a judgment or final order of a Municipal Trial
Court may be taken to the Regional Trial Court exercising jurisdiction
over the area to which the former pertains. The title of the case shall
remain as it was in the court of origin, but the party appealing the case
shall be further referred to as the appellant and the adverse party as the
appellee (Section 1, Rule 40, 1997 Revised Rules of Civil Procedure).
An appeal may be taken within fifteen (15) days after notice to the
appellant of the judgment or final order appealed from. Where a record on
appeal is required, the appellant shall file a notice of appeal and a record
on appeal within thirty (30) days after notice of the judgment or final order
(Section 2, Rule 40, 1997 Revised Rules of Civil Procedure).

The period of appeal shall be interrupted by a timely motion for


new trial or reconsideration. No motion for extension of time to file a
motion for new trial or reconsideration shall be allowed (Paragraph 2,
Section 2, Rule 40, 1997 Revised Rules of Civil Procedure).

B. Perfection of appeals and its effects


According to Section 4 of Rule 40, 1997 Revised Rules of Civil
Procedure, the perfection of the appeal and the effect thereof shall be
governed by the provisions of Section 9, Rule 41.
Section 4 of Rule 40, 1997 Revised Rules of Civil Procedure says:
Section 9. Perfection of appeal; effect thereof. A
party's appeal by notice of appeal is deemed perfected as to
him upon the filing of the notice of appeal in due time.
A party's appeal by record on appeal is deemed
perfected as to him with respect to the subject matter
thereof upon the approval of the record on appeal filed in
due time.
In appeals by notice of appeal, the court loses
jurisdiction over the case upon the perfection of the appeals
filed in due time and the expiration of the time to appeal of
the other parties.
In appeals by record on appeal, the court loses
jurisdiction only over the subject matter thereof upon the
approval of the records on appeal filed in due time and the
expiration of the appeal of the other parties.
In either case, prior to the transmittal of the original
record or the record on appeal, the court may issue orders
for the protection and preservation of the rights of the
parties which do not involve any matter litigated by the
appeal, approve compromises, permit appeals of indigent

litigants, order execution pending appeal in accordance


with 2 of Rule 39, and allow withdrawal of the appeal.

C. Payment of appellate and another lawful fees is a requirement


The general rule is that appeal is perfected by filing a notice of
appeal and paying the requisite docket fees and other lawful fees
(Baniqued v. Ramos, cited in Tanenglian vs.. Lorenzo, G.R. No. 173415,
March 28, 2008).
Within the period for taking an appeal, the appellant shall pay to
the clerk of the court which rendered the judgment or final order appealed
from the full amount of the appellate court docket and other lawful fees.
Proof of payment thereof shall be transmitted to the appellate court
together with the original record or the record on appeal, as the case may
be (Section 5, Rule 40, 1997 Revised Rules of Civil Procedure).

D. Duty of clerk of court of the first level court upon perfection of


appeal
Within fifteen (15) days from the perfection of the appeal, the clerk
of court or the branch clerk of court of the lower court shall transmit the
original record or the record on appeal, together with the transcripts and
exhibits, which he shall certify as complete, to the proper Regional Trial
Court. A copy of his letter of transmittal of the records to the appellate
court shall be furnished the parties (Section 5, Rule 40, 1997 Revised
Rules of Civil Procedure).

E. Duty of clerk of court of the second level court (appellate court)


upon receipt of record
Upon receipt of the complete record or the record on appeal, the
clerk of court of the Regional Trial Court shall notify the parties of such
fact (Section 7 (a), Rule 40, 1997 Revised Rules of Civil Procedure).

Within fifteen (15) days from such notice, it shall be the duty of
the appellant to submit a memorandum which shall briefly discuss the
errors imputed to the lower court, a copy of which shall be furnished by
him to the adverse party. Within fifteen (15) days from receipt of the
appellant's memorandum, the appellee may file his memorandum. Failure
of the appellant to file a memorandum shall be a ground for dismissal of
the appeal (Section 7 (b), Rule 40, 1997 Revised Rules of Civil
Procedure).
Upon the filing of the memorandum of the appellee, or the
expiration of the period to do so, the case shall be considered submitted
for decision. The Regional Trial Court shall decide the case on the basis of
the entire record of the proceedings had in the court of original and such
memoranda as are filed (Section 7 (c), Rule 40, 1997 Revised Rules of
Civil Procedure).
F. Appeal of dismissed case without trial or for lack of jurisdiction
If an appeal is taken from an order of the lower court dismissing
the case without a trial on the merits, the Regional Trial Court may affirm
or reverse it, as the case may be. In case of affirmance and the ground of
dismissal is lack of jurisdiction over the subject matter, the Regional Trial
Court, if it has jurisdiction thereover, shall try the case on the merits as if
the case was originally filed with it. In case of reversal, the case shall be
remanded for further proceedings (Section 8, Rule 40, 1997 Revised Rules
of Civil Procedure).
If the case was tried on the merits by the lower court without
jurisdiction over the subject matter, the Regional Trial Court on appeal
shall not dismiss the case if it has original jurisdiction thereof, but shall
decide the case in accordance with the preceding section, without
prejudice to the admission of amended pleadings and additional evidence
in the interest of justice (Paragraph 2, Section 8, Rule 40, 1997 Revised
Rules of Civil Procedure).

APPEALS FROM SECOND LEVEL COURT


TO COURT OF APPEALS

A. How and when to appeal?


The appeal is taken by filing a notice of appeal with the court that
rendered the judgment or final order appealed from. The notice of appeal
shall indicate the parties to the appeal, the judgment or final order or part
thereof appealed from, and state the amterial dates showing the timeliness
of the appeal (Section 3, Rule 40, 1997 Revised Rules of Civil
Procedure).
The appeal shall be taken within fifteen (15) days from notice of
the judgment or final order appealed from. Where a record on appeal is
required, the appellant shall file a notice of appeal and a record on appeal
within thirty (30) days from notice of the judgment or final order (Section
3, Rule 41, 1997 Revised Rules of Civil Procedure).
The period of appeal shall be interrupted by a timely motion for
new trial or reconsideration. No motion for extension of time to file a
motion for new trial or reconsideration shall be allowed (Paragraph 2,
Section 3, Rule 41, 1997 Revised Rules of Civil Procedure).
The notice of appeal shall indicate the parties to the appeal, specify
the judgment or final order or part thereof appealed from, specify the court
to which the appeal is being taken, and state the material dates showing
the timeliness of the appeal (Section 5, Rule 41, 1997 Revised Rules of
Civil Procedure).

B. Subject of appeal
An appeal may be taken from a judgment or final order that
completely disposes of the case, or of a particular matter therein when
declared by these Rules to be appealable (Section 1, Rule 41, 1997
Revised Rules of Civil Procedure).

C. Appeal is not permitted


No appeal may be taken from:

(a) An order denying a motion for new trial or reconsideration;


(b) An order denying a petition for relief or any similar motion
seeking relief from judgment;
(c) An interlocutory order;
(d) An order disallowing or dismissing an appeal;
(e) An order denying a motion to set aside a judgment by
consent, confession or compromise on the ground of fraud,
mistake or duress, or any other ground vitiating consent;
(f) An order of execution;
(g) A judgment or final order for or against one or more of
several parties or in separate claims, counterclaims, crossclaims and third-party complaints, while the main case is
pending, unless the court allows an appeal therefrom; and
(h) An order dismissing an action without prejudice.
In all the above instances where the judgment or final order is not
appealable, the aggrieved party may file an appropriate special civil action
under Rule 65 (Paragraph 2, Section 1, Rule 41, 1997 Revised Rules of
Civil Procedure).

D. The modes of appeal


The appeal to the Court of Appeals in cases decided by the
Regional Trial Court in the exercise of its original jurisdiction shall be
taken by filing a notice of appeal with the court which rendered the
judgment or final order appealed from and serving a copy thereof upon the
adverse party. No record on appeal shall be required except in special
proceedings and other cases of multiple or separate appeals where law on
these Rules so require. In such cases, the record on appeal shall be filed
and served in like manner (Section 2 (a), Rule 41, 1997 Revised Rules of
Civil Procedure).

This is the so-called ordinary appeal.


The appeal to the Court of Appeals in cases decided by the
Regional Trial Court in the exercise of its appellate jurisdiction shall be by
petition for review in accordance with Rule 42 (Section 2 (b), Rule 41,
1997 Revised Rules of Civil Procedure).
This is the so-called petition for review.
In all cases where only questions of law are raised or involved, the
appeal shall be to the Supreme Court by petition for review on certiorari
in accordance with the Rule 45 (Section 2 (c), Rule 41, 1997 Revised
Rules of Civil Procedure).
This is the so-called appeal by certiorari.

E. Perfection of appeal and its effect


A party's appeal by notice of appeal is deemed perfected as to him
upon the filing of the notice of appeal in due time (Section 9, Rule 41,
1997 Revised Rules of Civil Procedure).
A party's appeal by record on appeal is deemed perfected as to him
with respect to the subject matter thereof upon the approval of the record
on appeal filed in due time (Paragraph 2, Section 9, Rule 41, 1997
Revised Rules of Civil Procedure).
In appeals by notice of appeal, the court loses jurisdiction over the
case upon the perfection of the appeals filed in due time and the expiration
of the time to appeal of the other parties (Paragraph 3, Section 9, Rule
41, 1997 Revised Rules of Civil Procedure).
In appeals by record on appeal, the court loses jurisdiction only
over the subject matter thereof upon the approval of the records on appeal
filed in due time and the expiration of the appeal of the other parties
(Paragraph 4, Section 9, Rule 41, 1997 Revised Rules of Civil
Procedure).

In either case, prior to the transmittal of the original record or the


record on appeal, the court may issue orders for the protection and
preservation of the rights of the parties which do not involve any matter
litigated by the appeal, approve compromises, permit appeals of indigent
litigants, order execution pending appeal in accordance with 2 of Rule 39,
and allow withdrawal of the appeal (Paragraph 5, Section 9, Rule 41,
1997 Revised Rules of Civil Procedure).

F. Duty of clerk of court of the lower court upon perfection of


appeal
Within thirty (30) days after perfection of all the appeals in
accordance with the preceding section, it shall be the duty of the clerk of
court of the lower court:
(a) To verify the correctness of the original record or the record on
appeal, as the case may be aid to make certification of its
correctness;
(b) To verify the completeness of the records that will be,
transmitted to the appellate court;
(c) If found to be incomplete, to take such measures as may be
required to complete the records, availing of the authority that he
or the court may exercise for this purpose; and
(d) To transmit the records to the appellate court.
If the efforts to complete the records fail, he shall indicate in his
letter of transmittal the exhibits or transcripts not included in the records
being transmitted to the appellate court, the reasons for their nontransmittal, and the steps taken or that could be taken to have them
available.
The clerk of court shall furnish the parties with copies of his letter
of transmittal of the records to the appellate court (Section 10, Rule 41,
1997 Revised Rules of Civil Procedure).

Upon the perfection of the appeal, the clerk shall immediately


direct the stenographers concerned to attach to the record of the case five
(5) copies of the transcripts of the testimonial evidence referred to in the
record on appeal. The stenographers concerned shall transcribe such
testimonial evidence and shall prepare and affix to their transcripts an
index containing the names of the witnesses and the pages wherein their
testimonies are found, and a list of the exhibits and the pages wherein each
of them appears to have been offered and admitted or rejected by the trial
court. The transcripts shall be transmitted to the clerk of the trial court
who shall thereupon arrange the same in the order in which the witnesses
testified at the trial, and shall cause the pages to be numbered
consecutively (Section 10, Rule 41, 1997 Revised Rules of Civil
Procedure).
The clerk of the trial court shall transmit to the appellate court the
original record or the approved record on appeal within thirty (30) days
from the perfection of the appeal, together with the proof of payment of
the appellate court docket and other lawful fees, a certified true copy of
the minutes of the proceedings, the order of approval, the certificate of
correctness, the original documentary evidence referred to therein, and the
original and three (3) copies of the transcripts. Copies of the transcripts
and certified true copies of the documentary evidence shall remain in the
lower court for the examination of the parties (Section 12, Rule 41, 1997
Revised Rules of Civil Procedure).

G. Payment of appellate and another lawful fees is a requirement


Within the period for taking an appeal, the appellant shall pay to
the clerk of the court which rendered the judgment or final order appealed
from the full amount of the appellate court docket and other lawful fees.
Proof of payment thereof shall be transmitted to the appellate court
together with the original record or the record on appeal, as the case may
be (Section 5, Rule 40, 1997 Revised Rules of Civil Procedure).

H. Duty of clerk of court of the first level court upon perfection of


appeal

I. Record on appeal; form and contents


Again, the Notice of Appeal shall indicate the parties to the appeal,
specify the judgment or final order or part thereof appealed from, specify
the court to which the appeal is being taken, and state the material dates
showing the timeliness of the appeal (Section 5, Rule 41, 1997 Revised
Rules of Civil Procedure).
The record on appeal shall contain the full names of all the parties
to the proceedings shall be stated in the caption of the record on appeal
and it shall include the judgment or final order from which the appeal is
taken and, in chronological order, copies of only such pleadings, petitions,
motions and all interlocutory orders as are related to the appealed
judgment or final order for the proper understanding of the issue involved,
together with such data as will show that the appeal was perfected on time.
If an issue of fact is to be raised on appeal, the record on appeal shall
include by reference all the evidence, testimonial and documentary, taken
upon the issue involved. The reference shall specify the documentary
evidence by the exhibit numbers or letters by which it was identified when
admitted or offered at the hearing, and the testimonial evidence by the
names of the corresponding witnesses. If the whole testimonial and
documentary evidence in the case is to be included, a statement to that
effect will be sufficient without mentioning the names of the witnesses or
the numbers or letters of exhibits. Every record on appeal exceeding
twenty (20) pages must contain a subject index (Section 6, Rule 41, 1997
Revised Rules of Civil Procedure).
Copies of the notice of appeal and the record on appeal where
required, shall be served on the adverse party (Paragraph 4, Section 3,
Rule 40, 1997 Revised Rules of Civil Procedure).
J. Approval of record on appeal; and when both parties are
appellants
Upon the filing of the record on appeal for approval and if no
objection is filed by the appellee within five (5) days from receipt of a
copy thereof, the trial court may approve it as presented or upon its own
motion or at the instance of the appellee, may direct its amendment by the

inclusion of any omitted matters which are deemed essential to the


determination of the issue of law or fact involved in the appeal. If the trial
court orders the amendment of the record, the appellant, within the time
limited in the order, or such extension thereof as may be granted, or if no
time is fixed by the order within ten (10) days from receipt thereof, shall
redraft the record by including therein, in their proper chronological
sequence, such additional matters as the court may have directed him to
incorporate, and shall thereupon submit the redrafted record for approval,
upon notice to the appellee, in like manner as the original draft (Section 7,
Rule 41, 1997 Revised Rules of Civil Procedure).
Where both parties are appellants, they may file a joint record on
appeal within the time fixed by section 3 of this Rule, or that fixed by the
court (Section 8, Rule 41, 1997 Revised Rules of Civil Procedure).

K. Payment of appellate and another lawful fees is a requirement


Within the period for taking an appeal, the appellant shall pay to
the clerk of the court which rendered the judgment or final order appealed
from, the full amount of the appellate court docket and other lawful fees.
Proof of payment of said fees shall be transmitted to the appellate court
together with the original record or the record on appeal (Section 4, Rule
41, 1997 Revised Rules of Civil Procedure).
The appellate docket and other lawful fees must be paid within the
same period for taking an appeal (Enriquez vs. Enriquez, G.R. No.
139303, August 25, 2005).
The High Court in Enriquez vs. Enriquez, G.R. No. 139303,
August 25, 2005, said:
Prior to the effectivity of the 1997 Rules of Civil
Procedure, as amended, payment of appellate court docket
fee is not a prerequisite for the perfection of an appeal. In
Santos vs. Court of Appeals, this Court held that although
an appeal fee is required to be paid in case of an appeal
taken from the Municipal Trial Court to the Regional Trial
Court, it is not a prerequisite for the perfection of an appeal

under Sections 20 and 23 of the Interim Rules and


Guidelines issued by this Court on January 11, 1983
implementing the Judiciary Reorganization Act of 1981
(B.P. Blg. 129). Under these sections, there are only two
requirements for the perfection of an appeal, to wit: (a) the
filing with the trial court of a notice of appeal within the
reglementary period; and (b) the expiration of the last day
to appeal by any party.
However, the 1997 Rules of Civil Procedure, as
amended, which took effect on July 1, 1997, now require
that appellate docket and other lawful fees must be paid
within the same period for taking an appeal. This is clear
from the opening sentence of Section 4, Rule 41 of the
same Rules that, (W)ithin the period for taking an appeal,
the appellant shall pay to the clerk of the court which
rendered the judgment or final order appealed from, the full
amount of the appellate court docket and other lawful fees.
The use of the word shall underscores the
mandatory character of the Rule. The term shall is a
word of command, and one which has always or which
must be given a compulsory meaning, and it is generally
imperative or mandatory. Petitioners cannot give a
different interpretation to the Rule and insist that payment
of docket fee shall be made only upon their receipt of a
notice from the trial court to pay. For it is a rule in
statutory construction that every part of the statute must be
interpreted with reference to the context, i.e., that every
part of the statute must be interpreted together with the
other parts, and kept subservient to the general intent of the
whole enactment. Indeed, petitioners cannot deviate from
the Rule.
Also under Rule 41 of the same Rules, an appeal to
the Court of Appeals from a case decided by the RTC in the
exercise of the latters original jurisdiction, shall be taken
within fifteen (15) days from the notice of judgment or
final order appealed from. Such appeal is made by filing a

notice thereof with the court that rendered the judgment or


final order and by serving a copy of that notice upon the
adverse party. Furthermore, within this same period,
appellant shall pay to the clerk of court which rendered the
judgment or final order appealed from, the full amount of
the appellate court docket and other lawful fees. The
payment of docket fee within this period is mandatory for
the perfection of appeal. Otherwise, the appellate court
would not be able to act on the subject matter of the action,
and the decision sought to be appealed from becomes final
and executory.
Time and again, this Court has consistently held that
payment of docket fee within the prescribed period is
mandatory for the perfection of an appeal. Without such
payment, the appellate court does not acquire jurisdiction
over the subject matter of the action and the decision
sought to be appealed from becomes final and executory.

The appeal shall be taken within fifteen (15) days from notice of
the judgment or final order appealed from. Where a record on appeal is
required, the appellant shall file a notice of appeal and a record on appeal
within thirty (30) days from notice of the judgment or final order (Section
3, Rule 41, 1997 Rules of Civil Procedure).
The period of appeal shall be interrupted by a timely motion for
new trial or reconsideration. No motion for extension of time to file a
motion for new trial or reconsideration shall be allowed (Paragraph 2,
Section 3, Rule 41, 1997 Rules of Civil Procedure).

L. Failure to pay the appellate docket fee within the reglementary


period bestows on the appellate court a directory power to
dismiss an appeal
The appellants failure to pay the appellate docket fee within the
fifteen-day reglementary period bestows on the appellate court a directory,
not a mandatory, power to dismiss an appeal (Fontanar vs. Bonsubre).

The High Courts explanation in the Badillo consolidated cases,


thus:
We agree with the RTC that, insofar as appeals from
the MTC to the RTC are concerned, the 1997 Rules of Civil
Procedure do not mandate the dismissal of an appeal as a
consequence of the nonpayment of the required fee.
Martinez v. Court of Appeals holds that in such
appeals, the failure to pay the appellate docket fees does
not automatically result in the dismissal of the appeal, the
dismissal being discretionary on the part of the appellate
court. While that case was governed by Sections 20 and
23 of the Interim Rules and Guidelines issued by the Court
on January 11, 1983 to implement the Judiciary
Reorganization Act of 1981 (BP Blg. 129), the present
Rules lead to a similar conclusion.
Under the 1997 Rules of Civil Procedure, parties
perfect an appeal from the judgment of the MTC to the
RTC by filing a notice of appeal within the fifteen day
reglementary period, as provided under Section 4 of Rule
40 and Section 9 of Rule 41:
Rule 40 -SEC. 4. Perfection of appeal; effect thereof. The
perfection of the appeal and the effect thereof shall be
governed by the provisions of section 9, Rule 41.
Rule 41-SEC. 9. Perfection of appeal; effect thereof. - A partys
appeal by notice of appeal is deemed perfected as to him
upon filing of the notice of appeal in due time.
xxx

xxx

xxx

In appeals by notice of appeal, the court loses


jurisdiction over the case upon the perfection of the appeals
filed in due time and the expiration of the time to appeal of
the other party.
Fontanar v. Bonsubre is a case in point. It holds that
in appeals from the MTC to the RTC, failure to pay the
appellate docket fee within the fifteen-day reglementary
period bestows on the appellate court a directory, not a
mandatory, power to dismiss an appeal. The Court
ratiocinated as follows:
x x x [T]his Court restated the importance
and real purpose of the remedy of appeal as an
essential part of our judicial system and advised
the courts to proceed with caution so as not to
deprive a party of a right to appeal with the
instruction that every party-litigant should be
afforded the amplest opportunity for the proper
and just disposition if his cause, freed from the
constraints of technicalities. Rightly so, for the
payment of the appellate docket fee is not a
requirement for the protection of the prevailing
party, and non-compliance therewith within the
time prescribed causes no substantial prejudice
to anyone.
On the other hand, the cases cited by petitioners
involve appeals -- not from the MTC to the RTC -- but
from the RTC to the CA and from the CA to the SC, for
which the payment of appellate fees is indeed mandatory
according to the Rules. We quote Manalili v. Arsenio and
De Leon:
Appeal is not a right, but a mere statutory
privilege. Corollary to this principle is that the
appeal must be exercised strictly in accordance
with provisions set by law. x x x

x x x [T]he payment of the appellate


docket fee is not a mere technicality of law or
procedure. It is an essential requirement,
without which the decision or final order
appealed from would become final and
executory as if no appeal was filed at all.
In the instant cases, when the NHA filed a Notice of
Appeal on February 22, 2000 -- two days before the appeal
period lapsed it perfected its appeal and the MTC thereby
lost its jurisdiction. The MTC therefore acted without
jurisdiction in issuing the May 23, 2000 Order and the May
30, 2000 Writ of Execution.
-0-0-0A notice of appeal was timely filed but the appeal docket fee was
paid after three months when the notice of appeal because of the
messengers inadvertence in securing postal money order. The trial court
approved the notice of appeal basing the order in the phrase in the
interest of substantial justice.
Is the trial court correct?
No. The bare invocation of the phrase in the interest of
substantial justice is not a magic spell that will automatically allow the
court to suspend procedural rules, despite jurisdictional bar. The rule may
be relaxed only in exceptionally meritorious cases. The messengers
alleged inadvertence to secure a postal money order for appellate docket
fees is not a meritorious reason to justify as exception in our jurisprudence
(Ilusorio vs. Ilusorio-Yap,). The finality of a decision is a jurisdictional
event which cannot be made to depend on the convenience of the parties
(Ocampo vs. CA,).

M. Appellate court may motu proprio dismiss the appeal

Prior to the transmittal of the original record or the record on


appeal to the appellate court, the trial court may motu propio or on motion
dismiss the appeal for having been taken out of time (Section 13, Rule 41,
1997 Revised Rules of Civil Procedure).

N. A final judgment or order is one that finally disposes of a case


A final judgment or order is one that finally disposes of a case,
leaving nothing more for the court to do in respect thereto, such as an
adjudication on the merits which, on the basis of the evidence presented at
the trial, declares categorically what the rights and obligations of the
parties are and which party is in the right, or a judgment or order that
dismisses an action on the ground of res judicata or prescription, for
instance (Intramuros Tennis Club, Inc. vs. Philippine Tourism
Authority,).

O. Questions of fact and questions of law distinguished


A question of law arises when there is doubt as to what the law is
on a certain state of facts while there is a question of fact when the doubt
arises as to the truth or falsity of the alleged facts. A question of law may
be resolved by the court without reviewing or evaluating the evidence. No
examination of the probative value of the evidence would be necessary to
resolve a question of law. The opposite is true with respect to questions of
fact, which necessitate a calibration of the evidence (Macababbad vs.
Masirag,).
The nature of the issues to be raised on appeal can be gleaned from
the appellants notice of appeal filed in the trial court and in his or her
brief as appellant in the appellate court. In their Notice of Appeal, the
respondents manifested their intention to appeal the assailed RTC order on
legal grounds and on the basis of the environmental facts. Further, in
their Brief, the petitioners argued that the RTC erred in ruling that their
cause of action had prescribed and that they had slept on their rights. All
these indicate that questions of facts were involved, or were at least raised,
in the respondents appeal with the CA.

P. Issue on prescription may be a question of fact or question of


law
Issue on prescription may either be a question of law or fact; it is a
question of fact when the doubt or difference arises as to the truth or
falsity of an allegation of fact; it is a question of law when there is doubt
or controversy as to what the law is on a given state of facts. The test of
whether a question is one of law or fact is not the appellation given to the
question by the party raising the issue; the test is whether the appellate
court can determine the issue raised without reviewing or evaluating the
evidence. Prescription, evidently, is a question of fact where there is a
need to determine the veracity of factual matters such as the date when the
period to bring the action commenced to run (Crisostomo vs. Garcia, G.R.
No. 164787, January 31, 2006 cited in Macababbad vs. Masirag, G.R.
No. 161237, January 14, 2009).
The High Court in Macababbad said:
Ingjug-Tiro v. Casals instructively tells us too that a
summary or outright dismissal of an action is not proper
where there are factual matters in dispute which require
presentation and appreciation of evidence. In this cited case
whose fact situation is similar to the present case, albeit
with a very slight and minor variation, we considered the
improvident dismissal of a complaint based on prescription
and laches to be improper because the following must still
be proven by the complaining parties:
first, that they were the co-heirs and co-owners
of the inherited property; second, that their coheirs-co-owners sold their hereditary rights
thereto without their knowledge and consent;
third, that forgery, fraud and deceit were
committed in the execution of the Deed of
Extrajudicial Settlement and Confirmation of
Sale since Francisco Ingjug who allegedly
executed the deed in 1967 actually died in 1963,
hence, the thumbprint found in the document
could not be his; fourth, that Eufemio Ingjug

who signed the deed of sale is not the son of


Mamerto Ingjug, and, therefore, not an heir
entitled to participate in the disposition of the
inheritance; fifth, that respondents have not paid
the taxes since the execution of the sale in 1965
until the present date and the land in question is
still declared for taxation purposes in the name
of Mamerto Ingjug, the original registered
owner, as of 1998; sixth, that respondents had
not taken possession of the land subject of the
complaint nor introduced any improvement
thereon; and seventh, that respondents are not
innocent purchasers for value.
As in Ingjug-Tiro, the present case involves factual
issues that require trial on the merits. This situation rules
out a summary dismissal of the complaint.
Proper Mode of Appeal
Since the appeal raised mixed questions of fact and
law, no error can be imputed on the respondents for
invoking the appellate jurisdiction of the CA through an
ordinary appeal. Rule 41, Sec. 2 of the Rules of Court
provides:
Modes of appeal.

(a) Ordinary appeal - The appeal to the Court


of Appeals in cases decided by the
Regional Trial Court in the exercise of its
original jurisdiction shall be taken by
filing a notice of appeal with the court
which rendered the judgment or final order
appealed from and serving a copy thereof
upon the adverse party.
In Murillo v. Consul, this Court had the occasion to
clarify the three (3) modes of appeal from decisions of the

RTC, namely: (1) ordinary appeal or appeal by writ of


error, where judgment was rendered in a civil or criminal
action by the RTC in the exercise of original jurisdiction,
covered by Rule 41; (2) petition for review, where
judgment was rendered by the RTC in the exercise of
appellate jurisdiction, covered by Rule 42; and (3) petition
for review to the Supreme Court under Rule 45 of the Rules
of Court. The first mode of appeal is taken to the CA on
questions of fact or mixed questions of fact and law. The
second mode of appeal is brought to the CA on questions of
fact, of law, or mixed questions of fact and law. The third
mode of appeal is elevated to the Supreme Court only on
questions of law.

Q. Perfected is the appeal by filing a notice of appeal and paying


the requisite docket fees, exception
The general rule is that appeal is perfected by filing a notice of
appeal and paying the requisite docket fees and other lawful fees, but
admits an exception (Tanenglian vs. Lorenzo,).
Tanenglian said:
However, all general rules admit of certain
exceptions. In Mactan Cebu International Airport
Authority v. Mangubat where the docket fees were paid six
days late, we said that where the party showed willingness
to abide by the rules by immediately paying the required
fees and taking into consideration the importance of the
issues raised in the case, the same calls for judicial
leniency, thus:
In all, what emerges from all of the above
is that the rules of procedure in the matter of
paying the docket fees must be followed.
However, there are exceptions to the stringent
requirement as to call for a relaxation of the
application of the rules, such as: (1) most

persuasive and weighty reasons; (2) to relieve a


litigant from an injustice not commensurate with
his failure to comply with the prescribed
procedure; (3) good faith of the defaulting party
by immediately paying within a reasonable time
from the time of the default; (4) the existence of
special or compelling circumstances; (5) the
merits of the case; (6) a cause not entirely
attributable to the fault or negligence of the
party favored by the suspension of the rules; (7)
a lack of any showing that the review sought is
merely frivolous and dilatory; (8) the other party
will not be unjustly prejudiced thereby; (9)
fraud, accident, mistake or excusable negligence
without appellants fault; (10) peculiar legal and
equitable circumstances attendant to each case;
(11) in the name of substantial justice and fair
play; (12) importance of the issues involved;
and (13) exercise of sound discretion by the
judge guided by all the attendant circumstances.
Concomitant to a liberal interpretation of the
rules of procedure should be an effort on the
part of the party invoking liberality to
adequately explain his failure to abide by the
rules. Anyone seeking exemption from the
application of the Rule has the burden of
proving that exceptionally meritorious instances
exist which warrant such departure.
We have not been oblivious to or unmindful of the
extraordinary situations that merit liberal application of the
Rules, allowing us, depending on the circumstances, to set
aside technical infirmities and give due course to the
appeal. In cases where we dispense with the technicalities,
we do not mean to undermine the force and effectivity of
the periods set by law. In those rare cases where we did not
stringently apply the procedural rules, there always existed
a clear need to prevent the commission of a grave injustice.
Our judicial system and the courts have always tried to

maintain a healthy balance between the strict enforcement


of procedural laws and the guarantee that every litigant be
given the full opportunity for the just and proper
disposition of his cause. If the Highest Court of the land
itself relaxes its rules in the interest of substantive justice,
then what more the administrative bodies which exercise
quasi-judicial functions? It must be emphasized that the
goal of courts and quasi-judicial bodies, above else, must
be to render substantial justice to the parties.
In this case, petitioner was only one day late in paying
the appeal fee, and he already stands to lose his titles to the
subject properties. We find this too harsh a consequence for
a days delay. Worthy to note is the fact that petitioner
actually paid the appeal fee; only, he was a day late. That
petitioner immediately paid the requisite appeal fee a day
after the deadline displays his willingness to comply with
the requirement therefor.

---0---

MOTION FOR RECONSIDERATION


(Rule 37)
A. Two-fold nature of motion for reconsideration, purpose and
importance
As a rule, not all orders denying or granting motions for
reconsideration are final orders. It depends upon the kind of incident to
which the motion for reconsideration is filed or applied. If it is filed
arising out of an interlocutory order, let say for instance, against a motion
for bill of particulars, the order granting or denying the same is
interlocutory in character, because the court has still something to do with
the case. It does not finally dispose of the case.
If it is filed against a judgment, the order granting or denying the
same is in the nature of a final order. This is the so-called the two-fold

functional character of a motion for reconsideration the interlocutory


and final, depending upon its usage and applicability.
The importance of knowing whether an order granting or denying a
motion for reconsideration is to determine what will be the next applicable
remedy is available to the aggrieved party.
The aggrieved party is not allowed to file a second motion for
reconsideration of a judgment or final order (Paragraph 2, Section 5, rule
37, 1997 Revised Rules of Civil Procedure).
If an aggrieved party files a second motion for reconsideration of a
judgment or final order, thus allowing the period to appeal to lapsed.
Therefore, he shall loose his right to appeal, and the consequence of
which, he cannot resort to the filing of a petition for certiorari under Rule
65, because it is not a substitute for a lost appeal (Zarate vs. Maybank
Philippines, Inc,).

A. Motion for reconsideration that fails to comply with the rules


on motion not toll the period to file appeal
The motion for reconsideration which is filed without compliance
with the rules on motion is treated as a mere scrap of paper and will not
toll the period to file appeal.
In People vs. Court of Appeals, , the High Court held that a motion
without a notice of hearing is pro forma, a mere scrap of paper that does
not toll the period to appeal.
Said the High Court:
Not only did the defect render the motion for
reconsideration itself unworthy of consideration, it more
crucially failed to toll the period to appeal. A motion
without a notice of hearing is pro forma, a mere scrap of
paper that does not toll the period to appeal, and upon the
expiration of the 15-day period, the questioned order or

decision becomes final and executory. The rationale behind


this rule is plain: unless the movant sets the time and place
of hearing, the court will be unable to determine whether
the adverse party agrees or objects to the motion, and if he
objects, to hear him on his objection, since the rules
themselves do not fix any period within which he may file
his reply or opposition.
---0---

MOTION FOR NEW TRIAL


(Rule 37)
A. Concept, purpose and its allowance
A new trial is not a refuge for the obstinate.17
New trial is a remedy that seeks to temper the severity of a
judgment or prevent the failure of justice. The Rules allows the courts to
grant a new trial when there are errors of law or irregularities prejudicial
to the substantial rights of the accused committed during the trial, or when
there exists newly discovered evidence. (Ybiernas vs. Tanco-Gabaldon,).

B. Grant or denial is discretionary


The grant or denial of a new trial is addressed to the sound
discretion of the court which cannot be interfered with unless a clear abuse
thereof is shown (Ybiernas vs. Tanco-Gabaldon,).

17

Viking Industrial Corporation vs. Court of Appeals, G.R. No. 143794, July 13, 2004
cited in Atlas Consolidated Mining and Development Corporation vs. CIR, G.R. Nos.
141104 & 148763, June 8, 2007.

C. Only on newly discovered evidence, requirements


A new trial may be granted on the ground of newly discovered
evidence, provided that the following must be shown:
(1) that the evidence was discovered after trial;
(2) that such evidence could not have been discovered and
produced at the trial even with the exercise of reasonable
diligence;
(3) that it is material, not merely cumulative, corroborative, or
impeaching; and
(4) that the evidence is of such weight that it would probably
change the judgment if admitted (Custodio vs.
Sandiganbayan,).
If the alleged newly discovered evidence could have been very
well presented during the trial with the exercise of reasonable diligence,
the same cannot be considered newly discovered (Custodio vs.
Sandiganbayan,).
The movant for a new trial must not only act in a timely fashion in
gathering evidence in support of the motion; he must act reasonably and in
good faith as well (Custodio vs. Sandiganbayan,).
Only a final judgment or order may be the subject of a motion for
new trial (Ybiernas vs. Tanco-Gabaldon,).

D. Ignorance, inexperience or incompetence of counsel do not


qualify as a ground for new trial.
Blunders and mistakes in the conduct of the proceedings in the trial
court as a result of the ignorance, inexperience or incompetence of counsel
do not qualify as a ground for new trial. If such were to be admitted as
valid reasons for re-opening cases, there would never be an end to

litigation so long as a new counsel could be employed to allege and show


that the prior counsel had not been sufficiently diligent, experienced or
learned. This will put a premium on the willful and intentional
commission of errors by counsel, with a view to securing new trials in the
event of of conviction, or an adverse decision (Uy vs. First Metro
Integrated Steel Corp,).

E. Remedy of adverse party if motion for new trial is granted by the


trial court.
If a motion for new trial is granted by the trial court, the remedy of
the adverse party if the act of granting the motion is believed to be tainted
with grave abuse of discretion is to file a petition for certiorari under Rule
65 of the 1997 Revised Rules of Civil Procedure, because: first, this is
authorized by Section 1, paragaraph 2 (a) of Rule 41 of the same Rules of
Procedure says that no appeal may be taken from an order denying a
motion for new trial or reconsideration; second, this is because an order
granting a motion for new trial is interlocutory character (See Section 1,
paragraph 2 (c), Rule 41, 1997 Revised Rules of Civil Procedure). It is an
order which does not dispose of the case completely because it leaves
something for the trial court to do (Tan vs. Republic, ), as the original
judgment or final order is vacated (See Section 6, Rule 37, 1997 Revised
Rules of Civil Procedure, Effect of granting of motion for new trial). The
trial court in trial de novo shall re-try the case and to decide it after the retrial; and third, it is only when such interlocutory order was rendered
without or in excess of jurisdiction or with grave abuse of discretion that
certiorari under Rule 65 may be resorted to (Silverio, Jr. vs. CA, G.R. No.
178933, September 16, 2009 citing 1 F. Regalado, Remedial Law
Compendium 540 (8th revised ed.).
If it is believed that the act of the trial court in granting the motion
for new trial is not tainted with grave abuse of discretion, the remedy of
the adverse party is to wait for the new judgment to be rendered after trial
de novo, and if the judgment is not favorable, then file a notice of appeal.
Again, this is because an interlocutory order generally cannot be appealed
separately from the judgment. It is only after a judgment has been
rendered in the case that the ground for the appeal of the interlocutory

order may be included in the appeal of the judgment itself (Silverio, Jr. vs.
CA,).

F. Remedy of the aggrieved party if motion for new trial is denied


by the trial court.
The remedy available is not petition for certiorari under Rule 65,
but appeal under Rule 41 of the 1997 Revised Rules of Civil Procedure.
This is because:
First, Section 9 of Rule 37 of the 1997 Revised Rules of Civil
Procedure is very clear on the matter that an order denying a motion for
new trial is not appealable (the order itself denying a motion for new trial
is not appealable), the remedy being an appeal from the judgment or final
order;
Second, an order denying a motion for new trial is no longer an
interlocutory character;
Third, certiorari is only available if there is no appeal, or any plain,
speedy, and adequate remedy in the ordinary course of law (See Rule 65).
Appeal is available.
Lastly, it is a final order, because it decisively puts to a close, or
disposes of a case or a disputed issue leaving nothing else to be done by
the court in respect thereto (Santo Tomas University Hospital vs. Surla,
G).

Section 9, Rule 37 of the Rules of Court provides:


SEC. 9. Remedy against order denying a motion
for new trial or reconsideration. An order denying a
motion for new trial or reconsideration is not
appealable, the remedy being an appeal from the
judgment or final order.

Section 1, Rule 41 of the Rules of Court provides:


SECTION 1. Subject of appeal. An appeal may
be taken from a judgment or final order that completely
disposes of the case, or of a particular matter therein
when declared by these Rules to be appealable.
No appeal may be taken from:
(a) An order denying a motion for new
trial or reconsideration;
xxx
xxx
In all the above instances where the
judgment or final order is not appealable, the
aggrieved party may file an appropriate special
civil action under Rule 65.
Section 1, Rule 65 of the Rules of Court provides:
SECTION 1. Petition for certiorari. When any
tribunal, board or officer exercising judicial or quasijudical functions has acted with grave abuse of
discretion amouting to lack or excess of jurisdiction,
and there is no appeal, or any plain, speedy, and
adequate remedy in the course of law, a person
aggrieved thereby may file a verified petition in the
proper court, alleging the facts with certainty and
prayuing that judgment be rendered annulling or
modifying the proceedings of such tribunal, board or
officer, and granting such incidental releifs as law
and justice may require.

Section 9 of Rule 39, 1997 Revised Rules of Civil Procedure says


that an order denying a motion for new trial or reconsideration is not
appealable, the remedy being an appeal from the judgment or final order,
while, Section 1, paragaraph 2 (a) of Rule 41 of the same Rules of

Procedure says that no appeal may be taken from an order denying a


motion for new trial or reconsideration.
Santo Tomas University Hospital case had the occasion to
distinguish the concepts of a final judgment or order from an interlocutory
issuance, thus: a final judgment or order is that the former decisively puts
to a close, or disposes of a case or a disputed issue leaving nothing else to
be done by the court in respect thereto. Once that judgment or order is
rendered, the adjudicative task of the court is likewise ended on the
particular matter involved;18 and it is interlocutory if its effects would only
be provisional in character and would still leave substantial proceedings to
be further had by the issuing court in order to put the controversy to rest.19
While it is true that Section 9 of Rule 39, 1997 Revised Rules of
Civil Procedure says an order denying a motion for new trial or
reconsideration is not appealable, however, it also says that the remedy is
appeal from the judgment or final order. Therefore, the aggrieved party
should concentrate to the decision or judgment itself after the motion for
new trial is denied for purposes of filing an appeal (via notice of appeal,
See Section 3, Rule 40 on how to appeal) under Rule 41, and not to
elevate the order of denial (of motion for new trial) via certiorari under
Rule 65. Thus, applying Section 9, Rule 37 in relation to Section 1,
paragaraph 2 (a), Rule 41 and Section 1, Rule 65 of the 1997 Rules of
Civil Procedure it is humbly opined that the available remedy to the
aggrieved party is not certiorari under Rule 65, but appeal.
However, the case of Uy vs. First Metro Integrated Steel Corp, is
different. In this case, it was held that if a motion for new trial is denied,
the aggrieved party may resort to the filing of a petition for certiorari
under Rule 65 of the 1997 Revised Rules of Civil Procedure from the
denial of the motion for new trial. Certiorari was allowed by the High
Court because the denial of the motion for new trial is not on the substance
of the motion but on ground of technicality. In that case the trial court
denied the motion for new trial because the same was filed out of time,
18

Santo Tomas University Hospital vs. Surla, G.R. No. 129718, August 17, 1998 citing
Investments, Inc. vs. Court of Appeals, 147 SCRA 334; Denso (Phils.,) Inc. vs.
Intermediate Appellate Court, 148 SCRA 280.
19
Santo Tomas University Hospital vs. Surla, G.R. No. 129718, August 17, 1998 citing
Bairan vs. Tan Siu Lay, 18 SCRA 1235.

and was found out by the High Court that the same was filed within the
reglementary period. According to the High Court, citing Section 1, Rule
41, that certiorari under Rule 65 is applicable because no appeal may be
taken from an order denying a motion for new trial or
reconsideration.
The High Courts observation in Uy:
A scrutiny of the records discloses that while the
Motion for New Trial was received by the trial court on
April 28, 2003, the date on the Registry Receipt attached to
the Affidavit of Service as well as that stamped on the
envelope which contained the copy of the motion, reveals
that it was filed and served by registered mail on April 21,
2003, a Monday, because April 19, 2003, the last day for
filing the same was a Saturday. Section 1, Rule 22 of the
Rules of Court states in no uncertain terms that if the last
day of the period thus computed falls on a Saturday, a
Sunday, or a legal holiday in the place where the court sits,
the time shall not run until the next working day. Thus, the
motion was actually filed on time it having been filed on
April 21, 2003, the next working day, following the last day
for filing which fell on a Saturday.
Section 9, Rule 37 of the Rules of Court which provides
that the remedy to an order denying a motion for new trial is
to appeal the judgment or final order, must be read in
conjunction with Section 1, Rule 41 which provides that:
SEC. 1. Subject of appeal. An appeal
may be taken from a judgment or final order
that completely disposes of the case, or of a
particular matter therein when declared by these
rules to be appealable.
No appeal may be taken from:
(a)
An order denying a motion for
new trial or reconsideration;

xxxx
In all the above instances where the
judgment or final order is not appeasable, the
aggrieved party may file an appropriate special
civil action under Rule 65.
(Emphasis
supplied)
Thus, the filing by the petitioner of a petition for
certiorari with the Court of Appeals from the denial of the
motion for new trial by the trial court is proper.

G. Failure of counsel to attend the hearings for the reception of


evidence is inexcusable negligence
In Uy vs. First Metro Integrated Steel Corp, it was held that:
xxx xxx xxx, we find that the trial court correctly
denied petitioners motion for new trial.
Section 1, Rule 37 provides that a motion for new
trial may be filed within the period for taking an appeal
based on the following grounds:
(a)
Fraud, accident, mistake or
excusable negligence which ordinary prudence
could not have guarded against and by reason of
which such aggrieved party has probably been
impaired in his rights; or
xxxx
Negligence to be excusable must be one which
ordinary diligence and prudence could not have guarded
against.

In the instant case, we find the negligence of


petitioners counsel in failing to attend the hearings for the
reception of evidence inexcusable.
The trial court
scheduled the hearing for the reception of petitioners
evidence seven times. The initial hearing set on February
28, 2001 was cancelled because petitioner allegedly had
influenza. The hearings scheduled on April 26, 2001 and
May 10, 2001 were cancelled and moved to October 25,
2001 and December 13, 2001. Petitioner was represented
by Atty. Carpio, Jr. as collaborating counsel during the
hearing on October 25, 2001 but no evidence was
presented. Instead, the hearing was cancelled. On
December 13, 2001, Atty. Baares, petitioners new
counsel, appeared but he requested for a resetting. On
February 14, 2002, Atty. Baares moved to postpone the
hearing to February 28, 2002 as previously scheduled. On
February 28, 2002, Atty. Baares arrived late.
Scrutiny of the records disclose that the hearings were
postponed or cancelled without any justification. However,
the trial court accommodated the requests for postponement
or resetting in order to accord petitioner due process.
Under the circumstances, we find petitioners counsels
failure to attend the seven scheduled hearings without
justifiable reason tantamount to inexcusable neglect. As
such, it cannot be a ground for new trial.

H. Affidavit of merit is required in filing a motion for new trial


Also in Uy vs. First Metro Integrated Steel Corp, , it was held
that:
In addition, the Rule requires that motions for new
trial founded on fraud, accident, mistake or excusable
negligence must be accompanied by affidavits of merits,
i.e., affidavits showing the facts (not mere conclusions or
opinions) constituting the valid cause of action or defense

which the movant may prove in case a new trial is granted,


because a new trial would serve no purpose and would just
waste the time of the court as well as the parties if the
complaint is after all groundless or the defense is nil or
ineffective.
Under the Rules, the moving party must show that he
has a meritorious defense. The facts constituting the
movants good and substantial defense, which he may
prove if the petition were granted, must be shown in the
affidavit which should accompany the motion for a new
trial.20 We examined petitioners Affidavit of Merit and
find that it did not contain clear statements of the facts
constituting a good and valid defense which he might prove
if given the chance to introduce evidence. The allegations
that he has a meritorious defense and a good cause are
mere conclusions which did not provide the court with any
basis for determining the nature and merit of the case. An
affidavit of merit should state facts, and not mere opinion
or conclusions of law. Petitioners motion for new trial and
affidavit of merit did not mention the evidence which he
was prevented from introducing, nor did it allege that such
evidence would change the outcome of the case.
---0---

PETITION FOR RELIEF FROM JUDGMENT


(Rule 38)

A. Concept, nature and principles


Section 3, Rule 38 of the 1997 Rules of Civil Procedure provides
that a verified petition for relief must be filed within sixty (60) days after
the petitioner learns of the judgment, final order, or other proceeding to be

set aside and not more than six (6) months after such judgment or final
order has been entered or such proceeding has been taken.
Rule 38 of the Rules of Court only applies when the one deprived
of his right is a party to the case. If the movant is not a party or never been
a party to the case or even summoned to appear in case the remedy of
petition for relief from judgment under Rule 38 of the Rules of Court is
not proper. The word used is against a party (Lagula vs. Casimiro,).
A petition for relief from judgment is an exception to the public
policy of immutability of final judgments (Madarang vs. Morales,). It is
premised on equity; an act of grace and not regarded with favor, and
granted only in exceptional cases (Dirige vs. Biranya,), especially to any
person against whom a decision or order is entered into through fraud,
accident, mistake or excusable negligence (Somoso vs. CA,). It is
dismissible outright if filed beyond the reglementary period (Madarang
vs. Morales,). Good and substantial defense are also included as
requirements sine qua non conditions for its proper allowance (Dirige vs.
Biranya,).
When a party has another remedy available to him, which may be
either a motion for new trial or appeal from an adverse decision of the trial
court, and he was not prevented by fraud, accident, mistake or excusable
negligence from filing such motion or taking such appeal, he cannot avail
himself of this petition. Indeed, relief will not be granted to a party who
seeks avoidance from the effects of the judgment when the loss of the
remedy at law was due to his own negligence; otherwise the petition for
relief can be used to revive the right to appeal which had been lost thru
inexcusable negligence (Tuason vs. Court of Appeals,).
If a party has another adequate remedy available to him, which was
either a motion for new trial or appeal from adverse decisions of the lower
court, and he was not prevented by fraud, accident, mistake or excusable
negligence from filing such motion or taking the appeal he cannot avail
himself of the relief provided in Rule 38 (Somoso vs. CA,).
A party is not entitled to relief under Rule 38, Section 2, of the
Rules of Court if he was not prevented from filing his notice of appeal by
fraud, accident, mistake, or excusable negligence. Such relief will not be

granted to a party who seeks to be relieved from the effects of the


judgment, when the loss of the remedy at law was due to his own
negligence or to a mistaken mode of procedure for that matter; otherwise,
the petition for relief will be tantamount to reviving the right of appeal,
which has already been lost either due to inexcusable negligence or due to
a mistake of procedure by counsel (Fukuzumi vs. Sanritsu Great
International Corporation,).
The petition for relief from judgment must be accompanied with
affidavits showing the fraud, accident, mistake, or excusable negligence
relied upon, and the facts constituting petitioners good and substantial
cause of action or defense (Public Estates Authority vs. Yujuico,). This is
an indispensable statutory requirement under Section 3, Rule 38 of the
1997 Revised Rules of Civil Procedure which must be strictly observed.
Affidavit of merit serves as the jurisdictional basis for a court to entertain
a petition for relief, and when a petition for relief is flawed by such serious
defect, the court with which the petition is filed is not called upon to
entertain the same (Torno vs. IAC,).

B. Fraud as ground must be extrinsic or collateral


Like annulment of judgment, the fraud as ground for filing a
petition for relief from judgment must be extrinsic or collateral. The facts
upon which the extrinsic fraud is based have not been controverted or
resolved in the case where the judgment sought to be annulled was
rendered. For this purpose, fraud is regarded as extrinsic or collateral
where it has prevented a party from having a trial or from presenting all of
his case to the court. Intrinsic fraud takes the form of acts of the party in a
litigation during the trial, such as the use of forged instruments or perjured
testimony which did not affect the presentation of the case, but did prevent
a fair and just determination of the case (Libudan vs. Gil,).

C. Mistake and excusable negligence as a grounds


The mistake contemplated by Rule 38 of the Rules of Court
pertains generally to mistake of fact, not of law, which relates to the case.
The word mistake which grants relief from judgment does not apply and

was never intended to apply to a judicial error which the court might have
committed in the trial. Such error may be corrected by means of an appeal
(Agan vs. Heirs of Sps. Andres Nueva and Diosdada Nueva,), and not by
petition for relief form judgment.
In Madarang vs. Morales, , it was held that a failure of petitioners
former counsel to file the notice of appeal within the reglementary period
is not excusable negligence. The High Court said that that kind of an
argument stereotypes and demeans senior citizens. According to the High
Court, this cannot be done as it asked to assume that a person with
advanced age is prone to incompetence.

D. Reglementary period, when reckoned


The 60-day period must be counted after petitioner learns of the
judgment or final order, and the 6-month period is counted from the
finality of judgment or final order. These are the so-called double period.
These double periods are both inextendible and uninterruptible; is
jurisdictional and should be strictly complied with (Madarang vs.
Morales,).
The 60-day reglementary period shall reckone from the time the
partys counsel receives notice of the decision for notice to counsel of the
decision is notice to the party for purposes of Section 3 of Rule 38
(Mercury Drugs Corporation vs. CA,).
The grace period allowed by the rule within which to file petition for
relief from judgment must be taken as absolutely fixed, inextendible,
never interrupted and cannot be subjected to any condition or contingency.
Because the period fixed is itself devised to meet a condition or
contingency, the equitable remedy is an act of grace, as it were, designed
to give the aggrieved party another and last chance and failure to avail of
such last chance within the grace period fixed is fatal (Martinez vs.
Workmen's Compensation Commission,).

E. Must be filed in the same court that rendered the judgment

It must be filed in the same court that that rendered the judgment
and in the same cause wherein the judgment was rendered. If the court
finds the allegations of the petition to be true, it shall set aside the
judgment and try the principal case upon its merits as if a timely motion
for new trial had been granted therein (Braca vs. Tan,).

F. Petition for relief is only available against a final and executory


judgment
In Gomez vs.Montalban, , it was held that petitioners petition for
relief from judgment under Rule 38 of the Rules of Court was premature
and inappropriate, because when he filed his petition for relief the
judgment on May 28, 2004 against the judgment dated May 4, 2004 which
he received on May 14, 2004, the judgment had not attained finality, and
the 5-day period within which to file a motion for reconsideration or
appeal had not yet lapsed.

G. Petition is deniable when the loss of the remedy at law was due
to his own negligence or mistake
The petition for relief from judgment will not be granted to a party
who seeks to be relieved from the effects of the judgment when the loss of
the remedy at law was due to his own negligence or to a mistaken mode of
procedure for that matter, because if it will be granted the petition for
relief will be tantamount to reviving the right of appeal, a remedy which
has already been lost either due to inexcusable negligence or due to a
mistake of procedure by counsel (Fukuzumi vs. Sanritsu Great
International Corporation,).

H. Cases:
When a petition for relief is filed 61 days from receipt of the notice
of dismissal or one day late, the petition for relief must be ordered denied.
While it is true that the law gives an exception or last chance of a timely
petition for relief from judgment within the reglementary period (within

60 days from knowledge and 6 months from entry of judgment) under


Rule 38 supra, but such grace period must be taken as 'absolutely fixed,
inextendible, never interrupted and cannot be subject to any condition or
contingency. Because the period fixed is itself devised to meet a condition
or contingency (fraud, accident, mistake or excusable neglect), the
equitable remedy is an act of grace, as it were, designed to give the
ag-grieved party another and last chance' "and failure to avail of such last
chance within the grace period fixed by the statute or Rules of Court is
fatal." (Turqueza vs. Hernando,).
A petition for relief which was filed 61 days from receipt of the
notice of dismissal or one day late was held filed out of time (Philippine
Rabbit Bus Lines vs. Judge Arciaga,).
The High Court said in Philippine Rabbit Bus Lines:
It is undisputed that the Petition for Relief in this
case was filed 61 days from receipt of the notice of
dismissal or one day late. In fact, the records show that
counsel for private respondent learned of the dismissal on
the same day, April 29, 1967, when he arrived late for the
hearing so that the Petition for Relief was at least eight (8)
days late. The records further show that counsel for private
respondent did not move for reconsideration of the Order of
dismissal, nor for new trial. Neither did he appeal, thereby
allowing the decision to become final and executory. As a
last resort, he could have availed of the sixty day period
provided for by Rule 38 to file a Petition for Relief from
judgment but again he allowed this opportu-nity to lapse.
Indeed, to him is applicable, the well known maxim that
"equity aids the vigilant, not those who slumber on their
rights." (Henson v. Director of Lands, 55 Phil. 586).
In the case of Turqueza v. Hernando (97 SCRA 488
[1980]) the Supreme Court in disallowing the reopening of
the case which has become final, ruled that there is no
justification in law and in fact, for respondent judge's void
act of ordering the reopening of the case which has become
final and executory.

Thus, the Court held:


"The Court has said time and again that
the doctrine of finality of judgments is grounded
on fundamental considerations of public policy
and sound practice that at the risk of occasional
error the judgments of courts must become final
at some definite date fixed by law. The law
gives an exception or 'last chance of a timely
petition for relief from judgment within the
reglementary period (within 60 days from
knowledge and 6 months from entry of
judgment) under Rule 38 supra, but such grace
period must be taken as 'absolutely fixed,
inextendible, never interrupted and cannot be
subject to any condition or contingency.
Because the period fixed is itself devised to
meet a condition or contingency (fraud,
accident, mistake or excusable neg-lect), the
equitable remedy is an act of grace, as it were,
designed to give the ag-grieved party another
and last chance' "and failure to avail of such last
chance within the grace period fixed by the
statute or Rules of Court is fatal." (Turqueza v.
Hernando, supra).
In expressly reiterating the above-quoted decision the
Supreme Court in Arcilla v. Arcilla (138 SCRA 56 [1985]),
held that the Rule is that, for a petition for relief under Rule
38 to be entertained by the court, the petitioner must
satisfactorily show that he has faithfully and strictly
complied with the provisions of said Rule. Consequently,
it is incumbent upon the petitioner to show that the said
petition was filed within the reglementary period specified
in Sec. 3, of the same, otherwise on this ground alone, the
petition should be dismissed.

I. Petition for relief will not lie when plaintiff filed to personally
notify defendant of court processes
Petition for relief will not lie if plaintiff fails to notify the
defendant of court processes and/or fails to search for the address of
defendant so that he may be properly notified by the Court. Plaintiff is not
duty bound to personally notify the defendant of court processes. It is not
the plaintiffs duty to search for the defendants address so that they may
be properly notified by the Court. This is not the kind of fraud
contemplated by law. Bad faith cannot be presumed from inaction where
there is no duty to act. The grounds not having been clearly established,
petition for relief will not lie (David vs. CA,).

J. Counsels negligence not a ground for petition for relief from


judgment
The excusable negligence referred to in Rule 38 to warrant the
filing of the petition for relief from judgment is negligence of party NOT
negligence of counsel (Somoso vs. CA,). However, where reckless or
gross negligence of counsel deprives the client of due process of law; or
(2) when its application will result in outright deprivation of the clients
liberty or property; or (3) where the interests of justice so require, the
courts must step in and accord relief to a client who suffered thereby
(Sarraga, Sr., vs. Banco Filipino Savings and Mortgage Bank,).

K. Petition for relief from judgment filed out of time shall not be
granted, exceptions
The following are the exceptions:
(1) where reckless or gross negligence of counsel
deprives the client of due process of law;
(2) when its application will result in outright
deprivation of the clients liberty or property; or

(3) where the interests of justice so require (Sarraga,


Sr., vs. Banco Filipino Savings and Mortgage Bank, ).
In Peoples Homesite and Housing Corporation vs.), it was held
that the counsels client was denied of due process when the lawyer failed
to inform his clients of the scheduled hearing of the case and the hearing
proceeded despite the clients absence. In that case, the Court sensed that
there was something fishy with the actuations of the lawyer which
deprived client of their day in court.
In Sarraga vs. Banco Filipino Savings and Mortgage Bank, , the
negligence of newly-hired clerk with lack of work experience in the law
office who merely left the court order in her desk and eventually
misplaced it and failed to bring the matter to the attention of the lawyer
before the lawyer was appointed as Senior State Prosecutor in the
Department of Justice is gross negligence on the part of the lawyer, and
because such negligence will result in the deprivation of the clients
property petition for relief from judgment was allowed. Obtaining these
circumstances, the client should not be made to suffer the consequences.

L. Court can grant the relief different from that what is prayed in
the petition
Obviously, the court can grant relief which is different from that
what is prayed for in the petition. The prayer in a petition for relief from
judgment under Rule 38 is not necessarily the same prayer in the
petitioner's complaint, answer or other basic pleading, because once a
petition for relief is granted and the judgment subject thereof set aside, and
further proceedings are thereafter had, the court in its judgment on the
merits may properly grant the relief sought in the petitioner's basic
pleadings, although different from that stated in his petition for relief
(Cheesman vs. IAC,).

---0---

PETITION FOR RELIEF IS DENIED; REMEDY


A. If denied, petition for certiorari under Rule 65 may be filed
When a petition for relief for judgment is denied, the aggrieved
party may file a petition for certiorari under Rule 65 of the 1997 Revised
Rules of Civil Procedure.
Section 1(b), Rule 41 of the 1997 Revised Rules of Civil Procedure
is clear. It says that no appeal may be taken from an order denying a
petition for relief from judgment, and instead, the aggrieved party may file
an appropriate special civil action under Rule 65.
The normal remedy against a judgment denying relief under Rule
38 is appeal wherein the aggrieved party may assail the judgment on the
merits, upon the ground that it is not supported by the evidence or it is
contrary to law. However, if appeal would not be a remedy adequate under
the circumstances, since it would not promptly relieve the aggrieved party
from the injurious effects of acts of the trial court or tribunal wherein in
the latter for example has authorized execution of the judgment, a resort to
the special civil action of certiorari may exceptionally be allowed.21
---0---

REMEDY OF ADVERSE PARTY


IF PETITION FOR RELIEF IS GRANTED
How if the petition for relief from judgment is granted, what is the
remedy available to the adverse party?
Wait for the judgment, and if the judgment is not favorable, then
file a notice of appeal. This is because an interlocutory order generally
cannot be appealed separately from the judgment. It is only after a
21

PCGG vs. Sandiganbayan, G.R. No. 100733, June 18, 1992 citing Saludes vs. Pajarillo,
78 Phil. 754; Liwanag vs. Castillo, Oct. 20, 1959; Occea vs. Jabson, Oct. 29, 1986,
Silvestre vs. Torres, 57 Phil. 885; Pachoco vs. Tumungday, May 25, 1960, Lopez vs.
Alvendia, Dec. 24, 1964, cited in Moran, op. cit., Vol. 3, p. 177.

judgment has been rendered in the case that the ground for the appeal of
the interlocutory order may be included in the appeal of the judgment
itself (Silverio, Jr. vs. CA, G.R. No. 178933, September 16, 2009).
But if the order granting the petition for relief is believed to be
tainted with graved abuse of discretion, though not covered by Section
1(b), Rule 41 of the 1997 Revised Rules of Civil Procedure as it only
covers a situation wherein a petition for relief from judgment is denied
allowing the aggrieved party to resort to special civil action under Rule 65,
if tainted with graved abuse of discretion, the adverse party may still resort
to filing a petition certiorari under Rule 65 of the 1997 Revised Rules of
Civil Procedure by authority of Section 1(c), Rule 41 of the same Rules,
because the order interlocutory nature; an an order is one which does not
dispose of the case completely but leaves something to be decided upon
(Tan vs. Republic,). It is only when such interlocutory order was rendered
without or in excess of jurisdiction or with grave abuse of discretion that
certiorari under Rule 65 may be resorted to (Silverio, Jr. vs. CA,).

B. Motion for reconsideration is a requirement


A motion for reconsideration of the order denying the petition for
relief from judgment is the plain, speedy, and adequate remedy in the
ordinary course of law. Therefore it is a mandatory requirement to resort
first to the filing of a motion for reconsideration.
In Madarang vs. Morales, , the petition for certiorari assailing the
denial of the petition for relief from judgment was ordered dismissed for
failure of the petitioner to avail first the remedy of filing a motion for
reconsideration.

---0---

ANNULMENT OF JUDGMENT
(Rule 47)
A. Concept, where to file and grounds

Annulment of judgment under Rule 47 of the 1997 Revised Rules


of Civil Procedure is a recourse equitable in character and allowed only in
exceptional cases where the ordinary remedies of new trial, appeal,
petition for relief or other appropriate remedies are no longer available
through no fault of petitioner.22

Rule 47 of the 1997 Revised Rules of Civil Procedure covers


annulment by the Court of Appeals of judgments or final orders and
resolutions in civil actions of Regional Trial Courts for which the ordinary
remedies of new trial, appeal, petition for relief or other appropriate
remedies could no longer be availed of through no fault of the petitioner. It
is a remedy in law independent of the case where the judgment sought to
be annulled is rendered (Macalalag vs. Ombusdman,).
This remedy may not be invoked:
(1) where the party has availed himself of the remedy of
new trial, appeal, petition for relief or other appropriate
remedy and lost therefrom, or
(2) where he has failed to avail himself of those remedies
through his own fault or negligence (Macalalag vs.
Ombusdman,).

All annulment of judgment shall be within the exclusive original


jurisdiction of the Court of Appeals.
The grounds are extrinsic fraud and lack of jurisdiction (Paulino
vs. CA,). Through jurisprudential pronouncements, denial of due process
is included as an additional ground (Biaco vs. Philippine Countryside
Rural Bank, G).
22

City Government of Tagaytay vs. Judge Guerrero, G.R. Nos. 140743 & 140745 and
Justice Ameurfina Melencio-Herrera vs. Judge Guerrero, G.R. Nos. 141451-52,
September 17, 2009.

B. Fraud as ground, extrinsic or collateral not intrinsic


Fraud has been regarded as extrinsic or collateral. It is only
extrinsic or collateral fraud NOT intrinsic fraud that can serve as a basis
for annulment of judgment (Macabingkil vs. People's Homesite and
Housing Corporation,). To become a basis for annulment of judgment,
fraud has to be extrinsic or actual (Ancheta vs. Guersey-Dalaygon,), and
refers to acts outside the trial (Ybaez vs. CA,).
Fraud is extrinsic or collateral where it is on the effect of which
prevent a party from having a trial, or real contest, or from presenting all
of his case to the court, or where it operates upon matters pertaining, not to
the judgment itself, but to the manner in which it was procured so that
there is not a fair submission of the controversy (Macabingkil vs. People's
Homesite and Housing Corporation,). It takes the form of acts of a party
in a litigation during the trial, such as the use of forged instruments or
perjured testimony, which did not affect the presentation of the case, but
did prevent a fair and just determination of the case (Libudan vs. Palma,).
Extrinsic fraud refers to any fraudulent act of the prevailing party
in the litigation which is committed outside of the trial of the case,
whereby the defeated party has been prevented from exhibiting fully his
side of the case, by fraud or deception practiced on him by his opponent
(Macabingkil vs. People's), or acts of a party at a trial which prevented a
fair and just determination of the case and which could have been litigated
and determined at the trial or adjudication of the case. Examples of
intrinsic fraud are falsification and false testimony (Ybaez vs. CA,).
According to Ancheta vs. Guersey-Dalaygon, , fraud takes on
different shapes and faces. In that case, the High Court stated that man in
his ingenuity and fertile imagination will always contrive new schemes to
fool the unwary.
Said the High Court:
There is extrinsic fraud within the meaning of Sec. 9
par. (2), of B.P. Blg. 129, where it is one the effect of

which prevents a party from hearing a trial, or real contest,


or from presenting all of his case to the court, or where it
operates upon matters, not pertaining to the judgment itself,
but to the manner in which it was procured so that there is
not a fair submission of the controversy. In other words,
extrinsic fraud refers to any fraudulent act of the prevailing
party in the litigation which is committed outside of the
trial of the case, whereby the defeated party has been
prevented from exhibiting fully his side of the case by fraud
or deception practiced on him by his opponent. Fraud is
extrinsic where the unsuccessful party has been prevented
from exhibiting fully his case, by fraud or deception
practiced on him by his opponent, as by keeping him away
from court, a false promise of a compromise; or where the
defendant never had any knowledge of the suit, being kept
in ignorance by the acts of the plaintiff; or where an
attorney fraudulently or without authority connives at his
defeat; these and similar cases which show that there has
never been a real contest in the trial or hearing of the case
are reasons for which a new suit may be sustained to set
aside and annul the former judgment and open the case for
a new and fair hearing.

C. Extrinsic fraud already availed of as ground in petition for


relief or new trial, barred.
If extrinsic fraud was availed of or could have been availed of in a
motion for new trial or petition for relief, the extrinsic fraud can no longer
be availed of as a ground for annulment of judgment. It is considered
barred.
In Tolentino vs. Judge Leviste, , the High Court said that it is
provided in Section 2 of Rule 47 that extrinsic fraud shall not be a valid
ground if it was availed of, or could have been availed of, in a motion for
new trial or petition for relief. In other words, it is effectively barred if it
could have been raised as a ground in an available remedial measure.

D. Extrinsic fraud, prescriptive period


According to Article 1391 of the Civil Code of the Philippines, in
case of fraud an action for annulment must be brought within four (4)
years from the time of the discovery of the same. It commences to run
from the discovery of the fraud or fraudulent act or acts. Therefore,
annulment of judgment based on extrinsic fraud under Rule 47 of the 1997
Revised Rules of Civil Procedure must be brought within four (4) years
from the discovery of the fraud (Ancheta vs. Guersey-Dalaygon, G).

E. Intrinsic fraud, concept, examples and cases


Intrinsic fraud is not sufficient to attack a judgment (Yatco vs.
Sumagui,).

F. Introduction of false affidavit purely intrinsic


The use of a false affidavit of loss is intrinsic because it is similar
to the use of it during trial of forged instruments or perjured testimony. It
does not constitute extrinsic fraud to warrant the invalidation of a final
judgment (Demetriou vs. CA, The use of a forged instrument constituted
only intrinsic fraud for while perhaps it prevented a fair and just
determination of a case, the use of such instrument or testimony did not
prevent the adverse party from presenting his case fully and fairly
(Palanca vs. Republic,).

G. Lack of jurisdiction as a ground for annulment


Lack of jurisdiction as a ground for annulment of judgment refers
to either lack of jurisdiction over the person of the defending party or over
the subject matter of the claim (Paulino vs. CA, June 04, 2014).
A petition for annulment of judgment must be based on lack of
jurisdiction. Meaning, absence of or no jurisdiction; and that the court
should not have taken cognizance of the petition because the law does not

vest it with jurisdiction over the subject matter. The petitioner must show
not merely an abuse of jurisdictional discretion but an absolute lack of
jurisdiction (Durisol Philippines, Inc. vs. CA,).
If a complaint is filed and there is absence, or lack, of jurisdiction,
and the trial court dismisses the case outright, the dismissal cannot be a
subject of annulment of judgment. Appeal or certiorari under Rule 65, as
the case may be, is the proper remedy, or to re-file the case to the
appropriate court having jurisdiction over the case or subject matter.
The High Court in Paulino vs. CA, , gave emphasis that if the case
is dismissed on ground of lack of jurisdiction, the dismissal cannot be a
subject of an annulment of judgment. And no need to allege in the petition
that the ordinary remedy of new trial or reconsideration of the final order
or judgment or appeal therefrom is no longer available through no fault of
his own, precisely because the judgment rendered or the final order issued
by the RTC without jurisdiction is null and void. The High Court said in
Paulino:
The Court finds the petitions devoid of merit.
Under Section 2 of Rule 47, the only grounds for
annulment of judgment are extrinsic fraud and lack of
jurisdiction. Lack of jurisdiction as a ground for annulment
of judgment refers to either lack of jurisdiction over the
person of the defending party or over the subject matter of
the claim. In case of absence, or lack, of jurisdiction, a
court should not take cognizance of the case.
In these cases, the petition for annulment was based
on lack of jurisdiction over the subject matter. The rule is
that where there is want of jurisdiction over a subject
matter, the judgment is rendered null and void. A void
judgment is in legal effect no judgment, by which no rights
are divested, from which no right can be obtained, which
neither binds nor bars any one, and under which all acts
performed and all claims flowing out are void. It is not a
decision in contemplation of law and, hence, it can never
become executory. It also follows that such a void

judgment cannot constitute a bar to another case by reason


of res judicata.
Accordingly, the Court agrees with the CA that LRA
was not estopped from assailing the July 20, 2011 RTC
Decision because it never attained finality for being null
and void, having been rendered by a court without
jurisdiction over the reconstitution proceedings.
As early as the case of Strait Times, Inc. v. CA, the
Court has held that when the owners duplicate certificate
of title has not been lost, but is, in fact, in the possession of
another person, then the reconstituted certificate is void,
because the court that rendered the decision had no
jurisdiction. Reconstitution can be validly made only in
case of loss of the original certificate. This rule was
reiterated in the cases of Villamayor v. Arante, Rexlon
Realty Group, Inc. v. Court of Appeals, Eastworld Motor
Industries Corporation v. Skunac Corporation, Rodriguez v.
Lim, Villanueva v. Viloria, and Camitan v. Fidelity
Investment Corporation. Thus, with evidence that the
original copy of the TCT was not lost during the
conflagration that hit the Quezon City Hall and that the
owners duplicate copy of the title was actually in the
possession of another, the RTC decision was null and void
for lack of jurisdiction.
For the aforecited reason, the Court agrees that the
public respondent correctly availed of the remedy of
petition for annulment of judgment under Rule 47 without
need of exhausting other ordinary remedies of new trial,
appeal, petition for relief, or other appropriate remedies
because the RTC judgment was null and void.
Indeed, where a petition for annulment of a judgment
or a final order of the RTC filed under Rule 47 of the Rules
of Court is grounded on lack of jurisdiction over the person
of the respondent or over the nature or subject of the action,
the petitioner need not allege in the petition that the

ordinary remedy of new trial or reconsideration of the final


order or judgment or appeal therefrom is no longer
available through no fault of his own, precisely because the
judgment rendered or the final order issued by the RTC
without jurisdiction is null and void and may be assailed
any time either collaterally or in a direct action or by
resisting such judgment or final order in any action or
proceeding whenever it is invoked, unless barred by
laches.
In Ybaez vs. CA, The High Court dismissed the annulment of
judgment case, because what was filed with the CA was an annulment of
judgment and not petition for review under Rule 42. So, when the RTC
ruled on the issue of validity of the substituted service of summons over
the persons of the petitioners in the MTC level, it exercised its appellate
jurisdiction, thus, not a ground for annulment of judgment.

H. Denial of due process, as a ground


Section 2 of the said Rule provides that the annulment may be
based only on the grounds of extrinsic fraud and lack of jurisdiction,
although jurisprudence recognizes denial of due process as an additional
ground.

I. Cases
Demetriou case was permitted by the considered by the High
Court as annulment of judgment case but not on extrinsic fraud but on
lack of jurisdiction.
In Demetriou vs. CA, , petitioners and private respondent became
co-owners of the realty brought about by the execution of deeds of sale
executed by respondents co-owners in favor of the petitioner which at
that time the property was then under lease agreement. That time copy of
the title was delivered to petitioners, but because of the existing lease
agreement, petitioners waited for the lease agreement to expire. At the
registry of deeds, upon termination of the lease agreement, petitioners

tried to facilitate the transfer of the title over portions pertaining to them
but they failed because the duplicate owners copy of the title had already
been cancelled by virtue of a court order upon petitioners co-owners
petition for issuance of the second owners duplicate title by reason of its
loss as it was destroyed by the typhoon. Because the use of a false
affidavit of loss does not constitute extrinsic fraud to warrant the
invalidation of a final judgment, the CA dismissed the action. However,
the High Court reversed the Court of Appeals ruling and remanded the
case for further proceedings. The High Court said:
On the basis of these allegations the appellate court
ruled that the fraud alleged was, if at all, only intrinsic and
not extrinsic in character:
An action to annul a final judgment on the
ground of fraud will lie only if the fraud is
extrinsic or collateral in character. Extrinsic
fraud refers to any fraudulent act of the
prevailing party in the litigation which is
committed outside of the trial of the case,
whereby the defeated party has been prevented
from exhibiting fully his side of the case, by
fraud or deception practiced on him by his
opponent (Macabingkil vs. People's Homesite
and Housing Corporation, 72 SCRA 326 cited in
Canlas vs. CA, 164 SCRA 160). On the other
hand, intrinsic fraud takes the form of "acts of a
party in a litigation during the trial such as the
use of forged or false document or perjured
testimony, which did not affect the presentation
of the case, but did prevent a fair and just
determination of the case" (Libudan vs. Gil, 45
SCRA 17). In the present petition, the allegation
of fraud involves admission by the respondent
court of an alleged false affidavit of loss, which
alleged fraud is intrinsic in character. Thus, as
the alleged fraud committed by the private
respondent is not extrinsic in character, the
instant petition for annulment of the said

December 1, 1990 order of the lower court


should be dismissed.
The appellate court is certainly right in holding that
the use of a false affidavit of loss does not constitute
extrinsic fraud to warrant the invalidation of a final
judgment. The use of the alleged false affidavit of loss by
private respondent is similar to the use during trial of
forged instruments or perjured testimony. In the leading
case of Palanca v. Republic, it was held that the use of a
forged instrument constituted only intrinsic fraud for while
perhaps it prevented a fair and just determination of a case,
the use of such instrument or testimony did not prevent the
adverse party from presenting his case fully and fairly. In
the case at bar, petitioners were not really kept out of the
proceedings because of the fraudulent acts of the private
respondent. They could have rebutted or opposed the use
of the affidavit and shown its falsity since they were
theoretically parties in the case to whom notice had been
duly given.
But a judgment otherwise final may be annulled not
only on the ground of extrinsic fraud but also because of
lack of jurisdiction of the court which rendered it. In Serra
Sera v. Court of Appeals, on facts analogous to those
involved in this case, this Court already held that if a
certificate of title has not been lost but is in fact in the
possession of another person, the reconstituted title is void
and the court rendering the decision has not acquired
jurisdiction. Consequently the decision may be attacked
any time. Indeed, Rep. Act No. 26, 18 provides that "in
case a certificate of title, considered lost or destroyed be
found or recovered, the same shall prevail over the
reconstituted certificate of title." It was, therefore, error for
the Court of Appeals to dismiss the petition for annulment
of judgment of the petitioners.

J. Withdrawal of the appeal will not give rise to the filing of


annulment of judgment
Resorting to the filing of a petition for certiorari after the trial court
had issued a writ of execution pending appeals and later on withdrew the
appeal, the withdrawal of the appeal will not give rise to the filing of
annulment of judgment (Philippine Tourism Authority vs. Philippine
Golf Development & Equipment,).

K. Taxpayer is a proper party in annulment of judgment


In Remulla vs. Maliksi, , the High Court ruled that if the petition
for annulment of judgment is filed by a taxpayer who is representing the
interests of the province itself, the petition for annulment of judgment
should not be dismissed on ground of improper party. The petitioner being
a taxpayer who will be either benefited or injured by the execution of the
compromise judgment is a real-party-in-interest.

L. Dismissal on ground of specific reason, duty of the court


If the dismissal is for specific reasons the dismissal shall be clearly
set out so that the High Court will not be at sea if the resolution will be
later on assailed, and to make a definitive determination as to whether the
CA committed a reversible error in dismissing the petition. The dismissal
must contain the details whether the dismissal is on ground of extrinsic
fraud or lack of jurisdiction, procedural or substantial (Castigador vs.
Nicolas,).

M. Remedy if Court of Appeals denies or grants annulment of


judgment
If the Court of Appeals denies/dismisses or grants the petition for
annulment of judgment, the remedy available to the aggrieved party is to
file a Petition for Review on Certiorari under Rule 45 of the Revised Rules
of Civil Procedure with High Court (Supreme Court) within fifteen (15)

days from notice of the judgment or final order or resolution appealed


from, or of the denial of the petitioner's motion for new trial or
reconsideration filed in due time after notice of the judgment (Sections 1 23
and 2 24, Rule 45, 1997 Revised Rules of Civil Procedure).
---0---

---0---

SPECIAL CIVIL ACTIONS


Special civil actions are those actions governed by Rules 62,
63, 64, 65, 66, 67, 68 69, 70 and 71. These are the interpleader
(Rules 62), declaratory relief and similar remedies (Rules 63), review
of judgments and final orders of the Commission on Elections and
the Commission on Audit (Rules 64), certiorari, prohibition and
mandamus (Rules 65); quo warranto (Rules 66); expropriation
(Rules 67); foreclosure of real estate mortgage (Rules 68); partition
(Rules 69); forcible entry and unlawful detainer (Rules 70); and
contempt (Rules 71).

1. CERTIORARI (RULE 65)


23

Section 1. Filing of petition with Supreme Court. A party desiring to appeal by


certiorari from a judgment or final order or resolution of the Court of Appeals, the
Sandiganbayan, the Regional Trial Court or other courts whenever authorized by law,
may file with the Supreme Court a verified petition for review on certiorari. The petition
shall raise only questions of law which must be distinctly set forth.
24

Section 2. Time for filing; extension. The petition shall be filed within fifteen (15)
days from notice of the judgment or final order or resolution appealed from, or of the
denial of the petitioner's motion for new trial or reconsideration filed in due time after
notice of the judgment. On motion duly filed and served, with full payment of the docket
and other lawful fees and the deposit for costs before the expiration of the reglementary
period, the Supreme Court may for justifiable reasons grant an extension of thirty (30)
days only within which to file the petition.

A. The Rule
Certiorari under Rule 65 is a remedy designed for the correction of
errors of jurisdiction and not errors of judgment. An error of judgment is
not correctible by certiorari, and as long as the public respondent acted
with jurisdiction, any error committed by him or it in the exercise thereof
will amount to nothing more than an error of judgment which may be
reviewed or corrected only by appeal (Lim vs. Judge Vianzon,).
The acceptance of a petition for certiorari, as well as the grant of
due course thereto is, in general, addressed to the sound discretion of the
court. It must be stressed that certiorari, being an extraordinary remedy,
the party who seeks to avail of the same must strictly observe the rules laid
down by the law and non-observance thereof may not be brushed aside as
mere technicality (Garcia, Jr. vs. CA,).
The rule is that when any tribunal, board or officer exercising
judicial or quasi-judicial functions has acted without or in excess its or his
jurisdiction, or with grave abuse of discretion amounting to lack or excess
of jurisdiction, and there is no appeal, or any plain, speedy, and adequate
remedy in the ordinary course of law, a person aggrieved thereby may file
a verified petition in the proper court, alleging the facts with certainty and
praying that judgment be rendered annulling or modifying the proceedings
of such tribunal, board or officer, and granting such incidental reliefs as
law and justice may require (Section 1, Rule 65, 1997 Revised Rules of
Civil Procedure).
The petition shall be accompanied by a certified true copy of the
judgment, order or resolution subject thereof, copies of all pleadings and
documents relevant and pertinent thereto, and a sworn certification of nonforum shopping as provided in the third paragraph of section 3, Rule 46
(Paragraph 2, Section 1, Rule 65, 1997 Revised Rules of Civil
Procedure).
In Lim vs. Judge Vianzon, , the held that the petition is
procedurally flawed as it is not accompanied by copies of relevant
pleadings mandated by the second paragraph of Section 1, Rule 65 of the
1997 Rules of Civil Procedure.

The High Court said in In Lim:


Moreover, the instant petition is procedurally flawed
as it is not accompanied by copies of relevant pleadings
mandated by the second paragraph of Section 1, Rule 65 of
the 1997 Rules of Civil Procedure. Said provision reads as
follows:
SECTION 1. Petition for certiorari.
When any tribunal, board or officer exercising
judicial or quasi-judicial functions has acted
without or in excess of its or his jurisdiction, or
with grave abuse of discretion amounting to
lack or excess of jurisdiction, and there is no
appeal, nor any plain, speedy, and adequate
remedy in the ordinary course of law, a person
aggrieved thereby may file a verified petition in
the proper court, alleging the facts with certainty
and praying that judgment be rendered annulling
or modifying the proceedings of such tribunal,
board or officer, and granting such incidental
reliefs as law and justice may require.
The petition shall be accompanied by a
certified true copy of the judgment, order or
resolution subject thereof, copies of all
pleadings and documents relevant and
pertinent thereto, and a sworn certification of
no-forum shopping as provided on the third
paragraph of Section 3, Rule 46. (Emphasis
supplied.)
Specifically, as pointed out by respondents, the
instant petition is not accompanied by copies of the Motion
to Dismiss and Motion for Reconsideration that petitioners
filed with the trial court. These are documents important for
the Courts appraisal, evaluation and judicious disposition
of the case. Failing to fully apprise the Court of the relevant
details of the case, we find this egregious error a sufficient

cause for the dismissal of the instant petition. As held in


Santiago, Jr. v Bautista, to wit:
x x x the lower courts holding that
appellants failure to accompany his petition
with a copy of the judgment or order subject
thereof together with copies of all pleadings and
documents relevant and pertinent thereto is
fatal to his cause is supported not only by the
provision of that Rule but by precedents as well.
A party who seeks to avail of the extraordinary remedy
of certiorari must observe the rules laid down by law, and
non-observance of the said rules may not be brushed aside
as mere technicality.

B. Petition must be accompanied with clearly legible


duplicate original or certified copy of the judgment, order,
resolution, or ruling subject
What really required is that a petition shall be accompanied with
clearly legible duplicate original or certified copy of the judgment, order,
resolution, or ruling subject of the petition. Other relevant documents and
pleadings attached to it may be mere machine copies thereof is sufficient
(Garcia, Jr. vs. CA,).

The High Court in Garcia, Jr:


We dismiss the Petition.
The acceptance of a petition for for certiorari, as well
as the grant of due course thereto is, in general, addressed
to the sound discretion of the court. It must be stressed that

certiorari, being an extraordinary remedy, the party who


seeks to avail of the same must strictly observe the rules
laid down by the law and non-observance thereof may not
be brushed aside as mere technicality.
In the matter of the requirement that a petition for
certiorari be accompanied by a certified true copy of the
judgment, order or resolution subject thereof, , Section 1,
Rule 65 of the 1997 Rules of Civil Procedure, as amended,
provides:
SECTION 1. Petition for certiorari.xxxx
The petition shall be accompanied by a
certified true copy of the judgment, order or
resolution subject thereof, copies of all
pleadings and documents relevant and
pertinent thereto, and a sworn certification of
non-forum shopping as provided in the third
paragraph of Section 3, Rule 46.
Significantly, Section 3, Rule 46 of the same Rules,
provides:
SECTION 3. Contents and filing of
petition; effect of non-compliance with
requirements.
xxxx
It shall be filed in seven (7) clearly
legible copies together with proof of service
thereof on the respondent with the original
copy intended for the court indicated as such
by the petitioner, and shall be accompanied by
a clearly legible duplicate original or
certified true copy of the judgment, order,

resolution, or ruling subject thereof, such


material portions of the record as are referred
to therein, and other document or pertinent
thereto.
The
certification
shall
be
accomplished by the proper clerk of court or
by his duly authorized representative, or by
the proper officer of the court, tribunal,
agency or office involved or by his duly
authorized representative. The other requisite
number of copies of the petition shall be
accompanied by clearly legible plain copies of
all documents attached to the original.
xxxx
The failure of the petitioner to comply
with any of the foregoing requirements shall
be sufficient ground for the dismissal of the
petition.
It is true that Section 3 of Rule 46 does not require
that all supporting papers and documents accompanying a
petition be duplicate originals or certified true copies.
However, it explicitly directs that all cases originally filed
in the Court of Appeals shall be accompanied by a clearly
legible duplicate original or certified true copy of the
judgment, order, resolution or ruling subject thereof.
Similarly, under Rule 65, which covers certiorari,
prohibition and mandamus, petitions need to be
accompanied only by duplicate originals or certified true
copies of the questioned judgment, order or resolution.
Other releval documents and pleadings attached to it may
be mere machine copies thereof. In the case at bar,
petitioners failed to attach duplicate originals or certified
true copies of the assailed Orders of the RTC, dated 22
February 2005 and 7 April 2005. What they affixed were
machine or Xerox copies of the same. Plainly put,
petitioners contravened the obvious rudiments of the rules.

In Circular No. 3-96, we made the following


clarifications and supplemental rules on what is a duplicate
original or certified true copy:
1. The "duplicate original copy shall" be understood
to be that copy of the decision, judgment, resolution or
order which is intended for and furnished to a party in the
case or proceeding in the court or adjudicative body which
rendered and issued the same. The "certified true copy"
thereof shall be such other copy furnished to a party at his
instance or in his behalf, duly authenticated by the
authorized officers or representatives of the issuing entity
as hereinbefore specified.
2. The duplicate original copy must be duly signed or
initialed by the authorities or the corresponding officer or
representative of the issuing entity, or shall at least bear the
dry seal thereof or any other official indication of the
authenticity and completeness of such copy. For this
purpose, all courts, offices or agencies furnishing such
copies which may be used in accordance with Paragraph
(3) of Revised Circular No. 1-88 shall make arrangements
for and designate the personnel who shall be charged with
the implementation of this requirement.
3. The certified true copy must further comply with
all the regulations therefor of the issuing entity and it is the
authenticated original of such certified true copy, and not a
mere Xerox copy thereof, which shall be utilized as an
annex to the petition or other initiatory pleading.
4. Regardless of whether a duplicate original copy or
a certified true copy of the adjudicatory document is
annexed to the petition or initiatory pleading, the same
must be exact and complete copy of the original and all the
pages thereof must be clearly legible and printed on white
bond or equivalent paper of good quality with the same
dimensions as the original copy. Either of the aforesaid
copies shall be annexed to the original copy of the petition

or initiatory pleading filed in court, while plain copies


thereof may be attached to the other copies of the pleading.
5. It shall be the duty and responsibility of the party
using documents required by Paragraph (3) of Circular No.
1-88 to verify and ensure compliance with all the
requirements thereof as detailed in the proceeding
paragraphs. Failure to do so shall result in the rejection of
such annexes and the dismissal of the case. Subsequent
compliance shall not warrant any reconsideration unless the
court is fully satisfied that the noncompliance was not in
any way attributable to the party despite due diligence on
his part, and that there are highly justifiable and compelling
reasons for the court to make such other disposition as it
may deem just and equitable.
Based on the foregoing, it is incontrovertible that a
certified true copy is not a mere Xerox copy. Further, it is
imperative that the duplicate original copyu required by the
rules must be duly signed or initialed by the authorities or
the corresponding officer or representative of the issuing
entity, or shall at least bear the dry seal thereof or any other
official indication of the authenticity and completeness of
such copy. Petitioners Xerox copies are wanting in this
respect.
Petitioners seek a liberal application of the procedural
rules. For their failure to attach certified true copies of the
assailed orders of the RTC, petitioners place the blame on
the appellate court. Petitioners brazenly suggest that what
the Court of Appeals should have done was to issue an
Order directing them to comply with the rule on attaching
certified true copies, instead of dismissing the case on its
face. We do not see reason to grant liberality in the
application of the rules. It must be emphasized that the
liberality in the interpretation and application of the rules
applies only in proper cases and under justifiable causes
and circumstances. While it is true that litigation is not a
game of technicalities, it is equally tue that every case must

be prosecuted in accordance with the prescribed procedure


to insure an orderly and speedy administration of justice.
Only strong considerations of equity, which are wanting in
this case, will lead us to allow an exception to the
procedural rule in the interest of substantial justice. To
further suggest petitioners impervious attitude towards
rules, they even failed to attach certified true copies or
duplicate original copies of the assailed Orders in their
Motion for Reconsideration filed with the Court of
Appeals. Concomitant to a liberal application of the rules
of procedure should be an effort on the party invoking
liberality to at least explain its failure to comply with the
rules. Circular No. 3-96 is also unequivocal that it shall be
the duty and responsibility of the party to verify and ensure
compliance with all the requirements detailed therein. In
fact, failure to do so shall result in the rejection of such
annexes and the dismissal of the case.
Take note the requirement provided in A.M. No. 10-3-7-SC
[Rules on e-Filing] and A.M. No. 10-3-7-SC [Rule for the Efficient
Use of Paper] dated September 10, 2013.

A. The assessment of treble costs provided in Section 8 of Rule 65,


1997 Revised Rules of Civil Procedure, as amended by A.M.
No. 07-7-12-SC is not automatic or mandatory
After the comment or other pleadings required by the court are
filed, or the time for the filing thereof has expired, the court may hear the
case or require the parties to submit memoranda. If, after such hearing or
filing of memoranda or upon the expiration of the period for filing, the
court finds that the allegations of the petition are true, it shall render
judgment for such relief to which the petitioner is entitled. However, the
court may dismiss the petition if it finds the same patently without merit or
prosecuted manifestly for delay, or if the questions raised therein are too
unsubstantial to require consideration. In such event, the court may award
in favor of the respondent treble costs solidarily against the petitioner and
counsel, in addition to subjecting counsel to administrative sanctions

under Rule 139 and 139-B of the Rules of Court (First paragraph,
Section 8 of Rule 65, as amended by A.M. No. 07-7-12-SC).
The Court may impose motu proprio, based on res ipso loquitur,
other disciplinary sanctions or measures on erring lawyers for patently
dilatory and unmeritorious petitions for certiorari (Second paragraph,
Section 8 of Rule 65, as amended by A.M. No. 07-7-12-SC).
The use of the word "may" in the last sentence of the second
paragraph or Section 8, Rule 65, indicates that the assessment of treble
costs is not automatic or mandatory. It merely gives the court the
discretion and latitude to impose further sanctions where a petition is
dismissed for being "patently without merit," "prosecuted manifestly for
delay," or upon finding that the questions raised in the petition for
certiorari were "too to require consideration" (City of Davao vs. CA, G.R.
No. 200538, Augsut 13, 2014).
The court is afforded judicial discretion in imposing treble costs,
there remains a need to show that it is sound and with basis - that is
"taking all the pertinent circumstances into due consicleration.'' (Diaz vs.
People, 180677,).

B. When is treble cost imposable or awarded?


The parties and their counsels resort to deplorable dilatory tactics
to frustrate the fruition of justice (City of Davao vs. CA,).
Examples:
1. When the losing litigant repeatedly frustrated the execution of a
final and executory decision (Central Surety and Insurance Company vs.
Planters Product, Inc, 546 Phil. 479 cited in City of Davao vs. CA, G.R.
No. 200538, August 13, 2014).
In Central Surety and Insurance Company vs. Planters Product,
Inc,), the execution was delayed for more than five years because of his
dilatory tactics. When the winning party sought the execution by motion

beyond the period, he still opposed it despite the fact that the period was
suspended because of reasons attributable to him.
2. Treble costs were awarded because of the deplorable course
resorted to by the losing litigants in the hope of evading manifest
obligations. The Court stated that it viewed with disfavor the unjustified
delay in the enforcen1ent or the final decision and orders in the said case.
Once a judgment becomes tinal and executory, the prevailing party should
not be denied the fruits of his victory by some subterfuge devised by the
losing party. Unjustified delay in the enforcement of a judgment sets at
naught the role of courts in disposing justiciable controversies with finality
(Spouses Aguilar vs. The Manila Banking Corporation,).
3. Treble costs were imposed because the parties took the law into
their own hands or resorted to a wrong remedy (Uypitching vs.
Quiamco,).
In Uypitching vs. Quiamco, , instead of bringing the proper civil
action necessary to acquire legal possession of a motorcycle, the petitioner
took the law into his own hands and seized it without a search warrant or
court order. Worse, in the course or the illegal seizure or the motorcycle,
the petitioner even mouthed a slanderous statement. By doing so, he
transgressed the proper norms of human relations.
4. In Solen vs. A-1 Investors., the party claimed that he was denied
due process when the court adjudged a case against him even if he was not
served the summons. He then resorted to filing a complaint for injunction
to stop the execution of the final judgment. The Court said that he should
have resorted to an action for annulment under Rule 47 and so awarded
treble costs against him.
In City of Davao vs. CA, G, the imposition of treble costs was
striken off by the High Court for failure of the Court of Appeals to explain
and/or to give any reason for such imposition.

C. Timeliness of the petition for certiorari

The petition shall be filed not later than sixty (60) days from notice
of judgment, order or resolution. In case a motion for reconsideration or
new trial is timely filed, whether such motion is required or not, the sixty
(60) day period shall be counted from notice of the denial of said motion
(Section 4, Rule 65, 1997 Revised Rules of Civil Procedure).
The 60-day period starts to run from the date petitioner receives
the assailed judgment, final order or resolution, or the denial of the motion
for reconsideration or new trial timely filed, whether such motion is
required or not (Vinuya vs. Secretary Romulo,).
To establish the timeliness of the petition for certiorari, the date of
receipt of the assailed judgment, final order or resolution or the denial of
the motion for reconsideration or new trial must be stated in the petition;
otherwise, the petition for certiorari must be dismissed. The importance of
the dates cannot be understated, for such dates determine the timeliness of
the filing of the petition for certiorari (Vinuya vs. Secretary Romulo).

D. Three (3) essential dates that must be stated in a petition for


certiorari brought under Rule 65
First, the date when notice of the judgment or final order or
resolution was received; second, when a motion for new trial or
reconsideration was filed; and third, when notice of the denial thereof was
received. Failure of petitioner to comply with this requirement shall be
sufficient ground for the dismissal of the petition. Substantial compliance
will not suffice in a matter involving strict observance with the Rules
(Tambong vs. R. Jorge Development Corporation).
The requirement of setting forth the three (3) dates in a petition for
certiorari under Rule 65 is for the purpose of determining its timeliness.
Such a petition is required to be filed not later than sixty (60) days from
notice of the judgment, order or Resolution sought to be assailed.
Therefore, that the petition for certiorari was filed forty-one (41) days
from receipt of the denial of the motion for reconsideration is hardly
relevant. The Court of Appeals was not in any position to determine when
this period commenced to run and whether the motion for reconsideration
itself was filed on time since the material dates were not stated. It should

not be assumed that in no event would the motion be filed later than
fifteen (15) days. Technical rules of procedure are not designed to frustrate
the ends of justice. These are provided to effect the proper and orderly
disposition of cases and thus effectively prevent the clogging of court
dockets. Utter disregard of the Rules cannot justly be rationalized by
harking on the policy of liberal construction (Santos vs. CA,).

E. Motion for reconsideration is required before filing a petition


for certiorari under Rule 65 in the Higher Court; exception
Petition for certiorari before a Higher Court will not prosper unless
the inferior court has been given, through a motion for reconsideration, a
chance to correct the errors imputed to it. EXCEPTIONS: (1) when the
issue raised is purely of law; (2) when public interest is involved, or (3) in
case of urgency (Philippine International Trading Corporation vs.
COA,).

F. Order denying the demurrer to evidence reviewable by


Certiorari under Section 1, Rule 65 of the Rules of Court
Generally, the order denying the motion for leave of court to file
demurrer to evidence or the demurrer itself shall not be reviewable by
appeal or by certiorari before judgment,25 because action on a demurrer or
on a motion to dismiss rests on the sound exercise of judicial discretion. 26
It is an interlocutory order, not appealable neither can it be the subject of a
petition for certiorari.27
However, this admits an exception: when the denial of a demurrer
to evidence is attended by grave abuse of discretion, patently erroneous or
25

Section 23, Rule 119, 2000 Revised Rules of Criminal Procedure.


Tan vs. Court of Appeals, 347 Phil. 320, 329 (1997); Bernardo vs. Court of Appeals,
344 Phil. 335, 346 (1997) cited in Nicolas vs. Sandiganbayan and the companion case,
G.R. Nos. 175930-31, February 11, 2008. Also in People vs. Singh, G.R. No. 129782,
June 29, 2001, 360 SCRA 404; People vs. Mercado, No. L-33492, March 30, 1988, 159
SCRA 453 cited in People vs. Almendras, G.R. No. 145915, April 24, 2003.
27
David vs. Rivera, 464 Phil. 1006; Tadeo vs. People, 360 Phil. 914, 919 (1998); Cruz vs.
People, 363 Phil. 156; Katigbak vs. Sandiganbayan, 453 Phil. 515 cited in Nicolas vs.
Sandiganbayan and the companion case, G.R. Nos. 175930-31, February 11, 2008.
26

issued with grave abuse of discretion.28 Though interlocutory in character,


an order denying a demurrer to evidence may be the subject of a certiorari
proceeding, provided the petitioner can show that it was issued with grave
abuse of discretion; and that appeal in due course is not plain, adequate or
speedy under the circumstances. When the plaintiffs evidence is utterly
and patently insufficient to prove the complaint, it would be capricious for
a trial judge to deny the demurrer and to require the defendant to present
evidence to controvert a non- existing case. The denial of the demurrer to
evidence will constitute an unwelcome imposition on the courts docket
and an assault on the defendants resources and peace of mind, and if
denied, it effectively denies justice.29
Interlocutory refers to intervening developments between the
commencement of a suit and its complete termination; hence, it is a
development that does not end the whole controversy. An interlocutory
order merely rules on an incidental issue and does not terminate or
finally dispose of the case; it leaves something to be done before the case
is finally decided on the merits (Marmot vs. Anacay)
.
G. A one liner sentence denying a motion for reconsideration to
the final order or judgment for lack of merit is not
certiorariable (Rule 65)
A one liner sentence denying a motion for reconsideration to the
final order or judgment for lack of merit is not certiorariable.
The filing of a motion to amend order denying the reconsideration
on ground of null and void as it does not conform to the requirements of
Section 14, Article VIII of the Constitution and Section 1, Rule 36 of the
1997 Revised Rules of Civil Procedure will not and does not toll the
running of the period towards finality. The aggrieved party may avail any
of the following remedies:
1. He may file a Motion for New Trial under Rule 37; or

28

People vs. Ong, G.R. No. 140904, October 9, 2000 citing Cruz vs. People (303 SCRA
533 [1999]).
29
Choa vs. Choa, G.R. No. 143376, November 26, 2002.

2. He may file a Motion for Reconsideration, also under


Rule 37; or
3. He may file an Appeal under Rule 41; or
4. He may file a petition for Relief from Judgment under
Rule 38, or;
5. He may file a petition for Annulment of Judgment under
Rule 47.
A decision that has acquired finality becomes immutable and
unalterable and may no longer be modified in any respect even if the
modification is intended to correct erroneous conclusions of fact or law
and whether it will be made by the court that rendered it or by the highest
court of the land.30
0-0-0

APPEAL (RULE 41) VS. CERTIORARI (RULE 65)


An appeal may be taken from a judgment or final
order that completely disposes of the case or of a particular
matter therein when declared by these Rules to be
appealable (Section 1, Rule 41, 1997 Revised Rules of
Civil Procedure).

When any tribunal, board or officer exercising


judicial or quasi-judicial functions has acted without or in
excess of jurisdiction, or with grave abuse of discretion

30

Sofio vs. Valenzuela, G.R. No. 157810, February 15, 2012.

amounting to lack or excess of jurisdiction, and there is no


appeal, or any plain, speedy and adequate remedy in the
ordinary course of law, a person aggrieved thereby may file
a verified petition in the proper court, alleging the facts
with certainty and praying that judgment be rendered
annulling or modifying the proceedings of such tribunal,
board or officer, and granting such incidental reliefs as law
and justice may require (Section 1, Rule 65, 1997 Revised
Rules of Civil Procedure).
Tible & Tible Company, Inc. vs. Royal Savings and Loan Association,
G.R. No. 155806, April 8, 2008 citing Madrigal Transport, Inc. vs.
Lapanday Holdings Corporation, G.R. No. 156067, August 11, 2004 d
discussed the differences between the remedies provided under Rule 41
and Rule 65, thus:
Appeal and Certiorari Distinguished
Between an appeal and a petition for certiorari, there
are substantial distinctions which shall be explained below.
As to the Purpose. Certiorari is a remedy designed
for the correction of errors of jurisdiction, not errors of
judgment. In Pure Foods Corporation v. NLRC, we
explained the simple reason for the rule in this light:
When a court exercises its jurisdiction, an
error committed while so engaged does not
deprive it of the jurisdiction being exercised
when the error is committed. If it did, every
error committed by a court would deprive it of
its jurisdiction and every erroneous judgment
would be a void judgment. This cannot be
allowed. The administration of justice would
not survive such a rule. Consequently, an error
of judgment that the court may commit in the
exercise of its jurisdiction is not correctable
through the original civil action of certiorari.

The supervisory jurisdiction of a court over the


issuance of a writ of certiorari cannot be exercised for the
purpose of reviewing the intrinsic correctness of a
judgment of the lower court on the basis either of the law
or the facts of the case, or of the wisdom or legal soundness
of the decision. Even if the findings of the court are
incorrect, as long as it has jurisdiction over the case, such
correction is normally beyond the province of certiorari.
Where the error is not one of jurisdiction, but of an error of
law or fact a mistake of judgment appeal is the remedy.
As to the Manner of Filing. Over an appeal, the CA
exercises its appellate jurisdiction and power of review.
Over a certiorari, the higher court uses its original
jurisdiction in accordance with its power of control and
supervision over the proceedings of lower courts. An
appeal is thus a continuation of the original suit, while a
petition for certiorari is an original and independent action
that was not part of the trial that had resulted in the
rendition of the judgment or order complained of. The
parties to an appeal are the original parties to the action. In
contrast, the parties to a petition for certiorari are the
aggrieved party (who thereby becomes the petitioner)
against the lower court or quasi-judicial agency, and the
prevailing parties (the public and the private respondents,
respectively).
As to the Subject Matter. Only judgments or final
orders and those that the Rules of Court so declare are
appealable. Since the issue is jurisdiction, an original
action for certiorari may be directed against an
interlocutory order of the lower court prior to an appeal
from the judgment; or where there is no appeal or any
plain, speedy or adequate remedy.
As to the Period of Filing. Ordinary appeals should
be filed within fifteen days from the notice of judgment or
final order appealed from. Where a record on appeal is
required, the appellant must file a notice of appeal and a

record on appeal within thirty days from the said notice of


judgment or final order. A petition for review should be
filed and served within fifteen days from the notice of
denial of the decision, or of the petitioners timely filed
motion for new trial or motion for reconsideration. In an
appeal by certiorari, the petition should be filed also within
fifteen days from the notice of judgment or final order, or
of the denial of the petitioners motion for new trial or
motion for reconsideration.
On the other hand, a petition for certiorari should be
filed not later than sixty days from the notice of judgment,
order, or resolution. If a motion for new trial or motion for
reconsideration was timely filed, the period shall be
counted from the denial of the motion.
As to the Need for a Motion for Reconsideration.
A motion for reconsideration is generally required prior to
the filing of a petition for certiorari, in order to afford the
tribunal an opportunity to correct the alleged errors. Note
also that this motion is a plain and adequate remedy
expressly available under the law. Such motion is not
required before appealing a judgment or final order.
With these distinctions, it is plainly discernible why a
party is precluded from filing a petition for certiorari when
appeal is available, or why the two remedies of appeal and
certiorari are mutually exclusive and not alternative or
successive. Where appeal is available, certiorari will not
prosper, even if the ground availed of is grave abuse of
discretion.
---0---

2. PROHIBITION
A. The Rule

When the proceedings of any tribunal, corporation, board, officer


or person, whether exercising judicial, quasi-judicial or ministerial
functions, are without or in excess of its or his jurisdiction, or with grave
abuse of discretion amounting to lack or excess of jurisdiction, and there is
no appeal or any other plain, speedy, and adequate remedy in the ordinary
course of law, a person aggrieved thereby may file a verified petition in
the proper court, alleging the facts with certainty and praying that
judgment be rendered commanding the respondent to desist from further
proceedings in the action or matter specified therein, or otherwise granting
such incidental reliefs as law and justice may require (Section 2, Rule 65,
1997 Revised Rules of Civil Procedure).
The petition shall likewise be accompanied by a certified true copy
of the judgment, order or resolution subject thereof, copies of all pleadings
and documents relevant and pertinent thereto, and a sworn certification of
non-forum shopping as provided in the third paragraph of section 3, Rule
46 (Paragraph 2, Section 2, Rule 65, 1997 Revised Rules of Civil
Procedure).

B. Writ of prohibition an extraordinary writ


A writ of prohibition is an extraordinary writ. It may be issued
only in the absence of a plain, speedy and adequate remedy in the ordinary
course of law Purok Bagong Silang Association, Inc. vs. Judge Yuipco,).

C. General rule, a writ of prohibition cannot be issued against an


inferior court, exception
A writ of prohibition will not be issued against an inferior court
unless the attention of the court whose proceedings are sought to be stayed
has been called to the alleged lack or excess of jurisdiction. The
foundation of this rule is the respect and consideration due to the lower
court and the expediency of preventing unnecessary litigation; it cannot be
presumed that the lower court would not properly rule on a jurisdictional

objection if it were property presented to it (Esquivel vs. Hon.


Ombudsman,).

D. Prohibition lies against judicial or ministerial functions, but


not against legislative or quasi-legislative functions
Prohibition lies against judicial or ministerial functions, but not
against legislative or quasi-legislative functions. Generally, the purpose of
a writ of prohibition is to keep a lower court within the limits of its
jurisdiction in order to maintain the administration of justice in orderly
channels. Prohibition is the proper remedy to afford relief against
usurpation of jurisdiction or power by an inferior court, or when, in the
exercise of jurisdiction in handling matters clearly within its cognizance
the inferior court transgresses the bounds prescribed to it by the law, or
where there is no adequate remedy available in the ordinary course of law
by which such relief can be obtained. Where the principal relief sought is
to invalidate an IRR, petitioners remedy is an ordinary action for its
nullification, an action which properly falls under the jurisdiction of the
Regional Trial Court. In any case, petitioners allegation that respondents
are performing or threatening to perform functions without or in excess of
their jurisdiction may appropriately be enjoined by the trial court through
a writ of injunction or a temporary restraining order (Holy Spirit
Homeowners Association vs. Defensor,).
E. Prohibition does not lie to restrain an act which is already a fait
accompli
Prohibition does not lie to restrain an act which is already a fait
accompli. The operative fact doctrine protecting vested rights bars the
grant of the writ of prohibition (Chavez vs. NHA,).

F. Petition for prohibition venue of action


The petition for prohibition filed by respondents is a special civil
action which may be filed in the Supreme Court, the Court of Appeals, the
Sandiganbayan or the regional trial court, as the case may be. It is also a

personal action because it does not affect the title to, or possession of real
property, or interest therein. Thus, it may be commenced and tried where
the plaintiff or any of the principal plaintiffs resides, or where the
defendant or any of the principal defendants resides, at the election of the
plaintiff. The choice of venue is sanctioned by Section 2, Rule 4 of the
Rules of Court (GSIS Board of Trustees vs. Velasco,).
---0---

1. MANDAMUS
A. The Rule
When any tribunal, corporation, board, officer or person
unlawfully neglects the performance of an act which the law specifically
enjoins as a duty resulting from an office, trust, or station, or unlawfully
excludes another from the use and enjoyment of a right or office to which
such other is entitled, and there is no other plain, speedy and adequate
remedy in the ordinary course of law, the person aggrieved thereby may
file a verified petition in the proper court, alleging the facts with certainty
and praying that judgment be rendered commanding the respondent,
immediately or at some other time to be specified by the court, to do the
act required to be done to protect the rights of the petitioner, and to pay the
damages sustained by the petitioner by reason of the wrongful acts of the
respondent (Section 3, Rule 65, 1997 Revised Rules of Civil Procedure).
The petition shall also contain a sworn certification of non-forum
shopping as provided in the third paragraph of section 3, Rule 46
(Paragraph 2, Section 3, Rule 65, 1997 Revised Rules of Civil
Procedure).

B. Concept of ministerial duty


A ministerial duty is one which is so clear and specific as to leave
no room for the exercise of discretion in its performance. It is a duty
which an officer performs in a given state of facts in a prescribed manner
in obedience to the mandate of legal authority, without regard to the

exercise of his/her own judgment upon the propriety of the act done
(Symaco v. Hon. Aquino,).
Mandamus lies to compel the performance, when refused, of a
ministerial duty, but not to compel the performance of a discretionary
duty. A purely ministerial act or duty is one which an officer or tribunal
performs in a given state of facts, in a prescribed manner, in obedience to
the mandate of a legal authority, without regard to or the exercise of his
own judgment upon the propriety or impropriety of the act done. The duty
is ministerial only when the discharge of the same requires neither the
exercise of official discretion or judgment. When an official is required
and authorized to do a prescribed act upon a prescribed contingency, his
functions are ministerial only, and mandamus may be issued to control his
action upon the happening of the contingency (Paloma vs. Mora,).
The remedy of mandamus is available only to compel the
performance of a ministerial duty. The distinction between a ministerial
and discretionary act is well delineated. A purely ministerial act or duty is
one which an officer or tribunal performs in a given state of facts, in a
prescribed manner, in obedience to the mandate of a legal authority,
without regard to or the exercise of his own judgment upon the propriety
or impropriety of the act done. If the law imposes a duty upon a public
officer and gives him the right to decide how or when the duty shall be
performed, such duty is discretionary and not ministerial. The duty is
ministerial only when the discharge of the same requires neither the
exercise of official discretion or judgment (Torregoza vs. CSC,).
For a writ of mandamus to be issued, it is essential that petitioner
should have a clear legal right to the thing demanded and it must be the
imperative duty of the respondent to perform the act required. The writ
neither confers powers nor imposes duties. It is simply a command to
exercise a power already possessed and to perform a duty already
imposed. Mandamus applies as a remedy only where petitioners right is
founded clearly in law and not when it is doubtful. The writ will not be
granted where its issuance would be unavailing, nugatory, or useless
(Philippine Coconut Authority vs. Primex Coco Products).
If the law imposes a duty upon a public officer and gives him the
right to decide how or when the duty shall be performed, such duty is

discretionary and not ministerial (Philippine Coconut Authority vs.


Primex Coco Products, Inc,).
Mandamus is an extraordinary writ and discretionary remedy and
should not be granted when it will achieve no beneficial result such as
when act sought to be compelled has been performed (Philippine Coconut
Authority vs. Primex Coco Products,).
Mandamus will not be issued to compel the renewal of a license
for a period which has expired. If the right sought to be enforced by writ
of mandamus is or has become a mere abstract right, enforcement of
which will be of no substantial or practical benefit to the plaintiff, the writ
will not issue though the applicant would otherwise be entitled to it. To
warrant the issuance of a writ of mandamus, it must appear that the writ
will be effectual as a remedy, it should be denied where it would be
useless by reason of events occurring subsequent to commencement
proceedings (Philippine Coconut Authority vs. Primex Coco Products,
Inc,).
Mandamus is never granted to compel the performance of an act
until there has been an actual, as distinguished from an anticipated, refusal
to act. This is true even if there is a strong presumption that the persons
whom it is sought to coerce by the writ will refuse to perform their duty
when the proper time arrives. Its function is to compel the performance of
a present existing duty as to which there is default. It is not granted to take
effect prospectively, and it contemplates the performance of an act which
is incumbent on respondent when the application for a writ is made
(Philippine Coconut Authority vs. Primex Coco Products,).
---0---

FINALITY OF JUDGMENT
AND ENTRY OF JUDGMENT
A. Doctrine of finality of judgment
The doctrine of finality of judgments is grounded on fundamental
considerations of public policy and sound practice that at the risk of

occasional errors, the judgments of courts must become final at some


definite date fixed by law (Torno vs. IAC, ).
After the lapse of the 15-day reglementary period to appeal, an
order becomes executory, and it goes beyond the jurisdiction of the court
that rendered it to further amend or revoke. The subsequent filing of a
Motion for Reconsideration cannot disturb the finality of a judgment, nor
restore jurisdiction that had already been lost. A final and executor
judgment or order cannot be modified in any respect, even if the
modification sought is to correct an erroneous conclusion by the court that
rendered it (Tupaz vs. Judge Apurillo,).

B. Date of entry of judgment, determinative of


1. for purposes of ensuring the finality of judgment for
execution purposes;
2. for purposes of determining the period to file petition for
relief from judgment; and
3. for purposes of determining the period to file action for
revival of judgment.
The 60-day period must be counted after petitioner learns of the
judgment or final order, and the 6-month period is counted from the
finality of judgment or final order. These are the so-called double period.
These double periods are both inextendible and uninterruptible; is
jurisdictional and should be strictly complied with (Madarang vs.
Morales,). The 60-day reglementary period shall reckone from the time
the partys counsel receives notice of the decision for notice to counsel of
the decision is notice to the party for purposes of Section 3 of Rule 38
(Mercury Drugs Corporation vs. CA,).
---0---

LIBERAL INTERPRETATION OF THE RULES

A. The Rule
The 1997 Revised Rules of Civil Procedure shall be liberally
construed in order to promote their objective of securing a just, speedy and
inexpensive disposition of every action and proceeding (Section 6, Rule 1,
1997 Revised Rules of Civil Procedure).

B. Liberal interpretation on right to appeal


The right to appeal is not a natural right and is not part of due
process, but merely a statutory privilege to be exercised only in
accordance with the law. Being the party who sought to appeal, he must
comply with the requirements of the relevant rules; otherwise, he would
lose the statutory right to appeal (Magsino vs. Ocampo,).
The procedures regulating appeals as laid down in the Rules of
Court must be followed because strict compliance with them is
indispensable for the orderly and speedy disposition of justice (Polintan
vs. People,).
For appealed cases, the High Court in Galvez vs. CA, , recognized
three (3) guideposts for the Court of Appeals to consider in determining
whether or not the rules of procedures should be relaxed:
First, not all pleadings and parts of case records are
required to be attached to the petition. Only those which are
relevant and pertinent must accompany it. The test of
relevancy is whether the document in question will support
the material allegations in the petition, whether said
document will make out a prima facie case of grave abuse
of discretion as to convince the court to give due course to
the petition.
Second, even if a document is relevant and pertinent
to the petition, it need not be appended if it is shown that
the contents thereof can also found in another document
already attached to the petition. Thus, if the material
allegations in a position paper are summarized in a

questioned judgment, it will suffice that only a certified


true copy of the judgment is attached.
Third, a petition lacking an essential pleading or part
of the case record may still be given due course or
reinstated (if earlier dismissed) upon showing that
petitioner later submitted the documents required, or that it
will serve the higher interest of justice that the case be
decided on the merits.

C. Liberal interpretation on strict adherence to procedural


rules
The following are the justification for a court to suspend a strict
adherence to procedural rules:
(a) matters of life, liberty, honor or property in involved;
(b) the existence of special or compelling circumstances;
(c) the merits of the case;
(d) a cause not entirely attributable to the fault or
negligence of the party favored by the suspension of the
rules;
(e) a lack of any showing that the review sought is merely
frivolous and dilatory; and
(f) the fact that the other party will not be unjustly
prejudiced thereby (Sanchez vs. CA,).
---0---

You might also like